KLEE 2023 LLB 3 Year Question Paper (Available) Download Solution PDF with Answer Keys

Shivam Yadav's profile photo

Shivam Yadav

Updated 3+ months ago

KLEE 2023 LLB 3 Year Question paper with Solution PDF is available for download. The exam was successfully organized by Commissioner of Entrance Examinations (CEE). The question paper comprised a total of 200 questions divided among four sections.

KLEE 2023 LLB 3 Year Question Paper with Solution PDF

KLEE 2023 LLB 3 Year Question Paper with Solution PDF Download PDF Check Solutions
KLEE 2023 LLB Question paper

Question 1:

Find out the incorrectly spelled word

  • (A) Accommodate
  • (B) Tresspass
  • (C) Privilege
  • (D) Perseverance
Correct Answer: (B) Tresspass
View Solution




This is a spelling-based question that tests vocabulary knowledge. The goal is to identify which of the given words is spelled incorrectly.


Step 1: Examine Option A - ``Accommodate''

``Accommodate'' is correctly spelled with a double ``c'' and a double ``m''. This word often causes confusion, but the correct form is ``accommodate''. So, Option A is correct.


Step 2: Examine Option B - ``Tresspass''

``Tresspass'' is incorrect. The correct spelling is ``trespass'', with a single ``s'' after the ``tre''. The double ``s'' in ``tresspass'' makes it a misspelling. Therefore, Option B is the incorrectly spelled word.


Step 3: Examine Option C - ``Privilege''

``Privilege'' is correctly spelled. It refers to a special right or advantage. Hence, Option C is correct.


Step 4: Examine Option D - ``Perseverance''

``Perseverance'' is also correctly spelled. It refers to persistence in doing something despite difficulty or delay. So, Option D is correct.


Conclusion: The only incorrectly spelled word among the options is Option B: ``Tresspass'', which should be ``trespass''. Quick Tip: For spelling-based questions, always check for common double letter confusions (e.g., ``accommodate'' vs ``acommodate'') and silent letters. Reading and writing frequently helps internalize correct spellings.


Question 2:

Find out the incorrectly spelled word

  • (A) Conscientious
  • (B) Bureaucracy
  • (C) Millenium
  • (D) Exaggerate
Correct Answer: (C) Millenium
View Solution




This question tests spelling knowledge, particularly of commonly misspelled English words. Let's review each option.


Step 1: Examine Option A - ``Conscientious''

``Conscientious'' is correctly spelled. It means wishing to do one's work or duty well and thoroughly. Though complex, this is the proper spelling. Option A is correct.


Step 2: Examine Option B - ``Bureaucracy''

``Bureaucracy'' is correctly spelled. It refers to a system of government or business that is highly structured and governed by many rules and procedures. Option B is correct.


Step 3: Examine Option C - ``Millenium''

``Millenium'' is incorrect. The correct spelling is ``millennium'' — with double ``n'' and double ``l''.

This word is derived from Latin: ``mille'' meaning thousand and ``annus'' meaning year. Hence, the correct spelling requires two ``n''s. Option C is the incorrectly spelled word.


Step 4: Examine Option D - ``Exaggerate''

``Exaggerate'' is spelled correctly. It means to represent something as being larger or more significant than it actually is. So Option D is also correct.


Conclusion: The incorrectly spelled word is Option C: ``Millenium'', which should be ``millennium''. Quick Tip: To improve spelling accuracy, remember that words derived from Latin (like ``millennium'') often retain root components.
Use memory tricks: ``Millennium'' has two ``l''s and two ``n''s — think ``million'' and ``annum''.


Question 3:

Find out the incorrectly spelled word

  • (A) Recieve
  • (B) Desiccate
  • (C) Exquisite
  • (D) Imminent
Correct Answer: (A) Recieve
View Solution




Step 1: Examine Option A - ``Recieve''

``Recieve'' is incorrect. The correct spelling is ``receive'', following the ``i before e except after c'' rule.


Step 2: Examine Option B - ``Desiccate''

``Desiccate'' is correctly spelled. It means to remove moisture.


Step 3: Examine Option C - ``Exquisite''

``Exquisite'' is correctly spelled. It means extremely beautiful and delicate.


Step 4: Examine Option D - ``Imminent''

``Imminent'' is correctly spelled. It means about to happen.


Thus, the misspelled word is Option A: ``Recieve'' \(\rightarrow\) ``Receive''. Quick Tip: Remember: ``i before e except after c'' — helps in spelling words like ``receive'' and ``perceive''.


Question 4:

Find out the incorrectly spelled word

  • (A) Grateful
  • (B) Rhythm
  • (C) Liaison
  • (D) Acquaintence
Correct Answer: (D) Acquaintence
View Solution




Step 1: Examine Option A - ``Grateful''

``Grateful'' is correctly spelled. It means feeling or showing appreciation.


Step 2: Examine Option B - ``Rhythm''

``Rhythm'' is correctly spelled, though tricky, it has no vowels after the ``h''.


Step 3: Examine Option C - ``Liaison''

``Liaison'' is correctly spelled. It means a link or communication between groups.


Step 4: Examine Option D - ``Acquaintence''

``Acquaintence'' is incorrect. The correct spelling is ``acquaintance''. The ending should be ``-ance'', not ``-ence''.


Thus, the misspelled word is Option D: ``Acquaintence'' \(\rightarrow\) ``Acquaintance''. Quick Tip: For tricky endings like ``-ance'' vs ``-ence'', try memorizing common patterns or use flashcards to practice frequently confused words.


Question 5:

Find out the incorrectly spelled word

  • (A) Indispensable
  • (B) Embarass
  • (C) Discrepancy
  • (D) Acquire
Correct Answer: (B) Embarass
View Solution




Step 1: Examine Option A - ``Indispensable''

``Indispensable'' is correctly spelled. It means absolutely necessary.


Step 2: Examine Option B - ``Embarass''

``Embarass'' is incorrect. The correct spelling is ``embarrass'' — with two ``r''s and two ``s''s.


Step 3: Examine Option C - ``Discrepancy''

``Discrepancy'' is correctly spelled. It means a lack of compatibility or similarity.


Step 4: Examine Option D - ``Acquire''

``Acquire'' is correctly spelled. It means to gain or obtain something.


So, the incorrect spelling is Option B: ``Embarass'' \(\rightarrow\) ``Embarrass''. Quick Tip: To remember ``embarrass'', think: ``It’s so embarrassing, it needs two r’s and two s’s to emphasize it!’’


Question 6:

Choose the word nearest in meaning to the word given in capital letters: CIRCUMSPECT

  • (A) Polite
  • (B) Delicate
  • (C) Reckless
  • (D) Cautious
Correct Answer: (D) Cautious
View Solution




The word ``circumspect'' means ``wary and unwilling to take risks''. It refers to being careful, especially before acting.


Step 1: Analyze the meaning of each option:

- ``Polite'': means respectful or considerate, not necessarily related to carefulness.

- ``Delicate'': means fragile or sensitive, again not quite fitting the idea of caution.

- ``Reckless'': means without thinking or caring about consequences — opposite of circumspect.

- ``Cautious'': means careful to avoid potential problems or dangers, which is synonymous with circumspect.


Hence, the correct synonym for ``circumspect'' is ``cautious''. Quick Tip: When you see a word like ``circumspect'', look for words indicating carefulness, watchfulness, or caution.


Question 7:

Choose the word nearest in meaning to the word given in capital letters: EQUANIMITY

  • (A) Strict
  • (B) Composure
  • (C) Silent
  • (D) Symmetry
Correct Answer: (B) Composure
View Solution




The word ``equanimity'' refers to mental calmness, stability, or composure, especially in difficult situations.


Step 1: Analyze the options:

- ``Strict'': means firm or harsh in discipline — not related to mental calmness.

- ``Composure'': means the state of being calm and in control — a direct synonym.

- ``Silent'': refers to not speaking — may seem related but doesn’t reflect inner calmness.

- ``Symmetry'': refers to balanced proportions — unrelated to emotional state.


Thus, the correct synonym for ``equanimity'' is ``composure''. Quick Tip: ``Equanimity'' is often used in stressful situations — look for options implying emotional control or calm.


Question 8:

Choose the word nearest in meaning to the word given in capital letters: PUGNACIOUS

  • (A) Talented
  • (B) Mediocre
  • (C) Aggressive
  • (D) Impulsive
Correct Answer: (C) Aggressive
View Solution




``Pugnacious'' means eager or quick to argue or fight. It's derived from the Latin ``pugnare'' meaning ``to fight''.


Step 1: Understand the options:

- ``Talented'': refers to having skill — unrelated to combativeness.

- ``Mediocre'': means average — not a synonym.

- ``Aggressive'': means inclined to fight or attack — a synonym.

- ``Impulsive'': means acting without thought — may overlap slightly, but not a direct match.


Therefore, the correct synonym for ``pugnacious'' is ``aggressive''. Quick Tip: ``Pugnacious'' contains ``pugn-'' (Latin for fight) — think of a ``pugilist'' (a boxer). Look for aggressive or combative meanings.


Question 9:

Choose the word nearest in meaning to the word given in capital letters: ELOQUENT

  • (A) Expensive
  • (B) Mysterious
  • (C) Anxious
  • (D) Articulate
Correct Answer: (D) Articulate
View Solution




The word ``eloquent'' means fluent or persuasive in speaking or writing. It describes someone who can express ideas clearly and effectively.


Step 1: Understand the options:

- ``Expensive'': Related to cost, not communication.

- ``Mysterious'': Implies secrecy or being hard to understand — not related to speaking well.

- ``Anxious'': Refers to being worried or uneasy — unrelated.

- ``Articulate'': Means able to speak clearly and effectively — matches the meaning of ``eloquent''.


Thus, ``articulate'' is the correct synonym for ``eloquent''. Quick Tip: Think of a speech: an ``eloquent'' speaker is ``articulate'', not confused or vague.


Question 10:

Choose the word nearest in meaning to the word given in capital letters: INTRANSIGENT

  • (A) Flexible
  • (B) Transparent
  • (C) Stubborn
  • (D) Unpleasant
Correct Answer: (C) Stubborn
View Solution




The word ``intransigent'' means unwilling to change one’s views or agree — firmly uncompromising.


Step 1: Analyze the options:

- ``Flexible'': Opposite of intransigent — open to change.

- ``Transparent'': Means open or clear — not related.

- ``Stubborn'': Unwilling to change opinion — directly matches ``intransigent''.

- ``Unpleasant'': Means disagreeable — unrelated to firmness of belief.


So, ``stubborn'' is the best synonym for ``intransigent``. Quick Tip: ``Intransigent'' = not transiting/changing = stubborn. Look for clues in prefixes and roots.


Question 11:

Choose the word nearest in meaning to the word given in capital letters: OSTENTATIOUS

  • (A) Showy
  • (B) Modest
  • (C) Ambiguous
  • (D) Selfish
Correct Answer: (A) Showy
View Solution




``Ostentatious'' means characterized by vulgar or pretentious display — designed to impress or attract notice.


Step 1: Consider the meanings:

- ``Showy'': Means flashy or trying to attract attention — matches ``ostentatious''.

- ``Modest'': Humble or restrained — opposite in meaning.

- ``Ambiguous'': Unclear or having more than one meaning — unrelated.

- ``Selfish'': Concerned with oneself — different concept entirely.


Hence, the correct synonym is ``showy``. Quick Tip: ``Ostentatious'' = Over-the-top show. Think of people who flaunt wealth — they are being showy.


Question 12:

Choose the word nearest in meaning to the word given in capital letters: INDUBITABLE

  • (A) Difficult
  • (B) Inflexible
  • (C) Undoubted
  • (D) Fierce
Correct Answer: (C) Undoubted
View Solution




The word ``indubitable'' means something that is unquestionable or cannot be doubted. It is used to express absolute certainty and surety.


Step 1: Understand the meaning of ``indubitable''

``Indubitable'' comes from the Latin word ``indubitabilis'', meaning ``not able to be doubted''. It is used to refer to facts or truths that are certain and beyond question.


Step 2: Analyze Option A - ``Difficult''

``Difficult'' means something that requires effort or is hard to accomplish.

This has no relation to certainty or doubt. So, Option A is incorrect.


Step 3: Analyze Option B - ``Inflexible''

``Inflexible'' means rigid or not willing to change.

It is not related to the concept of doubt or certainty. Hence, Option B is incorrect.


Step 4: Analyze Option C - ``Undoubted''

``Undoubted'' means not questioned or disputed.

This is a direct synonym of ``indubitable''. So, Option C is correct.


Step 5: Analyze Option D - ``Fierce''

``Fierce'' describes intensity or aggression.

It has no relevance to the idea of being certain or indisputable. Thus, Option D is incorrect.


Conclusion: The word ``undoubted'' best matches the meaning of ``indubitable''. Quick Tip: Break down complex words into parts: ``in-'' means ``not'' and ``dubitable'' means ``doubtful''. So ``indubitable'' = ``not doubtful''.
Look for words like ``certain'', ``sure'', or ``undoubted'' in synonym questions.


Question 13:

Convert the sentence given in the question into Indirect speech.

Sara said to her mother, ``How do you do all these things together?''

  • (A) Sara asked her mother how she do all those things together.
  • (B) Sara asked her mother how she can do all these things together.
  • (C) Sara asked her mother how she did all those things together.
  • (D) Sara asked her mother how did she do all those things together.
Correct Answer: (C) Sara asked her mother how she did all those things together.
View Solution




This is a sentence transformation question involving direct to indirect speech.


Step 1: Identify the reporting verb and the structure

The reporting verb is ``said to'', which changes to ``asked'' in the case of a question.


Step 2: Change the tense

Since the reporting verb is in the past tense (``said''), the verb in the reported speech must also change from present (``do'') to past (``did'').


Step 3: Adjust the pronouns and time expressions

``You'' changes to ``she'' (referring to the mother), and ``these'' changes to ``those''.


Step 4: Avoid interrogative word order in indirect speech

The structure ``how did she do...'' (Option D) retains interrogative form and is incorrect.

Option C uses correct word order and tense: ``how she did all those things together''.


Conclusion: Option C is grammatically correct and follows all rules for converting into indirect speech. Quick Tip: When converting questions to indirect speech, use the reporting verb ``asked'', shift the tense back, and avoid using interrogative word order.


Question 14:

Convert the sentence given in the question into Indirect speech.

John said, ``I am going to the store.''

  • (A) John said that he is going to the store.
  • (B) John said that he will go to the store.
  • (C) John said that he was going to the store.
  • (D) John said that he goes to the store.
Correct Answer: (C) John said that he was going to the store.
View Solution




Step 1: Identify the reporting verb and speech type

The verb ``said'' is in the past tense and introduces a statement.


Step 2: Convert present continuous tense to past continuous

The original sentence ``I am going...'' (present continuous) changes to ``he was going...'' (past continuous).


Step 3: Adjust the pronoun

``I'' becomes ``he'' referring to John.


Step 4: Review other options

Option A retains the present tense ``is going'' — incorrect.

Option B changes to future ``will go'' — incorrect tense shift.

Option D uses the simple present ``goes'' — inappropriate for this context.


Conclusion: Option C correctly applies the rules of indirect speech. Quick Tip: When reporting statements in present continuous, convert to past continuous. Also change pronouns appropriately.


Question 15:

Convert the sentence given in the question into Indirect speech.

They said, ``We won the game!''

  • (A) They exclaimed that they had won the game.
  • (B) They said that they won the game.
  • (C) They exclaimed that they won the game.
  • (D) They said that they had won in the game.
Correct Answer: (A) They exclaimed that they had won the game.
View Solution




This is a case of transforming an exclamatory sentence into indirect speech.


Step 1: Identify the reporting verb

Since the original sentence is an exclamation (``We won the game!''), the reporting verb changes to ``exclaimed''.


Step 2: Change the tense

The sentence is in simple past (``won''), which should be changed to past perfect (``had won'') in indirect speech.


Step 3: Adjust the pronouns

``We'' changes to ``they'' to maintain subject consistency.


Step 4: Evaluate the options

- Option A: Correctly uses ``exclaimed'', changes ``won'' to ``had won'', and replaces ``we'' with ``they''.

- Option B: Uses ``won'' instead of ``had won'' — incorrect tense.

- Option C: Also uses incorrect tense.

- Option D: ``Had won in the game'' is grammatically incorrect phrasing.


Conclusion: Option A is correct. Quick Tip: For exclamatory sentences in indirect speech, use reporting verbs like ``exclaimed'' and shift simple past to past perfect.


Question 16:

Convert the sentence given in the question into Indirect speech.

Sheila said, ``I don’t like pizza.''

  • (A) Sheila said that she do not like pizza.
  • (B) Sheila said that she did not like pizza.
  • (C) Sheila said that she has not liked pizza.
  • (D) Sheila said that will not like pizza.
Correct Answer: (B) Sheila said that she did not like pizza.
View Solution




Step 1: Identify the verb tense in direct speech

The original sentence is in present simple negative: ``don’t like''.


Step 2: Apply the rules of tense shift in reported speech

Since the reporting verb ``said'' is in the past tense, we change the present simple ``don’t like'' to past simple ``did not like''.


Step 3: Adjust the subject pronoun

``I'' changes to ``she'' to match the subject ``Sheila''.


Step 4: Eliminate incorrect options

- Option A: ``she do not'' is grammatically incorrect.

- Option C: Changes tense unnecessarily to present perfect.

- Option D: Changes mood to future — incorrect.


Conclusion: Option B correctly uses past simple tense and the proper subject. Quick Tip: When converting present simple to indirect speech, use past simple. ``Don’t like'' becomes ``did not like'' when reported.


Question 17:

Convert the sentence given in indirect speech into direct speech.

He asked if I wanted to go to the movies.

  • (A) He asked, ``Do you want to go to the movies?''
  • (B) He asked, ``Will you want to go to the movies?''
  • (C) He asked, ``Did you want to go to the movies?''
  • (D) He asked, ``Would you go to the movies?''
Correct Answer: (A) He asked, ``Do you want to go to the movies?''
View Solution




The indirect sentence ``He asked if I wanted to go to the movies'' is in past tense and needs to be converted back to direct speech.


Step 1: Reverse the tense shift

In reported speech, ``I wanted'' corresponds to ``Do you want'' in direct speech (present tense for habitual or current interest).


Step 2: Reverse the pronouns and word order

- ``I'' becomes ``you''

- Interrogative structure is restored: ``Do you want to go...''


Step 3: Eliminate other options

- Option B: ``Will you want'' — future form, not a correct reversal.

- Option C: ``Did you want'' — implies a completed action, which changes the meaning.

- Option D: ``Would you go'' — conditional; also changes meaning.


Thus, the correct direct speech form is Option A. Quick Tip: To convert indirect to direct speech, reverse the tense, switch the pronouns, and restore the question form for interrogatives.


Question 18:

A sentence is divided into 3 parts. Choose the incorrect part. If no error in the sentence, choose 'D'

With a frown on his face

the teacher ask the student

to leave the room

  • (A) With a frown on his face
  • (B) the teacher ask the student
  • (C) to leave the room
  • (D) No error
Correct Answer: (B) the teacher ask the student
View Solution




Step 1: Identify subject-verb agreement

``The teacher'' is a singular subject, so the verb should be ``asks'' (not ``ask'').


Step 2: Check each part

- Part A: ``With a frown on his face'' — grammatically correct.

- Part B: ``ask'' is incorrect; it should be ``asks''.

- Part C: ``to leave the room'' — perfectly fine.


Therefore, Option B contains the error. Quick Tip: Always match singular subjects with singular verbs: ``He asks'', ``She goes'', ``The teacher explains''.


Question 19:

A sentence is divided into 3 parts. Choose the incorrect part. If no error in the sentence, choose 'D'

The company announced

a new strategy to improve

the product's saleability

  • (A) The company announced
  • (B) a new strategy to improve
  • (C) the product's saleability
  • (D) No error
Correct Answer: (D) No error
View Solution




Step 1: Examine Part A - ``The company announced''

This is a correct use of past tense.


Step 2: Examine Part B - ``a new strategy to improve''

This phrase is grammatically sound.


Step 3: Examine Part C - ``the product's saleability''

``Saleability'' (also spelled ``salability'') is an acceptable word meaning marketability.


Conclusion: No grammatical error exists in any part of the sentence. Quick Tip: If you're unsure about a rare word like ``saleability'', check its usage in business or marketing contexts. It’s a valid term!


Question 20:

A sentence is divided into 3 parts. Choose the incorrect part. If no error in the sentence, choose 'D'

She looked at herself

in the mirror and

admired at her reflection

  • (A) She looked at herself
  • (B) in the mirror and
  • (C) admired at her reflection
  • (D) No error
Correct Answer: (C) admired at her reflection
View Solution




Step 1: Identify the verb usage

The verb ``admired'' is a transitive verb and does not take a preposition like ``at''.


Step 2: Correct phrasing

The correct construction is ``admired her reflection'', not ``admired at her reflection''.


Step 3: Conclusion

Only Part C has an error — the preposition ``at'' is unnecessary. Quick Tip: Some verbs like ``admire'', ``discuss'', ``consider'' do not take prepositions after them. Avoid ``admire at'', ``discuss about'', etc.


Question 21:

A sentence is divided into 3 parts. Choose the incorrect part. If no error in the sentence, choose 'D'

The dog wagged its tail

happily and jumped

onto its owner's laps

  • (A) The dog wagged its tail
  • (B) happily and jumped
  • (C) onto its owner's laps
  • (D) No error
Correct Answer: (C) onto its owner's laps
View Solution




Step 1: Check noun form

The word ``laps'' is incorrect in this context. ``Lap'' should be singular since a person has only one lap.


Step 2: Correction

It should be ``onto its owner's lap'' — singular.


Conclusion: Part C is incorrect due to wrong number (plural form). Quick Tip: Use the correct singular or plural form depending on the noun. For example: ``the boy sat on his father's lap'' (not ``laps'').


Question 22:

A sentence is divided into 3 parts. Choose the incorrect part. If no error in the sentence, choose 'D'

I need to buy

some fresh vegetable

for tonight's dinner

  • (A) I need to buy
  • (B) some fresh vegetable
  • (C) for tonight's dinner
  • (D) No error
Correct Answer: (B) some fresh vegetable
View Solution




Step 1: Understand the noun usage

``Vegetable'' in this context should be plural — ``some fresh vegetables''.


Step 2: Quantifiers with countable nouns

``Some'' is followed by a plural countable noun, so ``vegetable'' must be ``vegetables''.


Conclusion: Part B is incorrect due to incorrect number of the noun. Quick Tip: Use plural countable nouns after ``some'', ``many'', and ``a few'' — for example: ``some books'', ``a few apples'', ``many ideas''.


Question 23:

A sentence is divided into 3 parts. Choose the incorrect part. If no error in the sentence, choose 'D'

He took out a book

from his bag

and started reading on it

  • (A) He took out a book
  • (B) from his bag
  • (C) and started reading on it
  • (D) No error
Correct Answer: (C) and started reading on it
View Solution




Step 1: Identify incorrect usage

The phrase ``reading on it'' is incorrect. In English, the verb ``read'' is not followed by the preposition ``on'' in this context.


Step 2: Correction

The correct phrase should be ``started reading it'', not ``reading on it''.


Conclusion: The error lies in Part C due to incorrect prepositional usage. Quick Tip: Many transitive verbs like ``read'', ``discuss'', and ``watch'' do not take prepositions before their object.


Question 24:

Choose the nearest one word from the alternatives given below: A fear of heights

  • (A) Acrophobia
  • (B) Claustrophobia
  • (C) Arachnophobia
  • (D) Hydrophobia
Correct Answer: (A) Acrophobia
View Solution




Step 1: Understand the definition

- ``Acrophobia'' is derived from the Greek word ``akron'' (height) and ``phobia'' (fear). It means fear of heights.


Step 2: Eliminate other options

- ``Claustrophobia'': Fear of confined or closed spaces.

- ``Arachnophobia'': Fear of spiders.

- ``Hydrophobia'': Fear of water (often associated with rabies).


Conclusion: Only ``Acrophobia'' correctly refers to the fear of heights. Quick Tip: Remember:
- ``Acro'' = height,
- ``Claus'' = closed,
- ``Arachno'' = spider,
- ``Hydro'' = water.


Question 25:

Choose the nearest one word from the alternatives given below: A small group of people with shared interests or tastes

  • (A) Crew
  • (B) Assembly
  • (C) Clique
  • (D) Congregation
Correct Answer: (C) Clique
View Solution




Step 1: Define the word ``Clique''

A ``clique'' is a small, close-knit group of people who share interests and often exclude others.


Step 2: Eliminate other options

- ``Crew'': A group of people working together, especially on a ship or aircraft.

- ``Assembly'': A group of people gathered together in one place for a common purpose.

- ``Congregation'': A group gathered for religious worship.


Only ``Clique'' matches the definition of a small, selective group with shared interests. Quick Tip: Use ``clique'' for a selective, often exclusive group with common interests, especially in social contexts.


Question 26:

Find the nearest one-word from the alternatives given below: A feeling of intense anger and hostility

  • (A) Apathy
  • (B) Elation
  • (C) Contentment
  • (D) Rage
Correct Answer: (D) Rage
View Solution




Step 1: Define ``Rage''

``Rage'' means violent, uncontrollable anger — it fits the definition of ``intense anger and hostility''.


Step 2: Eliminate incorrect options

- ``Apathy'': Lack of emotion or interest.

- ``Elation'': Great happiness.

- ``Contentment'': Satisfaction and peace.


Only ``Rage'' expresses intense anger and hostility. Quick Tip: Remember:
- ``Rage'' = anger,
- ``Elation'' = joy,
- ``Apathy'' = indifference,
- ``Contentment'' = satisfaction.


Question 27:

Find the nearest one-word from the alternatives given below: A brief and concise statement expressing a general truth

  • (A) Paradox
  • (B) Metaphor
  • (C) Simile
  • (D) Aphorism
Correct Answer: (D) Aphorism
View Solution




Step 1: Understand the term ``Aphorism''

An ``aphorism'' is a pithy or concise statement that expresses a general truth or principle, often in a witty or wise manner.


Step 2: Evaluate other options

- ``Paradox'': A statement that contradicts itself but might be true.

- ``Metaphor'': A figure of speech comparing two things without using ``like'' or ``as''.

- ``Simile'': A comparison using ``like'' or ``as''.


Only ``Aphorism'' fits the definition of a brief truth-expressing statement. Quick Tip: An ``aphorism'' is a wise or witty one-liner like ``Actions speak louder than words'' — short but meaningful.


Question 28:

Find the nearest one-word from the alternatives given below: A feeling of great joy and excitement

  • (A) Euphoria
  • (B) Melancholy
  • (C) Indifference
  • (D) Anxiety
Correct Answer: (A) Euphoria
View Solution




Step 1: Define ``Euphoria''

Euphoria is a feeling or state of intense excitement and happiness. It fits exactly with ``great joy and excitement''.


Step 2: Eliminate incorrect options

- ``Melancholy'': Sadness or depression.

- ``Indifference'': Lack of interest or concern.

- ``Anxiety'': Nervousness or worry.


Only ``Euphoria'' captures the intended emotion. Quick Tip: Remember: ``Euphoria'' = extreme joy, ``Melancholy'' = deep sadness, ``Indifference'' = no emotion, ``Anxiety'' = worry.


Question 29:

Identify the meaning of the idiom or phrase in capitals from the choices.

She was ON CLOUD NINE after receiving her job offer.

  • (A) Feeling extremely happy and ecstatic
  • (B) Feeling disappointed and upset
  • (C) Feeling confused and uncertain
  • (D) Feeling tired and exhausted
Correct Answer: (A) Feeling extremely happy and ecstatic
View Solution




Step 1: Understand the idiom ``on cloud nine''

This idiom is commonly used to express a state of extreme happiness or elation.


Step 2: Context clue

Receiving a job offer is a positive event, so a happy emotion is expected.


Step 3: Eliminate incorrect meanings

- Option B: Disappointment is the opposite of the context.

- Option C and D: Confusion or tiredness are irrelevant.


Only Option A fits perfectly. Quick Tip: ``On cloud nine'' means feeling very happy or elated — often used after something wonderful happens.


Question 30:

Identify the meaning of the idiom or phrase in capitals from the choices.

The professor HIT THE NAIL ON THE HEAD with his insightful analysis.

  • (A) Completely miss the point
  • (B) Make a loud noise
  • (C) Speak loudly and confidently
  • (D) Make an accurate statement or assessment
Correct Answer: (D) Make an accurate statement or assessment
View Solution




Step 1: Idiom meaning

``Hit the nail on the head'' means to be exactly right about something — especially in analysis or judgment.


Step 2: Evaluate the context

The professor's analysis being ``insightful'' supports the idea that he was spot-on.


Step 3: Eliminate other options

- Option A: Opposite of what idiom means.

- Option B and C: Not related to correctness or insight.


Only Option D matches the idiom's true meaning. Quick Tip: If someone ``hits the nail on the head'', they are exactly right. Use this for correct evaluations or insights.


Question 31:

Find the nearest one-word from the alternatives given below: A person who is skilled in many areas or fields

  • (A) Specialist
  • (B) Novice
  • (C) Polymath
  • (D) Dilettante
Correct Answer: (C) Polymath
View Solution




Let us understand the meaning of the words one by one:


- ``Specialist'': A person who is an expert in a particular field. This means they focus on one area only. So, this is not correct.

- ``Novice'': A beginner or someone who is new to something. This is the opposite of someone skilled.

- ``Polymath'': A person who has knowledge and skills in many different subjects. This is exactly what the question is asking.

- ``Dilettante'': Someone who is interested in many things but does not study them deeply or seriously. Not someone skilled.


So, the correct answer is ``Polymath''. Quick Tip: A ``polymath'' is like a multi-talented person — skilled in science, arts, writing, and more.


Question 32:

Identify the meaning of the idiom or phrase in capitals from the choices.

The new employee was THROWN IN AT THE DEEP END on his first day.

  • (A) Given an easy and simple task
  • (B) Provided with all the necessary resources
  • (C) Given a challenging or difficult task with little guidance
  • (D) Allowed to take a break from work
Correct Answer: (C) Given a challenging or difficult task with little guidance
View Solution




Let’s break the idiom down. ``Thrown in at the deep end'' comes from swimming — if someone is dropped into deep water without help, they must struggle to swim.


In work or life, this idiom means giving someone a difficult task or challenge without much help or preparation.


Now let’s evaluate the options:

- Option A: Easy task – incorrect, the idiom talks about something difficult.

- Option B: Given everything needed – not true.

- Option C: Difficult task with little help – exactly what the idiom means.

- Option D: Taking a break – completely wrong.


Therefore, Option C is the right choice. Quick Tip: If you are ``thrown in at the deep end,'' you are expected to do something hard without much help.


Question 33:

Identify the meaning of the idiom or phrase in capitals from the choices.

She had to FACE THE MUSIC after making a mistake at work.

  • (A) Listen to music as a source of comfort
  • (B) Face the consequences for one's actions
  • (C) Perform in front of an audience
  • (D) Resolve a conflict through peaceful means
Correct Answer: (B) Face the consequences for one's actions
View Solution




The idiom ``face the music'' is often used when someone must accept punishment or criticism for what they did.


In the sentence, the person made a mistake at work — so now she has to accept whatever results from that mistake.


Let’s look at the options:

- Option A: Listening to music is not related to the meaning.

- Option B: Facing consequences fits perfectly.

- Option C: Performing is unrelated.

- Option D: Conflict resolution doesn’t match the sentence.


Hence, the best meaning is Option B. Quick Tip: ``Face the music'' means to take responsibility when you’re in trouble.


Question 34:

Identify the meaning of the idiom or phrase in capitals from the choices.

He SPILLED THE BEANS about the surprise party.

  • (A) Accidentally dropped some beans
  • (B) Shared a secret or confidential information
  • (C) Cooked a delicious meal
  • (D) Created a mess
Correct Answer: (B) Shared a secret or confidential information
View Solution




This idiom is often used when someone reveals something that was meant to be kept a secret.


In the context of a surprise party, if someone ``spilled the beans,'' it means they told others about it — ruining the surprise.


So the correct meaning is: ``Shared a secret.''


Options A, C, and D do not match the idiom's true meaning. Quick Tip: ``Spill the beans'' = Accidentally tell a secret that was supposed to be hidden.


Question 35:

Pick out the best filler which can complete the given sentence correctly.

She plays the piano as if she _____

  • (A) Is a professional pianist
  • (B) Played the piano professionally
  • (C) Plays the piano professionally
  • (D) Is professional pianist
Correct Answer: (A) Is a professional pianist
View Solution




We need to choose the correct grammatical structure.


Let’s test each one:

- (A) ``Is a professional pianist'' – grammatically correct, sounds smooth.

- (B) ``Played the piano professionally'' – tense doesn’t match ``plays'' in the first part.

- (C) ``Plays the piano professionally'' – repeats ``plays'', sounds odd.

- (D) ``Is professional pianist'' – missing article ``a'', so grammatically incorrect.


Thus, Option A fits both grammar and meaning. Quick Tip: After ``as if'', use present tense with correct article. Say: ``as if she is a professional pianist''.


Question 36:

Pick out the best filler which can complete the given sentence correctly.

The students studied so diligently that they _____

  • (A) May have passed the exam
  • (B) Passed the exam
  • (C) Passing the exam
  • (D) Had passed the exam
Correct Answer: (B) Passed the exam
View Solution




The structure ``so [adjective/adverb] that...'' is used to show cause and effect.

Here, ``studied so diligently that...'' implies a definite result.


Let's evaluate the options:

- (A) ``May have passed'' — suggests uncertainty, but the sentence implies a certain result.

- (B) ``Passed the exam'' — this fits perfectly. It’s in the past tense and shows a direct result.

- (C) ``Passing the exam'' — grammatically incorrect in this context.

- (D) ``Had passed'' — past perfect is not needed here because there’s no second event for comparison.


Hence, Option B is correct. Quick Tip: When a sentence uses ``so... that,'' use a past simple tense if it shows a clear and completed result.


Question 37:

Pick out the best filler which can complete the given sentence correctly.

He spoke so loudly that I _____

  • (A) Cannot hear him
  • (B) Was hearing him
  • (C) Could hear him
  • (D) Should hear him
Correct Answer: (C) Could hear him
View Solution




The phrase ``so loudly that I...'' suggests that the speaker was audible due to the loudness.


Let's analyze:

- (A) ``Cannot hear him'' — present tense, not suitable here.

- (B) ``Was hearing him'' — awkward construction, not typically used.

- (C) ``Could hear him'' — past tense, correct and natural in context.

- (D) ``Should hear him'' — implies obligation, which doesn't fit.


Therefore, Option C is the best match. Quick Tip: Use ``could'' in past contexts to show ability or possibility due to another action (like speaking loudly).


Question 38:

Find the correct preposition:

She is allergic _____ seafood.

  • (A) On
  • (B) With
  • (C) To
  • (D) At
Correct Answer: (C) To
View Solution




The word ``allergic'' always pairs with the preposition ``to''.

So the correct phrase is ``allergic to something''.


Let's look at the options:

- (A) ``On'' — incorrect

- (B) ``With'' — incorrect

- (C) ``To'' — correct and standard usage

- (D) ``At'' — incorrect


Thus, Option C is correct. Quick Tip: Always say ``allergic to [noun]'' — e.g., allergic to dust, allergic to peanuts, allergic to seafood.


Question 39:

Find the correct preposition: He apologized ____ his mistake.

  • (A) For
  • (B) On
  • (C) To
  • (D) By
Correct Answer: (A) For
View Solution




To determine the correct preposition to fill in the blank in the sentence "He apologized \underline{\hspace{1cm his mistake," we need to understand the grammatical rules governing the verb "apologize" and its preposition usage in English.

This question tests your knowledge of prepositions, which are words that show the relationship between a verb or noun and another word in the sentence. Prepositions often depend on the verb they accompany, and "apologize" has specific preposition patterns.


Step 1: Understand the Verb "Apologize"

The verb "apologize" means to express regret or say sorry for something you have done wrong.

In English, the verb "apologize" is typically followed by a preposition to indicate what the apology is for or to whom it is directed.

We need to identify the correct preposition that fits the sentence structure: "He apologized [preposition] his mistake." Here, "his mistake" is the reason for the apology.


Step 2: Analyze Option A - "For"

Option A suggests the preposition "for," making the sentence: "He apologized for his mistake."

In English, the phrase "apologize for" is a standard and correct construction when referring to the reason or cause of the apology.

For example:

- "She apologized for being late." (The reason for the apology is "being late.")

- "He apologized for breaking the vase." (The reason is "breaking the vase.")

In this case, "his mistake" is the reason for the apology, so "apologize for his mistake" is grammatically correct and conveys the intended meaning: he is sorry about the mistake he made.

Option A seems promising, so let’s keep it in mind and check the other options to confirm.


Step 3: Analyze Option B - "On"

Option B suggests the preposition "on," making the sentence: "He apologized on his mistake."

The preposition "on" typically indicates a surface, time, or topic (e.g., "on the table," "on Monday," "a lecture on biology").

However, "apologize on" is not a standard or correct construction in English for this context.

You might say "comment on a mistake" or "focus on a mistake," but not "apologize on a mistake." The preposition "on" does not fit the verb "apologize" when referring to the reason for the apology.

For example, saying "He apologized on breaking the vase" sounds unnatural and incorrect to native speakers.

Therefore, option B is incorrect.


Step 4: Analyze Option C - "To"

Option C suggests the preposition "to," making the sentence: "He apologized to his mistake."

The preposition "to" is often used with "apologize," but it indicates the recipient of the apology, not the reason for it.

For example:

- "He apologized to his friend." (The apology is directed to "his friend.")

- "She apologized to her teacher for being rude." (Here, "to her teacher" is the recipient, and "for being rude" is the reason.)

In the sentence "He apologized to his mistake," "his mistake" is not a person who can receive an apology; it is the reason for the apology.

Using "to" in this context is grammatically incorrect because "apologize to" requires a person or entity as the object, not an action or thing like "his mistake."

Therefore, option C is incorrect.


Step 5: Analyze Option D - "By"

Option D suggests the preposition "by," making the sentence: "He apologized by his mistake."

The preposition "by" often indicates the means, agent, or time (e.g., "by car," "by tomorrow," "painted by the artist").

However, "apologize by" is not a standard construction in this context.

You might use "by" in other contexts, such as "He made up for his mistake by working harder," but not directly with "apologize" to indicate the reason for the apology.

Saying "He apologized by his mistake" implies that the mistake is the means of apologizing, which is nonsensical.

For example, "He apologized by breaking the vase" does not make sense in English.

Therefore, option D is incorrect.


Step 6: Confirm the Correct Preposition with Grammar Rules

Let’s confirm the correct usage of "apologize":

- When "apologize" is followed by the reason for the apology (e.g., a mistake, an action), the preposition "for" is used: "apologize for something."

- When "apologize" is directed to a person or entity, the preposition "to" is used: "apologize to someone."

- Optionally, both can be combined: "apologize to someone for something" (e.g., "He apologized to his boss for his mistake.").

In this question, the sentence is "He apologized [preposition] his mistake," and we are focusing on the reason for the apology ("his mistake"), not the recipient.

Thus, the correct preposition is "for," and the sentence becomes: "He apologized for his mistake."

This matches option A, confirming that it is indeed the correct answer.


Step 7: Why the Other Options Are Incorrect - A Broader Perspective

- Option B ("On"): "Apologize on" is not a valid phrase in English for this context. "On" is used with other verbs like "comment" or "focus," but not with "apologize" when indicating the reason.

- Option C ("To"): "Apologize to" is correct when specifying the person receiving the apology, but "his mistake" is not a person, so "to" is inappropriate here.

- Option D ("By"): "Apologize by" does not fit grammatically, as "by" suggests a means or agent, not the reason for an apology.


Step 8: Practical Usage and Examples

Understanding prepositions with "apologize" is important for clear communication:

- "I apologized for my behavior." (Correct: "for" indicates the reason.)

- "I apologized to my friend." (Correct: "to" indicates the recipient.)

- "I apologized to my friend for my behavior." (Correct: Combines both the recipient and reason.)

In this question, since only the reason ("his mistake") is provided without a recipient, "for" is the appropriate preposition.

For beginners, practicing with similar sentences can help solidify this rule: "She apologized for her delay," "They apologized for the confusion."


Step 9: Key Concepts

- Prepositions with Verbs: Some verbs in English require specific prepositions. For "apologize," use "for" when stating the reason and "to" when stating the recipient.

- Common Patterns:

- "Apologize for something" (reason): "He apologized for his mistake."

- "Apologize to someone" (recipient): "He apologized to his friend."

- "Apologize to someone for something": "He apologized to his friend for his mistake."

- Avoid Common Mistakes: Don’t confuse prepositions like "on," "by," or "to" in contexts where they don’t fit. Test the sentence by speaking it aloud to see if it sounds natural.

- Practice with Examples: Create your own sentences using "apologize" to get comfortable with its prepositions.
Quick Tip: To master prepositions with verbs:
- Learn verb-preposition pairs: "Apologize for" (reason), "apologize to" (recipient).
- Practice with sentences: "I apologized for my error," "She apologized to her teacher."
- Test correctness: Say the sentence aloud—if it sounds unnatural (e.g., "He apologized on his mistake"), the preposition is likely wrong.
- Expand vocabulary: Other verbs with specific prepositions include "depend on," "agree with," "believe in."


Question 40:

Find the correct preposition: The book is written \hspace{1cm} John Green.

  • (A) By
  • (B) In
  • (C) Of
  • (D) At
Correct Answer: (A) By
View Solution




To determine the correct preposition to fill in the blank in the sentence "The book is written \underline{\hspace{1cm John Green," we need to understand the grammatical rules governing the passive voice construction and preposition usage in English.

This question tests your knowledge of prepositions used with passive voice, particularly in theomenti

Step 1: Understand the Sentence Structure

The sentence "The book is written \underline{\hspace{1cm John Green" is in the passive voice, where the subject (the book) is receiving the action (being written).

In passive constructions, the doer of the action (the agent) is often introduced by a preposition.

Here, "John Green" is the agent—the person who wrote the book—so we need a preposition that correctly indicates the author of the book.


Step 2: Analyze Option A - "By"

Option A suggests the preposition "by," making the sentence: "The book is written by John Green."

In English, the preposition "by" is commonly used in passive voice constructions to indicate the agent performing the action.

For example:

- "The song was sung by Adele." (Adele is the agent who performed the action of singing.)

- "The house was built by my grandfather." (My grandfather is the agent who built the house.)

In this case, "John Green" is the author who wrote the book, so "written by John Green" is grammatically correct and standard in English.

Option A seems correct, but let’s evaluate the other options to confirm.


Step 3: Analyze Option B - "In"

Option B suggests the preposition "in," making the sentence: "The book is written in John Green."

The preposition "in" typically indicates location, time, or manner (e.g., "written in English," "written in 2020").

However, "John Green" is a person, not a language, time, or place. Saying "written in John Green" does not make sense grammatically or semantically, as it implies the book was written inside John Green, which is absurd.

Therefore, option B is incorrect.


Step 4: Analyze Option C - "Of"

Option C suggests the preposition "of," making the sentence: "The book is written of John Green."

The preposition "of" often indicates possession or association (e.g., "a book of poems," "the works of Shakespeare").

However, in this context, "written of John Green" is not a standard construction. It might suggest the book is made of John Green (which is nonsensical) or that the book is about John Green, but the latter would be phrased differently (e.g., "a biography of John Green").

In passive voice, "of" is not used to introduce the agent of the action.

Therefore, option C is incorrect.


Step 5: Analyze Option D - "At"

Option D suggests the preposition "at," making the sentence: "The book is written at John Green."

The preposition "at" typically indicates a specific location or point in time (e.g., "at the library," "at 5 p.m.").

"John Green" is a person, not a place or time, so "written at John Green" is grammatically incorrect and does not convey a meaningful idea.

Therefore, option D is incorrect.


Step 6: Confirm with Standard Usage

In English, when indicating the author of a written work in a passive construction, "by" is the standard preposition:

- "The Fault in Our Stars was written by John Green."

- "This article was written by a journalist."

This usage is consistent across formal and informal writing, confirming that option A, "by," is the correct choice.


Step 7: Why the Other Options Are Incorrect - A Broader Perspective

- Option B ("In"): "In" is used for languages, styles, or time periods (e.g., "written in French"), not for people as agents.

- Option C ("Of"): "Of" might be used in phrases like "a novel of John Green" to mean a novel belonging to him, but not in a passive construction to indicate authorship.

- Option D ("At"): "At" is for locations or times, not for indicating the agent of an action.


Step 8: Key Concepts

- Passive Voice: In passive voice, the subject receives the action, and the agent (doer) is often introduced by "by" (e.g., "The book was written by the author").

- Prepositions with Agents: Use "by" to indicate the agent in passive constructions. Other prepositions like "with" might be used in different contexts (e.g., "written with a pen").

- Authorship: When stating who wrote a book, "by" is the standard preposition in both active and passive voice.
Quick Tip: To understand prepositions in passive voice:
- Use "by" to indicate the agent in passive constructions: "The song was performed by the band."
- Avoid using prepositions like "in," "of," or "at" for agents—they have different roles (e.g., "written in Spanish," "a book of stories").
- Practice with examples: "The poem was written by Emily Dickinson," "The house was designed by an architect."


Question 41:

Find the correct preposition: She is interested _____ learning foreign languages.

  • (A) On
  • (B) With
  • (C) In
  • (D) At
Correct Answer: (C) In
View Solution




To determine the correct preposition for the sentence "She is interested \underline{\hspace{1cm learning foreign languages," we need to identify the preposition that correctly pairs with the adjective "interested" in English grammar.

This question tests your knowledge of adjective-preposition collocations, which are specific combinations of adjectives and prepositions that are standard in English.


Step 1: Understand the Adjective "Interested"

The adjective "interested" describes a state of wanting to know or learn about something.

In English, "interested" is typically followed by a preposition to indicate what someone is interested in.

The phrase "learning foreign languages" is a gerund phrase (a verb form acting as a noun), so we need a preposition that works with "interested" and a gerund.


Step 2: Analyze Option A - "On"

Option A suggests the preposition "on," making the sentence: "She is interested on learning foreign languages."

The preposition "on" typically indicates a surface, time, or topic (e.g., "on the table," "on Monday," "a book on history").

However, "interested on" is not a standard collocation in English.

Saying "interested on learning" sounds unnatural to native speakers and is grammatically incorrect.

For example, "She is interested on studying" would not be used; instead, we say "interested in studying."

Therefore, option A is incorrect.


Step 3: Analyze Option B - "With"

Option B suggests the preposition "with," making the sentence: "She is interested with learning foreign languages."

The preposition "with" often indicates association or means (e.g., "happy with the results," "help with homework").

However, "interested with" is not a standard collocation.

Saying "interested with learning" is incorrect and does not convey the intended meaning.

For example, "She is interested with studying" sounds awkward and is not used in English.

Therefore, option B is incorrect.


Step 4: Analyze Option C - "In"

Option C suggests the preposition "in," making the sentence: "She is interested in learning foreign languages."

The collocation "interested in" is a standard and correct construction in English, especially when followed by a noun or gerund.

For example:

- "He is interested in history." (Followed by a noun.)

- "She is interested in traveling." (Followed by a gerund.)

In this case, "learning foreign languages" is a gerund phrase, and "interested in learning foreign languages" is grammatically correct and natural.

Option C seems to be the correct choice, but let’s check the final option to confirm.


Step 5: Analyze Option D - "At"

Option D suggests the preposition "at," making the sentence: "She is interested at learning foreign languages."

The preposition "at" typically indicates a specific point or location (e.g., "good at math," "look at the sky").

However, "interested at" is not a standard collocation in English.

Saying "interested at learning" is incorrect and sounds unnatural.

For example, "She is interested at studying" would not be used by native speakers.

Therefore, option D is incorrect.


Step 6: Confirm with Grammar Rules

The adjective "interested" is almost always followed by the preposition "in" when indicating the object of interest, especially with gerunds or nouns:

- "They are interested in art."

- "She is interested in learning new skills."

This rule applies consistently in English grammar, confirming that option C, "in," is the correct answer.


Step 7: Why the Other Options Are Incorrect - A Broader Perspective

- Option A ("On"): "On" is used with other adjectives like "keen on" (e.g., "keen on learning"), but not with "interested."

- Option B ("With"): "With" pairs with adjectives like "satisfied with" or "pleased with," but not "interested."

- Option D ("Att"): "At" pairs with adjectives like "good at" or "bad at," but not "interested."


Step 8: Key Concepts

- Adjective-Preposition Collocations: Certain adjectives require specific prepositions (e.g., "interested in," "good at," "afraid of").

- Gerunds: When a verb acts as a noun (e.g., "learning"), it can follow a preposition like "in" after "interested."

- Practice: Test collocations by creating sentences: "I’m interested in music," "She’s interested in cooking."
Quick Tip: To master adjective-preposition pairs:
- Learn common collocations: "interested in," "good at," "afraid of."
- Practice with gerunds: "interested in learning," "good at drawing."
- Test sentences: If "interested on learning" sounds wrong, try "interested in learning."


Question 42:

Find the correct preposition: He is responsible ____ the project’s success.

  • (A) For
  • (B) In
  • (C) With
  • (D) At
Correct Answer: (A) For
View Solution




To determine the correct preposition for the sentence "He is responsible ____ the project’s success," we need to identify the preposition that correctly pairs with the adjective "responsible" in this context.

This question tests your knowledge of adjective-preposition collocations and the meaning of "responsible" in English.


Step 1: Understand the Adjective "Responsible"

The adjective "responsible" means being the cause of something or having the duty to take care of it.

When "responsible" is used to indicate what someone is accountable for, it is typically followed by a preposition.

Here, "the project’s success" is the outcome or duty he is accountable for, so we need the correct preposition to pair with "responsible."


Step 2: Analyze Option A - "For"

Option A suggests the preposition "for," making the sentence: "He is responsible for the project’s success."

The collocation "responsible for" is standard in English when indicating what someone is accountable for or the cause of.

For example:

- "She is responsible for the team’s performance." (She has the duty to ensure the team’s performance.)

- "He is responsible for the accident." (He caused the accident.)

In this case, "the project’s success" is the outcome he is accountable for, so "responsible for the project’s success" is correct and natural.

Option A seems correct, but let’s check the other options.


Step 3: Analyze Option B - "In"

Option B suggests the preposition "in," making the sentence: "He is responsible in the project’s success."

The preposition "in" often indicates involvement or location (e.g., "involved in the project").

However, "responsible in" is not a standard collocation.

Saying "responsible in the project’s success" implies he is responsible while inside the success, which is not meaningful.

For example, "He is responsible in the outcome" sounds incorrect.

Therefore, option B is incorrect.


Step 4: Analyze Option C - "With"

Option C suggests the preposition "with," making the sentence: "He is responsible with the project’s success."

The preposition "with" often indicates association or means (e.g., "careful with money").

However, "responsible with" is not correct in this context.

Saying "responsible with the project’s success" does not convey the intended meaning and is grammatically incorrect.

For example, "He is responsible with the outcome" sounds unnatural.

Therefore, option C is incorrect.


Step 5: Analyze Option D - "At"

Option D suggests the preposition "at," making the sentence: "He is responsible at the project’s success."

The preposition "at" indicates a point or location (e.g., "good at sports").

However, "responsible at" is not a standard collocation.

Saying "responsible at the project’s success" is incorrect and does not make sense.

For example, "He is responsible at the result" is not a valid construction.

Therefore, option D is incorrect.


Step 6: Confirm with Grammar Rules

The adjective "responsible" is typically followed by "for" when indicating what someone is accountable for:

- "They are responsible for the event’s success."

- "She is responsible for paying the bills."

This usage is consistent in English, confirming that option A, "for," is the correct answer.


Step 7: Why the Other Options Are Incorrect - A Broader Perspective

- Option B ("In"): "In" might be used in phrases like "involved in the success," but not with "responsible."

- Option C ("With"): "With" pairs with adjectives like "good with" (e.g., "good with kids"), but not "responsible."

- Option D ("At"): "At" is used with adjectives like "good at," but not "responsible."


Step 8: Key Concepts

- Adjective-Preposition Collocations: "Responsible for" is used to indicate accountability or causation.

- Meaning of "Responsible": It can mean both causing something and having a duty for something, and "for" works in both cases.

- Practice: "I’m responsible for the report," "He’s responsible for the delay."
Quick Tip: To master "responsible":
- Use "responsible for" to show accountability: "responsible for the outcome."
- Avoid incorrect prepositions like "in," "with," or "at" with "responsible" in this context.
- Practice: "She’s responsible for the team," "He’s responsible for the error."


Question 43:

We _____ to the new restaurant for dinner last night.

  • (A) Were going
  • (B) Went
  • (C) Have gone
  • (D) Will go
Correct Answer: (B) Went
View Solution




To determine the correct verb phrase for the sentence "We ____ to the new restaurant for dinner last night," we need to analyze the tense and meaning of the sentence.

This question tests your knowledge of verb tenses, particularly how to match the tense with a time-specific adverb like "last night."


Step 1: Understand the Time Context

The phrase "last night" indicates a specific time in the past.

In English, when a sentence includes a past time marker like "last night," "yesterday," or "in 2020," the verb typically needs to be in a past tense to match the time frame.

We need to choose a verb phrase that correctly reflects an action completed in the past.


Step 2: Analyze Option A - "Were going"

Option A suggests the phrase "were going," making the sentence: "We were going to the new restaurant for dinner last night."

The phrase "were going" is in the past continuous tense, which describes an ongoing action in the past.

It often implies that the action was in progress but may not have been completed, or it sets the scene for another event (e.g., "We were going to the restaurant when it started raining.").

In this sentence, however, "last night" suggests a completed action—we went to the restaurant and had dinner.

Using "were going" leaves the action feeling incomplete or interrupted, which doesn’t fully match the context of having dinner last night.

While "were going" is grammatically possible, it’s less natural than a simple past tense for a completed action.

Let’s check the other options to confirm.


Step 3: Analyze Option B - "Went"

Option B suggests the verb "went," making the sentence: "We went to the new restaurant for dinner last night."

The verb "went" is in the simple past tense, which is used for actions that were completed at a specific time in the past.

For example:

- "I went to the store yesterday." (A completed action in the past.)

- "They went to the movies last weekend." (A completed action.)

Here, "last night" indicates the action happened and finished in the past, and "went" perfectly matches this time frame.

The sentence "We went to the new restaurant for dinner last night" is grammatically correct and natural, indicating that the action of going to the restaurant was completed.

Option B seems to be the correct choice, but let’s evaluate the remaining options.


Step 4: Analyze Option C - "Have gone"

Option C suggests the phrase "have gone," making the sentence: "We have gone to the new restaurant for dinner last night."

The phrase "have gone" is in the present perfect tense, which describes actions that happened at an unspecified time in the past or actions that have relevance to the present moment.

For example:

- "We have gone to that restaurant before." (At some unspecified time in the past.)

- "We have gone to the restaurant, so we can tell you about it." (The action has present relevance.)

However, "last night" is a specific past time marker, and the present perfect tense is not typically used with specific past times like "yesterday," "last night," or "in 2020."

Saying "We have gone to the restaurant last night" is grammatically incorrect because the present perfect tense conflicts with the specific past time frame.

Therefore, option C is incorrect.


Step 5: Analyze Option D - "Will go"

Option D suggests the phrase "will go," making the sentence: "We will go to the new restaurant for dinner last night."

The phrase "will go" is in the simple future tense, which describes actions that will happen after the present moment.

For example: "We will go to the restaurant tomorrow." (A future action.)

However, "last night" refers to a time in the past, so using a future tense verb like "will go" creates a tense mismatch.

Saying "We will go to the restaurant last night" is grammatically incorrect and logically impossible, as a future action cannot occur in the past.

Therefore, option D is incorrect.


Step 6: Re-Evaluate Option A vs. Option B

Let’s revisit option A ("were going") and option B ("went") to ensure we’re choosing the most natural option:

- "We were going to the new restaurant for dinner last night." (Option A)

- "We went to the new restaurant for dinner last night." (Option B)

While "were going" (past continuous) can be used to describe an ongoing action, it often implies the action was interrupted or sets the scene for another event (e.g., "We were going to the restaurant when we got a flat tire.").

In this sentence, there’s no indication of an interruption or additional event—just a statement that the action happened.

The simple past "went" (option B) is more straightforward and natural for a completed action like having dinner last night.

Native speakers would typically say "We went to the restaurant last night" rather than "We were going to the restaurant last night" in this context.

Therefore, option B is the better choice.


Step 7: Why the Other Options Are Incorrect - A Broader Perspective

- Option A ("Were going"): While grammatically possible, "were going" implies an ongoing or interrupted action, which doesn’t fully match the completed nature of the sentence.

- Option C ("Have gone"): Present perfect tense cannot be used with specific past time markers like "last night."

- Option D ("Will go"): Future tense cannot be used for a past event.


Step 8: Key Concepts

- Simple Past Tense: Use the simple past (e.g., "went") for completed actions at a specific time in the past.

- Past Continuous: Use past continuous (e.g., "were going") for ongoing past actions, often with an interruption.

- Present Perfect: Use present perfect (e.g., "have gone") for unspecified past actions or those with present relevance, not with specific past times.

- Tense and Time Markers: Match the verb tense to the time marker (e.g., "last night" requires a past tense).
Quick Tip: To master verb tenses with time markers:
- Use simple past for completed actions in the past: "We went yesterday."
- Use past continuous for ongoing past actions: "We were going when it rained."
- Avoid present perfect with specific past times: Not "We have gone last night."
- Match the tense to the time: "Last night" requires a past tense like "went."


Question 44:

The bird ____ away when it saw the approaching predator.

  • (A) Flew
  • (B) Was flying
  • (C) Has flown
  • (D) Will fly
Correct Answer: (A) Flew
View Solution




To determine the correct verb phrase for the sentence "The bird \underline{\hspace{1cm away when it saw the approaching predator," we need to analyze the verb tense and the relationship between the two clauses in the sentence.

This question tests your understanding of verb tenses in dependent clauses and how actions in the past relate to each other.


Step 1: Understand the Sentence Structure

The sentence has two clauses:

- Main clause: "The bird \underline{\hspace{1cm away."

- Dependent clause: "when it saw the approaching predator."

The dependent clause "when it saw the approaching predator" uses the verb "saw," which is in the simple past tense, indicating that the predator’s approach happened at a specific time in the past.

The main clause describes the bird’s action (flying away) in response to seeing the predator, so we need a verb tense that matches the past time frame and the sequence of events.


Step 2: Analyze Option A - "Flew"

Option A suggests the verb "flew," making the sentence: "The bird flew away when it saw the approaching predator."

The verb "flew" is in the simple past tense, which describes a completed action in the past.

The sequence of events is:

- The bird saw the predator (past action: "saw").

- The bird flew away (past action: "flew").

In English, when two actions happen one after the other in the past, both verbs are often in the simple past tense, especially with a conjunction like "when" that indicates a cause-and-effect relationship.

For example:

- "She ran when she heard the noise." (Both verbs in simple past.)

- "He left when the meeting ended." (Both verbs in simple past.)

Here, "The bird flew away when it saw the predator" indicates that seeing the predator caused the bird to fly away, and both actions are completed in the past.

Option A seems correct, but let’s check the other options.


Step 3: Analyze Option B - "Was flying"

Option B suggests the phrase "was flying," making the sentence: "The bird was flying away when it saw the approaching predator."

The phrase "was flying" is in the past continuous tense, which describes an ongoing action in the past.

It often sets the scene for another event that interrupts or happens at the same time.

For example: "I was reading when the phone rang." (The ongoing action "was reading" is interrupted by "rang.")

In this sentence, "The bird was flying away when it saw the approaching predator" reverses the cause-and-effect relationship: it implies the bird was already flying away when it saw the predator, which changes the meaning.

The intended meaning is that seeing the predator caused the bird to fly away, not that the bird was already flying.

While "was flying" is grammatically possible, it doesn’t match the intended cause-and-effect relationship as naturally as the simple past "flew."

Let’s keep option B in mind but check the others.


Step 4: Analyze Option C - "Has flown"

Option C suggests the phrase "has flown," making the sentence: "The bird has flown away when it saw the approaching predator."

The phrase "has flown" is in the present perfect tense, which describes actions that happened at an unspecified time in the past or have relevance to the present.

For example: "The bird has flown away, so it’s safe now." (Present relevance.)

However, the dependent clause "when it saw the approaching predator" uses the simple past tense ("saw"), indicating a specific past event.

The present perfect tense ("has flown") is not typically used with specific past time markers or in clauses with past tense verbs like "saw."

Saying "The bird has flown away when it saw the predator" is grammatically incorrect because the tenses don’t match.

Therefore, option C is incorrect.


Step 5: Analyze Option D - "Will fly"

Option D suggests the phrase "will fly," making the sentence: "The bird will fly away when it saw the approaching predator."

The phrase "will fly" is in the simple future tense, describing an action that will happen after the present moment.

However, the dependent clause "when it saw the approaching predator" is in the simple past tense ("saw"), indicating a past event.

Using a future tense ("will fly") with a past tense ("saw") creates a tense mismatch.

Saying "The bird will fly away when it saw the predator" is grammatically incorrect and logically impossible, as a future action cannot be caused by a past event in this context.

Therefore, option D is incorrect.


Step 6: Re-Evaluate Option A vs. Option B

Let’s compare options A (``flew") and B ("was flying") to ensure we’re choosing the most appropriate one:

- "The bird flew away when it saw the approaching predator." (Option A)

- "The bird was flying away when it saw the approaching predator." (Option B)

Option B (``was flying") suggests the bird was already in the process of flying away when it saw the predator, implying that the flying started before seeing the predator.

Option A (``flew") suggests that seeing the predator caused the bird to fly away, which aligns with the natural interpretation of the sentence: the bird saw the predator and then took action by flying away.

In cause-and-effect sentences with "when," the simple past tense is typically used for both clauses when the actions are sequential: "I jumped when I heard the sound."

Therefore, option A (``flew") is the better choice for the intended meaning.


Step 7: Why the Other Options Are Incorrect - A Broader Perspective

- Option B (``Was flying"): Past continuous implies an ongoing action, which changes the meaning to suggest the bird was already flying, not that it flew as a result of seeing the predator.

- Option C (``Has flown"): Present perfect doesn’t match the past tense of "saw."

- Option D (``Will fly"): Future tense cannot be used with a past event.


Step 8: Key Concepts

- Simple Past in Sequential Actions: Use simple past for sequential past actions with "when": "She left when she saw him."

- Past Continuous: Use past continuous to show an ongoing action interrupted by another: "She was leaving when he arrived."

- Tense Consistency: Ensure tenses align in dependent clauses (e.g., don’t mix present perfect with simple past).
Quick Tip: To handle verb tenses in dependent clauses:
- Use simple past for sequential past actions: "She ran when she saw the dog."
- Use past continuous for ongoing actions: "She was running when she saw the dog."
- Keep tenses consistent: Don’t mix future or present perfect with past in this context.
- Focus on cause-and-effect: "When" often pairs with simple past for cause-and-effect in the past.


Question 45:

Find out the word which is nearest to the opposite in meaning to the key word given in capitals: CACOPHONY

  • (A) Discord
  • (B) Enormous
  • (C) Melancholy
  • (D) Harmony
Correct Answer: (D) Harmony
View Solution




To determine the word that is nearest to the opposite in meaning to the key word "CACOPHONY," we need to understand the meaning of "cacophony" and compare it with the given options.

This question tests your vocabulary skills, specifically your ability to identify antonyms (words with opposite meanings).


Step 1: Understand the Meaning of "CACOPHONY"

The word "CACOPHONY" refers to a harsh, unpleasant mixture of sounds, often described as noise or dissonance.

It is typically used to describe sounds that are jarring, chaotic, or discordant, such as a mix of loud, clashing noises (e.g., "The cacophony of car horns and shouting filled the street.").

To find the opposite, we need a word that means pleasant, harmonious, or agreeable sound.


Step 2: Analyze Option A - "Discord"

Option A is "Discord," which means disagreement or conflict, but in the context of sound, it refers to a lack of harmony, producing an unpleasant or jarring effect.

For example, in music, "discord" describes notes that clash and create an unpleasing sound (e.g., "The discord in the song made it hard to listen to.").

Since "discord" is similar in meaning to "cacophony" (both describe unpleasant, disharmonious sounds), it is not the opposite.

Therefore, option A is incorrect.


Step 3: Analyze Option B - "Enormous"

Option B is "Enormous," which means very large in size, quantity, or extent (e.g., "An enormous elephant stood in the field.").

"Enormous" describes size or magnitude and has no direct relation to sound or the concept of harmony or dissonance.

Since "enormous" does not convey a meaning related to sound, it cannot be the opposite of "cacophony," which is specifically about sound.

Therefore, option B is incorrect.


Step 4: Analyze Option C - "Melancholy"

Option C is "Melancholy," which means a feeling of sadness, gloom, or depression (e.g., "She felt a deep melancholy after the loss.").

While "melancholy" might be associated with certain types of music (e.g., a "melancholy tune"), it primarily describes an emotional state, not the quality of sound itself.

"Cacophony" is about the harshness of sound, not its emotional tone, so "melancholy" does not directly oppose the meaning of "cacophony."

A melancholy sound could still be cacophonous if it’s harsh and discordant, or it could be harmonious if it’s soothing and sad.

Therefore, option C is not the best opposite.


Step 5: Analyze Option D - "Harmony"

Option D is "Harmony," which refers to a pleasing arrangement of parts, often used in the context of sound to describe notes or voices that blend well together (e.g., "The choir sang in perfect harmony.").

In music and sound, "harmony" is the opposite of dissonance or discord—it describes sounds that are pleasant, smooth, and agreeable.

Since "cacophony" means harsh, discordant sound, "harmony" directly opposes it by meaning pleasant, concordant sound.

For example, while a "cacophony of screams" is harsh and chaotic, a "harmony of voices" is soothing and unified.

Option D appears to be the correct choice, as it is the nearest in meaning to the opposite of "cacophony."


Step 6: Re-Evaluate and Confirm

Let’s double-check the options:

- "Discord" (A) is a synonym of "cacophony," not an antonym, as both describe unpleasant sound.

- "Enormous" (B) is unrelated to sound and doesn’t fit the context.

- "Melancholy" (C) is about emotion, not the quality of sound, so it’s not a direct opposite.

- "Harmony" (D) directly contrasts with "cacophony" by describing pleasant, agreeable sound.

Additionally, the question asks for the word "nearest to the opposite," and "harmony" is the most precise antonym for "cacophony" in the context of sound.

Therefore, option D, "Harmony," is the correct answer, despite the provided answer being option A, which seems to be a mistake.


Step 7: Why the Other Options Are Incorrect - A Broader Perspective

- Option A ("Discord"): "Discord" is essentially a synonym for "cacophony," as both describe a lack of harmony in sound.

- Option B ("Enormous"): This word pertains to size, not sound, and is irrelevant to the meaning of "cacophony."

- Option C ("Melancholy"): While it might describe the mood of a sound, it doesn’t address the harmony or dissonance of the sound itself.


Step 8: Key Concepts

- Antonyms: Antonyms are words with opposite meanings. For "cacophony" (harsh sound), the antonym is "harmony" (pleasant sound).

- Context Matters: Ensure the antonym fits the context of the word—in this case, "cacophony" is about sound, so the antonym must also relate to sound.

- Vocabulary Building: Learn related words: "cacophony," "discord," and "dissonance" are similar; "harmony," "melody," and "concord" are opposites.
Quick Tip: To find antonyms:
- Understand the key word’s meaning: "CACOPHONY" means harsh, unpleasant sound.
- Look for a word that means the opposite in the same context: "harmony" means pleasant sound.
- Eliminate unrelated words: "enormous" (size) and "melancholy" (emotion) don’t fit the context of sound.
- Build vocabulary by learning synonyms and antonyms: "cacophony" vs. "harmony," "big" vs. "small."


Question 46:

Find out the word which is nearest to the opposite in meaning to the key word given in capitals: QUERULOUS

  • (A) Tolerant
  • (B) Pessimistic
  • (C) Painful
  • (D) Ambitious
Correct Answer: (A) Tolerant
View Solution




To determine the word that is nearest to the opposite in meaning to the key word "QUERULOUS," we need to understand the meaning of "querulous" and compare it with the given options.

This question tests your vocabulary and ability to identify antonyms.


Step 1: Understand the Meaning of "QUERULOUS"

The word "QUERULOUS" means complaining or whining in a petulant or irritable manner (e.g., "The querulous child kept whining about the long trip.").

It describes someone who is habitually grumbling, fretful, or discontented, often in a way that annoys others.

To find the opposite, we need a word that means accepting, patient, or not complaining.


Step 2: Analyze Option A - "Tolerant"

Option A is "Tolerant," which means showing patience, acceptance, or willingness to endure something without complaint (e.g., "She was tolerant of the delays and didn’t complain.").

A "tolerant" person is the opposite of "querulous" because they are patient and understanding, rather than irritable and complaining.

For example, while a "querulous" person might constantly grumble about a situation, a "tolerant" person would accept it calmly.

Option A seems to be the correct choice, but let’s check the others to confirm.


Step 3: Analyze Option B - "Pessimistic"

Option B is "Pessimistic," which means having a negative outlook or expecting the worst outcome (e.g., "He was pessimistic about the project’s success.").

While a "querulous" person might complain due to dissatisfaction, "pessimistic" focuses on a mindset of negativity, not necessarily the act of complaining.

A pessimistic person might not complain—they might just feel hopeless. Conversely, a querulous person might complain without being pessimistic (e.g., they might just be irritable).

"Pessimistic" is not a direct opposite of "querulous," as it doesn’t address the behavior of complaining.

Therefore, option B is incorrect.


Step 4: Analyze Option C - "Painful"

Option C is "Painful," which means causing physical or emotional pain (e.g., "The injury was painful.").

"Painful" describes a sensation or experience, not a personality trait or behavior.

"Querulous" is about a person’s tendency to complain, while "painful" is unrelated to complaining or tolerance.

A situation might be painful, but that doesn’t make it the opposite of querulousness.

Therefore, option C is incorrect.


Step 5: Analyze Option D - "Ambitious"

Option D is "Ambitious," which means having a strong desire to achieve goals or succeed (e.g., "She is ambitious and works hard to achieve her dreams.").

"Ambitious" describes someone driven and goal-oriented, but it doesn’t directly relate to complaining or patience.

An ambitious person might complain (be querulous) or might not—it’s not inherent to ambition.

"Ambitious" is not the opposite of "querulous," as it addresses a different trait.

Therefore, option D is incorrect.


Step 6: Confirm the Correct Answer

Let’s review:

- "Tolerant" (A) means patient and accepting, directly opposing the complaining nature of "querulous."

- "Pessimistic" (B) is about negativity, not the act of complaining, so it’s not the opposite.

- "Painful" (C) is about pain, not behavior or personality, so it’s unrelated.

- "Ambitious" (D) is about drive and goals, not tolerance or complaining.

Option A, "Tolerant," is indeed the word nearest to the opposite of "querulous," as it describes someone who does not complain and is patient, contrasting with the irritable, whining nature of "querulous."


Step 7: Why the Other Options Are Incorrect - A Broader Perspective

- Option B ("Pessimistic"): Pessimism is a mindset, not a behavior, and doesn’t directly oppose complaining.

- Option C ("Painful"): This word describes a feeling, not a trait, and is irrelevant to "querulous."

- Option D ("Ambitious"): Ambition relates to goals, not to the act of complaining or being patient.


Step 8: Key Concepts

- Antonyms for Traits: For personality traits like "querulous" (complaining), the opposite is a trait like "tolerant" (not complaining).

- Behavioral Focus: Focus on the behavior described by the word—here, complaining versus not complaining.

- Vocabulary Expansion: Learn related words: "querulous," "petulant," "whiny" are similar; "tolerant," "patient," "calm" are opposites.
Quick Tip: To identify antonyms for personality traits:
- Define the trait: "QUERULOUS" means complaining and irritable.
- Find a trait that opposes it: "tolerant" means patient and accepting.
- Eliminate unrelated words: "painful" (sensation) and "ambitious" (goals) don’t fit.
- Practice with similar words: "irritable" vs. "calm," "grumpy" vs. "cheerful."


Question 47:

Find out the word which is nearest to the opposite in meaning to the key word given in capitals: PERNICIOUS

  • (A) Harmless
  • (B) Ridiculous
  • (C) Destructive
  • (D) Beneficial
Correct Answer: (D) Beneficial
View Solution




To determine the word that is nearest to the opposite in meaning to the key word "PERNICIOUS," we need to understand the meaning of "pernicious" and compare it with the given options.

This question tests your vocabulary and ability to identify antonyms.


Step 1: Understand the Meaning of "PERNICIOUS"

The word "PERNICIOUS" means having a harmful or destructive effect, often in a subtle or gradual way (e.g., "The pernicious effects of smoking can take years to appear.").

It describes something that causes serious harm, injury, or damage, often in a way that’s not immediately obvious.

To find the opposite, we need a word that means helpful, positive, or not harmful.


Step 2: Analyze Option A - "Harmless"

Option A is "Harmless," which means not causing or capable of causing harm (e.g., "The snake is harmless and won’t bite.").

"Harmless" directly opposes the idea of harm, which is central to "pernicious."

While "pernicious" means actively harmful or destructive, "harmless" means not harmful at all, making it a potential antonym.

For example, a "pernicious rumor" causes harm, while a "harmless rumor" does not.

Option A is a strong candidate, but let’s check the other options to see if there’s a better fit.


Step 3: Analyze Option B - "Ridiculous"

Option B is "Ridiculous," which means absurd, laughable, or deserving of mockery (e.g., "His ridiculous outfit made everyone laugh.").

"Ridiculous" describes something foolish or silly, but it doesn’t directly relate to harm or benefit.

A "pernicious" thing causes harm, but a "ridiculous" thing is simply absurd—it neither causes nor prevents harm inherently.

Therefore, "ridiculous" is not the opposite of "pernicious."

Option B is incorrect.


Step 4: Analyze Option C - "Destructive"

Option C is "Destructive," which means causing damage or destruction (e.g., "The destructive storm ruined the village.").

"Destructive" is actually a synonym of "pernicious," as both describe something that causes harm or damage.


Since we’re looking for the opposite of "pernicious," "destructive" cannot be the correct answer.

Option C is incorrect.


Step 5: Analyze Option D - "Beneficial"

Option D is "Beneficial," which means having a positive or helpful effect; advantageous (e.g., "Eating vegetables is beneficial to your health.").

"Beneficial" describes something that promotes well-being or provides a positive outcome, which directly opposes "pernicious," as "pernicious" describes something that causes harm or destruction.

For example, a "pernicious habit" (like smoking) harms health, while a "beneficial habit" (like exercising) improves health.

Option D is also a strong candidate, so let’s compare it with option A to determine the best fit.


Step 6: Compare Options A ("Harmless") and D ("Beneficial")

- "Harmless" (A) means not causing harm, which is a direct negation of "pernicious" (causing harm).

- "Beneficial" (D) means actively causing good or providing a benefit, which goes beyond just not causing harm—it implies a positive effect.

The question asks for the word "nearest to the opposite" of "pernicious."

- "Harmless" is a neutral term—it means the absence of harm.

- "Beneficial" is a positive term—it means the presence of good.

Since "pernicious" is actively harmful (a negative quality), its most direct opposite would be something actively good, like "beneficial," rather than something neutral like "harmless."

For example:

- A "pernicious policy" causes harm to society.

- A "beneficial policy" improves society (direct opposite).

- A "harmless policy" simply doesn’t cause harm, but it doesn’t necessarily improve anything.

"Beneficial" more completely opposes "pernicious" by reversing its effect (from harmful to helpful), while "harmless" only negates the harm without adding a positive effect.

Therefore, option D, "Beneficial," is the better answer.


Step 7: Why the Other Options Are Incorrect - A Broader Perspective

- Option A ("Harmless"): While "harmless" negates harm, it’s a neutral term, not a positive one, so it’s less directly opposite to "pernicious" than "beneficial."

- Option B ("Ridiculous"): "Ridiculous" is unrelated to harm or benefit, making it irrelevant.

- Option C ("Destructive"): "Destructive" is a synonym, not an antonym, of "pernicious."


Step 8: Key Concepts

- Antonyms and Intensity: For a strongly negative word like "pernicious," the best antonym is often a strongly positive word like "beneficial," rather than a neutral one like "harmless."

- Nuance in Meaning: "Harmless" negates harm; "beneficial" adds a positive effect, making it a more complete opposite.

- Vocabulary Expansion: Learn related words: "pernicious," "destructive," "harmful" are similar; "beneficial," "helpful," "positive" are opposites.
Quick Tip: To find antonyms for impact-related words:
- Define the word: "PERNICIOUS" means harmful or destructive.
- Look for a word with the opposite impact: "beneficial" means helpful and positive.
- Consider intensity: "harmless" is neutral, but "beneficial" is actively positive, better opposing "pernicious."
- Practice with similar words: "harmful" vs. "helpful," "dangerous" vs. "safe."


Question 48:

Find out the word which is nearest to the opposite in meaning to the key word given in capitals: EXACERBATE

  • (A) Alleviate
  • (B) Incriminate
  • (C) Aggravate
  • (D) Vibrant
Correct Answer: (A) Alleviate
View Solution




To determine the word that is nearest to the opposite in meaning to the key word "EXACERBATE," we need to understand the meaning of "exacerbate" and compare it with the given options.

This question tests your vocabulary and ability to identify antonyms.


Step 1: Understand the Meaning of "EXACERBATE"

The word "EXACERBATE" means to make a situation, problem, or feeling worse or more severe (e.g., "His harsh words exacerbated the conflict.").

It is often used in the context of worsening a negative situation, such as increasing pain, tension, or difficulty.

To find the opposite, we need a word that means to make a situation better, less severe, or to improve it.


Step 2: Analyze Option A - "Alleviate"

Option A is "Alleviate," which means to make a problem, pain, or situation less severe or to relieve it (e.g., "The medicine alleviated her headache.").

"Alleviate" directly opposes "exacerbate" because it means to improve or lessen a negative situation, while "exacerbate" means to worsen it.

For example:

- "Stress exacerbates the illness" (makes it worse).

- "Rest alleviates the illness" (makes it better).

Option A seems to be the correct choice, but let’s check the other options to confirm.


Step 3: Analyze Option B - "Incriminate"

Option B is "Incriminate," which means to make someone appear guilty of a crime or wrongdoing (e.g., "The evidence incriminated the suspect.").

"Incriminate" is related to blame or guilt, not to the severity of a situation.

"Exacerbate" is about worsening a situation, while "incriminate" is about accusing someone—it’s a completely different context.

Therefore, "incriminate" is not the opposite of "exacerbate."

Option B is incorrect.


Step 4: Analyze Option C - "Aggravate"

Option C is "Aggravate," which means to make a situation worse or more serious, or to annoy someone (e.g., "His behavior aggravated the problem.").

"Aggravate" is actually a synonym of "exacerbate," as both describe the act of making something worse.

Since we’re looking for the opposite of "exacerbate," "aggravate" cannot be the correct answer.

Option C is incorrect.


Step 5: Analyze Option D - "Vibrant"

Option D is "Vibrant," which means full of energy, life, or bright color (e.g., "The vibrant festival attracted many visitors.").

"Vibrant" describes something lively or energetic, but it doesn’t relate to worsening or improving a situation.

"Exacerbate" is about the severity of a problem, while "vibrant" is about energy or liveliness—it’s not relevant to the context.

Therefore, "vibrant" is not the opposite of "exacerbate."

Option D is incorrect.


Step 6: Confirm the Correct Answer

Let’s review:

- "Alleviate" (A) means to lessen or improve a situation, directly opposing "exacerbate" (worsen).

- "Incriminate" (B) is about blame, not severity, so it’s unrelated.

- "Aggravate" (C) is a synonym of "exacerbate," not an antonym.

- "Vibrant" (D) describes energy, not the act of improving or worsening a situation.

Option A, "Alleviate," is indeed the word nearest to the opposite of "exacerbate," as it reverses the action of worsening by improving or relieving the situation.


Step 7: Why the Other Options Are Incorrect - A Broader Perspective

- Option B ("Incriminate"): "Incriminate" deals with guilt, not the severity of a situation, so it’s irrelevant.

- Option C ("Aggravate"): "Aggravate" means the same as "exacerbate," making it a synonym, not an antonym.

- Option D ("Vibrant"): "Vibrant" is about energy and liveliness, not about improving or worsening a situation.


Step 8: Key Concepts

- Antonyms for Actions: For a verb like "exacerbate" (make worse), the antonym is a verb like "alleviate" (make better).

- Contextual Relevance: Ensure the antonym fits the context—here, "exacerbate" is about severity, so the opposite must address severity.

- Vocabulary Expansion: Learn related words: "exacerbate," "aggravate," "worsen" are similar; "alleviate," "relieve," "ease" are opposites.
Quick Tip: To find antonyms for verbs:
- Define the verb: "EXACERBATE" means to make worse.
- Find a verb that reverses the action: "alleviate" means to make better.
- Eliminate synonyms: "aggravate" also means to make worse, so it’s not the opposite.
- Practice with similar verbs: "worsen" vs. "improve," "intensify" vs. "reduce."


Question 49:

In the question, a sentence has been jumbled. Choose the proper sequence which are labelled P, Q, R, and S to construct a logical sentence:

The concert

P: that I have ever attended

Q: was the most amazing

R: with incredible performances

S: by talented musicians

  • (A) SQRP
  • (B) RPSQ
  • (C) PRSQ
  • (D) RSQP
Correct Answer: (D) RSQP
View Solution




We need to arrange the jumbled parts P, Q, R, and S to form a meaningful and grammatically correct sentence starting with "The concert."


Given parts:

P: that I have ever attended

Q: was the most amazing

R: with incredible performances

S: by talented musicians


Let's analyze each option:

Option A: SQRP

S: by talented musicians

Q: was the most amazing

R: with incredible performances

P: that I have ever attended

Sentence: "The concert by talented musicians was the most amazing with incredible performances that I have ever attended."

This sentence is awkward because "with incredible performances" placed after "was the most amazing" is less natural. Also, "that I have ever attended" at the end makes the sentence clumsy.


Option B: RPSQ

R: with incredible performances

P: that I have ever attended

S: by talented musicians

Q: was the most amazing

Sentence: "The concert with incredible performances that I have ever attended by talented musicians was the most amazing."

This is grammatically incorrect and confusing because "by talented musicians" comes after "that I have ever attended," disrupting the flow.


Option C: PRSQ

P: that I have ever attended

R: with incredible performances

S: by talented musicians

Q: was the most amazing

Sentence: "The concert that I have ever attended with incredible performances by talented musicians was the most amazing."

This is close but the phrase "that I have ever attended with incredible performances" is awkward and makes the sentence hard to follow.


Option D: RSQP

R: with incredible performances

S: by talented musicians

Q: was the most amazing

P: that I have ever attended

Sentence: "The concert with incredible performances by talented musicians was the most amazing that I have ever attended."

This sentence is grammatically correct and logical. It clearly describes the concert and ends with a relative clause "that I have ever attended," which fits well.


Therefore, the correct and most logical sequence is RSQP, which corresponds to option (D).
Quick Tip: When rearranging jumbled sentences:
- Look for the main clause and identify modifiers.
- Relative clauses like "that I have ever attended" usually come at the end.
- Prepositional phrases describing characteristics (e.g., "with incredible performances," "by talented musicians") typically come before the main verb.
- Ensure the sentence flows naturally and is grammatically correct.


Question 50:

In the question, a sentence has been jumbled. Choose the proper sequence which are labelled P, Q, R, and S to construct a logical sentence:

She

P: to visit the historical sites

Q: and immerse herself

R: traveled to a foreign country

S: in the local culture

  • (A) RQPS
  • (B) QSRP
  • (C) RPSQ
  • (D) SQRP
Correct Answer: (A) RQPS
View Solution




We need to arrange the jumbled parts P, Q, R, and S to form a meaningful and grammatically correct sentence starting with "She."


Given parts:

P: to visit the historical sites

Q: and immerse herself

R: traveled to a foreign country

S: in the local culture


Let's analyze each option:

Option A: RQPS

R: traveled to a foreign country

Q: and immerse herself

P: to visit the historical sites

S: in the local culture

Sentence: "She traveled to a foreign country and immerse herself to visit the historical sites in the local culture."

Though "immerse" should be "immersed" to maintain tense consistency, the sequence logically fits and conveys a meaningful idea about traveling and experiencing culture. This is the closest and most logical option.


Option B: QSRP

Q: and immerse herself

S: in the local culture

R: traveled to a foreign country

P: to visit the historical sites

Sentence: "She and immerse herself in the local culture traveled to a foreign country to visit the historical sites."

This is grammatically incorrect and does not start well with "She."


Option C: RPSQ

R: traveled to a foreign country

P: to visit the historical sites

S: in the local culture

Q: and immerse herself

Sentence: "She traveled to a foreign country to visit the historical sites in the local culture and immerse herself."

This sentence is almost correct and logical but the placement of "and immerse herself" at the end makes it slightly awkward.


Option D: SQRP

S: in the local culture

Q: and immerse herself

R: traveled to a foreign country

P: to visit the historical sites

Sentence: "She in the local culture and immerse herself traveled to a foreign country to visit the historical sites."

This is grammatically incorrect and illogical.
Quick Tip: For sentence rearrangement:
- Identify the subject and main verb first.
- Link clauses logically with conjunctions like "and."
- Ensure modifiers like "to visit the historical sites" and "in the local culture" come after the main action.
- Check tense consistency when reconstructing sentences.


Question 51:

In the question, a sentence has been jumbled. Choose the proper sequence which are labelled P, Q, R, and S to construct a logical sentence:

The team

P: strategies for the game

Q: discussed various

R: in order to improve their performance

S: during the halftime break

  • (A) PSRQ
  • (B) QSPR
  • (C) QPRS
  • (D) SRPQ
Correct Answer: Question Cancelled
View Solution




This question has been cancelled and therefore no correct sequence or solution is provided.
Quick Tip: If a question is cancelled, it means it should be disregarded in assessments or practice.
Always verify if the question has an official cancellation notice before spending time solving it.


Question 52:

In the question, a sentence has been jumbled. Choose the proper sequence which are labelled P, Q, R, and S to construct a logical sentence:

The novel

P: captivated readers

Q: with its intriguing plot

R: and well-developed characters

S: and became a bestseller

  • (A) QRPS
  • (B) SRPQ
  • (C) QSRP
  • (D) RSPQ
Correct Answer: (A) QRPS
View Solution




The sentence parts are:

P: captivated readers

Q: with its intriguing plot

R: and well-developed characters

S: and became a bestseller


We need to arrange these parts to form a coherent and logical sentence following the sequence of ideas.


Step 1: Identify the introductory phrase

The phrase "with its intriguing plot" (Q) and "and well-developed characters" (R) both describe attributes that draw readers' interest. They logically precede the action "captivated readers" (P).


Step 2: Analyze the flow

- Start with "The novel" (given)

- Next, mention the qualities that made it interesting: Q ("with its intriguing plot")

- Add R ("and well-developed characters") to complete the description of the novel's appealing features

- Then, describe the effect on the readers: P ("captivated readers")

- Finally, mention the result: S ("and became a bestseller")


Putting it together:

"The novel with its intriguing plot and well-developed characters captivated readers and became a bestseller."


This matches sequence Q R P S, which corresponds to option (A).


Step 3: Why other options are incorrect

- Option B (S R P Q) starts with "and became a bestseller," which cannot logically start the sentence.

- Option C (Q S R P) disrupts the flow by placing "and became a bestseller" (S) before "and well-developed characters" (R).

- Option D (R S P Q) also places the phrases out of logical order.


Therefore, option (A) is the correct sequence. Quick Tip: When unscrambling sentences:
- Identify descriptive phrases that logically precede actions.
- Look for cause-effect relationships.
- Place phrases that describe qualities before the resulting actions.
- Read the sequence aloud to check if it forms a coherent sentence.


Question 53:

In the question, a sentence has been jumbled. Choose the proper sequence which are labelled P, Q, R, and S to construct a logical sentence:

The students

P: to participate in the competition

Q: were encouraged

R: and showcase their talents

S: by the school faculty

  • (A) QSRP
  • (B) SRPQ
  • (C) RPQS
  • (D) QPSR
Correct Answer: Question Cancelled
View Solution




The sentence parts are:

P: to participate in the competition

Q: were encouraged

R: and showcase their talents

S: by the school faculty


Since the question has been cancelled, no correct sequence is applicable here.


Note: Sometimes, questions are cancelled if the sentence fragments do not logically fit into a coherent sentence or if there is an error in the question options. Quick Tip: If a question is cancelled, it means it is invalid or ambiguous.
Always look for clear and logical connections between parts when unscrambling sentences.
If no proper sequence forms a meaningful sentence, the question may be cancelled.


Question 54:

Read the following passage and answer the question by choosing among the four alternatives given:

The year was 1888, and the streets of London were plagued by a series of gruesome murders. The infamous killer, Jack the Ripper, struck fear into the hearts of the city's residents as he preyed upon vulnerable women in the dimly lit alleys of Whitechapel. The murders were characterized by their brutality, with the victims being mutilated in a manner that suggested a deep-seated rage and a twisted sense of pleasure for the perpetrator. Despite the efforts of Scotland Yard and a fervent public outcry, Jack the Ripper managed to elude capture, leaving behind a trail of unsolved mysteries and countless theories about his true identity. Over the years, numerous suspects have emerged, ranging from ordinary citizens to prominent figures in society. The case has fascinated criminologists, historians, and amateur sleuths alike, who have tirelessly analyzed the available evidence, examined witness testimonies, and delved into the historical context of the time. Even to this day, the identity of Jack the Ripper remains one of the most enigmatic and perplexing mysteries in criminal history. Who was the man behind the grisly crimes? What drove him to commit such heinous acts? Will we ever uncover the truth behind Jack the Ripper's reign of terror?

Why is the case of Jack the Ripper considered one of the most perplexing mysteries in criminal history?

  • (A) The murders occurred in various locations simultaneously.
  • (B) The killer was never identified or captured.
  • (C) The victims were all wealthy individuals.
  • (D) The case involved complex forensic evidence.
Correct Answer: (B) The killer was never identified or captured.
View Solution




The passage describes a series of brutal murders in London in 1888 committed by Jack the Ripper, whose identity remains unknown despite extensive investigation by Scotland Yard and public efforts.


Step 1: Understanding the core mystery

The central reason the case is perplexing is that the killer was never caught or identified, which left many unanswered questions and theories.


Step 2: Evaluate the options

- Option (A) is incorrect because the passage does not mention simultaneous murders at different locations.

- Option (B) is correct since the killer’s identity and capture remained elusive.

- Option (C) is incorrect; the victims were vulnerable women, not wealthy individuals.

- Option (D) is incorrect because forensic science was limited then and is not highlighted as a complexity in the passage.


Thus, option (B) best captures why the case remains one of the most perplexing mysteries in criminal history.
Quick Tip: To answer comprehension questions effectively:
- Focus on the main idea of the passage.
- Eliminate options contradicted by the text.
- Select the answer that directly addresses the question.


Question 55:

Read the following passage and answer the question by choosing among the four alternatives given:

In the distant land of Euphoria, a realm shrouded in magic and mystery, a prophecy has long foretold the arrival of a chosen hero. According to ancient lore, this hero possesses extraordinary powers and is destined to save the kingdom from impending doom. The prophecy speaks of a perilous quest that the hero must undertake, facing formidable challenges and overcoming insurmountable odds. Legends whisper of a hidden artifact, known as the Crystal of Enlightenment, which holds the key to unlocking the hero's true potential. It is said that the Crystal is guarded by mythical creatures and hidden deep within the treacherous Abyssal Caverns. Many have sought the Crystal, but none have returned. The fate of Euphoria hangs in the balance as the kingdom eagerly awaits the emergence of the chosen hero, the one who will bring light to the encroaching darkness.



What is the main objective of the chosen hero in the prophecy?

  • (A) To obtain the Crystal of Enlightenment.
  • (B) To explore the treacherous Abyssal Caverns.
  • (C) To uncover the secrets of the mythical creatures.
  • (D) To save the kingdom of Euphoria.
Correct Answer: (D) To save the kingdom of Euphoria.
View Solution




The passage describes a prophecy about a chosen hero whose destiny is to save the kingdom of Euphoria from impending doom. Although the hero must obtain the Crystal of Enlightenment, which is hidden and guarded by mythical creatures within the Abyssal Caverns, the main goal of the hero is clearly stated as saving the kingdom.


Step 1: Understanding the prophecy

The prophecy foresees a chosen hero with extraordinary powers who will save the kingdom from danger. The quest involves facing challenges, including obtaining the Crystal.


Step 2: Analyze options

- Option (A) mentions obtaining the Crystal, which is a task in the quest but not the ultimate objective.

- Option (B) refers to exploring the caverns, which is part of the journey.

- Option (C) is about uncovering secrets of mythical creatures, which is incidental.

- Option (D) captures the overarching purpose — saving the kingdom from doom.


Thus, the main objective of the chosen hero is to save the kingdom of Euphoria, making option (D) the correct choice.
Quick Tip: When answering questions based on passages, focus on the main idea or the ultimate goal stated by the author.
Supporting tasks or challenges are important but secondary to the main objective.


Question 56:

Read the following passage and answer the question by choosing among the four alternatives given:

In the ever-evolving world of business, competition is fierce, and companies are constantly seeking strategies to gain a competitive edge. One such strategy is innovation. Innovation is the lifeblood of business growth, as it allows companies to develop groundbreaking products, services, and processes that meet the changing needs and desires of customers. Innovative companies understand that standing still is not an option; they actively foster a culture of creativity and exploration. They encourage employees to think outside the box, take calculated risks, and embrace new ideas. These companies invest in research and development, allocate resources for experimentation, and collaborate with external partners to harness the power of collective intelligence. By embracing innovation, companies can stay ahead of the curve, seize new opportunities, and create a sustainable advantage in the dynamic marketplace.


Why is innovation important for businesses?

  • (A) To create a sense of competition among employees.
  • (B) To maintain a static and unchanging product lineup.
  • (C) To meet the evolving needs of customers.
  • (D) To avoid collaborating with external partners.
Correct Answer: (C) To meet the evolving needs of customers.
View Solution




The passage explains that innovation is crucial because it allows companies to develop new products and services that adapt to the changing needs and desires of customers. Without innovation, companies would struggle to keep up with market demands.


Step 1: Identify key points

- Innovation drives business growth.

- It enables companies to create groundbreaking products and services.

- It helps meet changing customer needs.


Step 2: Analyze options

- Option (A) is incorrect as competition among employees is not the main focus.

- Option (B) is opposite to the passage's message; innovation opposes static product lines.

- Option (D) is incorrect; the passage promotes collaboration with external partners.

- Option (C) correctly captures the main importance of innovation.


Therefore, the correct answer is (C) To meet the evolving needs of customers.
Quick Tip: When answering comprehension questions, focus on the main reasons or benefits highlighted in the passage.
Eliminate options that contradict the passage or are irrelevant.


Question 57:

Read the following passage and answer the question by choosing among the four alternatives given:

The Paris Agreement, signed in 2015, represents a landmark event in the realm of international relations and environmental policy. It was a global effort to address the pressing issue of climate change and mitigate its impacts. The agreement aimed to limit global warming to well below 2 degrees Celsius above pre-industrial levels and to pursue efforts to limit the temperature increase to 1.5 degrees Celsius. Under the Paris Agreement, countries committed to implementing measures to reduce greenhouse gas emissions, adapt to the impacts of climate change, and provide financial assistance to developing nations in their climate actions. The agreement emphasized the principle of common but differentiated responsibilities, recognizing that developed countries should take the lead in emission reductions and provide support to developing countries. The Paris Agreement garnered widespread international support, with 197 parties signing and 191 ratifying the agreement. It symbolizes a collective global effort to address the existential threat of climate change and marks a significant step forward in international cooperation for a sustainable future.


What does the Paris Agreement emphasize in its approach to climate change?

  • (A) Sustainable economic growth and development.
  • (B) Equal contribution from all nations.
  • (C) Common but differentiated responsibilities.
  • (D) Strict regulations and penalties.
Correct Answer: (C) Common but differentiated responsibilities.
View Solution




The passage explains that the Paris Agreement emphasizes the principle of common but differentiated responsibilities. This means all countries share the responsibility to combat climate change, but developed nations, having historically contributed more to greenhouse gas emissions, are expected to lead in emission reductions and provide financial support to developing countries.


Step 1: Understand the principle

This approach recognizes the different capabilities and historical responsibilities of nations, promoting fairness and equity.


Step 2: Analyze options

- Option (A) is broader and not the specific emphasis of the Agreement.

- Option (B) is incorrect as the Agreement does not require equal contributions, but differentiated ones.

- Option (D) is not highlighted; while regulations exist, the principle focuses on responsibilities.


Therefore, option (C) correctly captures the Agreement's core emphasis.
Quick Tip: Remember key principles of global agreements by focusing on their fairness and equity measures.
“Common but differentiated responsibilities” is a hallmark of international climate policies.


Question 58:

Read the following passage and answer the question by choosing among the four alternatives given:

The Olympic Games, held every four years, bring together athletes from around the world in a celebration of sport and unity. The origins of the modern Olympics can be traced back to ancient Greece, where athletic competitions were held to honor the gods. Today, the Olympic Games encompass a wide range of sports, from track and field to swimming, gymnastics, and beyond. The Games provide a platform for athletes to showcase their skills, determination, and sportsmanship on an international stage. Athletes spend years honing their craft, pushing the boundaries of human potential, and striving for excellence. The Olympic Games foster a spirit of camaraderie, where competitors from different nations come together in friendly rivalry, promoting peace and understanding through the universal language of sports. The excitement, passion, and memorable moments created during the Olympic Games have captivated audiences for decades, inspiring generations to pursue their athletic dreams and embrace the values of fair play, perseverance, and respect.


What is the primary objective of the Olympic Games?

  • (A) To honor the gods of ancient Greece.
  • (B) To bring athletes from around the world together in friendly competition.
  • (C) To achieve worldwide fame and recognition for participating athletes.
  • (D) To promote commercial interests and generate revenue.
Correct Answer: (B) To bring athletes from around the world together in friendly competition.
View Solution




The passage highlights that the Olympic Games are a global event that unites athletes worldwide in a spirit of friendly rivalry and sportsmanship.


Step 1: Understand the essence

The Games promote peace, unity, and understanding among nations through sports, transcending political and cultural differences.


Step 2: Analyze options

- Option (A) refers to the historical origin but is not the current primary objective.

- Option (B) correctly reflects the main purpose: fostering friendly competition and unity.

- Option (C) and (D) focus on individual fame or commercial aspects, which are secondary.


Hence, option (B) is the correct answer.
Quick Tip: Focus on the core values of global events like the Olympics: unity, peace, and friendly competition.
Distinguish between historical origins and present-day objectives.


Question 59:

Identify the meaning of the idiom or phrase in capitals from the choices. She had to BITE THE BULLET in the current scenario.

  • (A) Literally bite on a bullet
  • (B) Avoid confronting the fears
  • (C) Overcome fears and face a difficult situation
  • (D) Seek help from others to handle fears
Correct Answer: (C) Overcome fears and face a difficult situation.
View Solution




The idiom "bite the bullet" means to endure a painful or difficult situation bravely and without complaining.


Step 1: Literal vs idiomatic meaning

Literally biting a bullet refers to holding something painful in the mouth to endure pain, historically during surgeries.


Step 2: Meaning in context

Figuratively, it means accepting and facing a challenging or unpleasant situation.


Step 3: Analyze options

- Option (A) is literal and incorrect.

- Option (B) implies avoidance, opposite of the idiom’s meaning.

- Option (C) correctly captures the meaning.

- Option (D) suggests seeking help, which is different from facing the situation oneself.


Therefore, option (C) is the correct interpretation of the idiom.
Quick Tip: Learn idioms by their figurative meanings, not literal words.
"To bite the bullet" means to bravely face difficulty without avoiding it.


Question 60:

Identify the meaning of the idiom or phrase in capitals from the choices. He always KEEPS HIS CARDS CLOSE TO THE CHEST.

  • (A) Keeps his plans and intentions secret
  • (B) Plays cards strategically
  • (C) Reveals his plans and intentions openly
  • (D) Shows his emotions easily
Correct Answer: (A) Keeps his plans and intentions secret
View Solution




The idiom "keeps his cards close to the chest" means to keep one's plans, thoughts, or intentions secret and not reveal them to others.


Step 1: Understanding the idiom

This phrase originates from card games where holding cards close to the chest prevents opponents from seeing them.


Step 2: Analyze the options

- Option (A) accurately describes the meaning of the idiom.

- Option (B) literally refers to playing cards but does not capture the figurative meaning.

- Option (C) is opposite to the idiom’s meaning.

- Option (D) relates to emotions, which is unrelated here.


Therefore, option (A) is the correct interpretation of the idiom. Quick Tip: To understand idioms:
- Learn the figurative meanings, which often differ from literal meanings.
- Use context to infer the correct interpretation.
- Practice common idioms regularly for better recall.


Question 61:

Who was conferred with the Highest Civilian honour of Kerala, "Kerala Jyothi" recently?

  • (A) Mammootty
  • (B) M T Vasudevan Nair
  • (C) K J Yesudas
  • (D) Kochouseph Chittilappilly
Correct Answer: (B) M T Vasudevan Nair
View Solution




The "Kerala Jyothi" is the highest civilian honour conferred by the Government of Kerala to recognize outstanding contributions in various fields.


Step 1: About the recipient

M T Vasudevan Nair is a renowned Indian author, screenplay writer, and film director from Kerala, known for his significant contributions to Malayalam literature and cinema.


Step 2: Recent recognition

Recently, he was awarded the "Kerala Jyothi" for his lifetime achievements and immense impact on the cultural landscape of Kerala.


Step 3: About other options

- Mammootty and K J Yesudas are also celebrated personalities from Kerala, but they were not conferred this particular honour recently.

- Kochouseph Chittilappilly is an industrialist and philanthropist, not the recipient of this award recently.


Thus, option (B) is the correct answer. Quick Tip: For current affairs questions:
- Keep updated with recent awards and honours announced by government sources.
- Note key personalities in arts, culture, and public service.
- Follow official news releases and reputable sources regularly.


Question 62:

'Vayu Link' communication system which helps pilots to deal with adverse conditions by maintaining the communication network with the base station has been developed by?

  • (A) ISRO
  • (B) Indian Air Force
  • (C) Hindustan Aeronautics Limited
  • (D) NTRO
Correct Answer: (B) Indian Air Force
View Solution




The 'Vayu Link' communication system is an advanced system developed specifically to assist pilots in maintaining continuous and reliable communication with their base stations, especially during adverse conditions such as poor weather or technical challenges.


Step 1: Understanding the system

Maintaining communication is critical for flight safety, and systems like 'Vayu Link' ensure that pilots can stay connected without interruption.


Step 2: Developer identification

The Indian Air Force, which operates and manages military aviation and related systems, has developed this technology to enhance operational efficiency and safety of its pilots.


Step 3: Why other options are incorrect

- ISRO is mainly involved in space research and satellite launches, not specifically in pilot communication systems.

- Hindustan Aeronautics Limited manufactures aircraft but is not credited with developing this system.

- NTRO (National Technical Research Organisation) is an intelligence agency, unrelated to pilot communication systems.


Hence, option (B) is the correct answer. Quick Tip: To answer technology-related questions:
- Identify the organization’s primary function.
- Match the technology with the agency best suited for its development.
- Keep updated on recent technological advancements in defence and aviation.


Question 63:

Who has been formally declared as the new Monarch of Britain recently?

  • (A) Charles III
  • (B) George IX
  • (C) Charles VI
  • (D) Edward VII
Correct Answer: (A) Charles III
View Solution




Following the passing of Queen Elizabeth II, Charles III was formally declared the new monarch of Britain, continuing the British royal lineage.


Step 1: Understanding the succession

The eldest son of Queen Elizabeth II, Charles ascended the throne as per the rules of hereditary monarchy.


Step 2: Analysis of options

- Charles III is the correct title taken by the new monarch.

- Other options such as George IX, Charles VI, and Edward VII are previous monarchs or incorrect titles.


Therefore, option (A) is the correct answer. Quick Tip: For current affairs on royal events:
- Follow official announcements from the British Royal Family.
- Remember the numbering convention for monarchs (e.g., Charles III).
- Keep updated with recent news for any changes in monarchy.


Question 64:

India's first semi conductor manufacturing plant will be opened in

  • (A) Telangana
  • (B) Kerala
  • (C) Maharashtra
  • (D) Gujarat
Correct Answer: (D) Gujarat
View Solution




India is setting up its first semiconductor manufacturing plant to boost its electronics and chip production capabilities. The plant will be located in Gujarat, a state actively promoting industrial and technological infrastructure.


Step 1: Importance of semiconductor plants

Semiconductor manufacturing is critical for electronic devices, and establishing this plant will reduce India's dependence on imports.


Step 2: Choice of location

Gujarat has been chosen due to its favorable industrial policies, infrastructure, and connectivity.


Hence, option (D) is the correct answer. Quick Tip: Remember key recent developments in technology and industry by following government announcements.
Note states investing in cutting-edge manufacturing for economic growth.


Question 65:

Which Panchayat in Kerala was declared to be the first fully digitally literate panchayat in India?

  • (A) Chavara
  • (B) Irikkur
  • (C) Pullampara
  • (D) Vijayapuram
Correct Answer: (C) Pullampara
View Solution




Pullampara Panchayat in Kerala was declared the first fully digitally literate panchayat in India, meaning all its residents have basic digital literacy skills.


Step 1: Understanding digital literacy

Digital literacy enables citizens to use digital devices and the internet effectively.


Step 2: Significance of Pullampara

Pullampara's achievement reflects Kerala's commitment to advancing digital inclusion and governance.


Therefore, option (C) is correct. Quick Tip: For questions on digital initiatives, track government programs and state achievements.
Know key milestones in digital literacy across India.


Question 66:

Which organization was founded by Gopal Krishna Gokhale?

  • (A) Servants of India Society
  • (B) Poona Sarvajanik Sabha
  • (C) Bombay Presidency Association
  • (D) Hindustani Socialist Republican Association
Correct Answer: (A) Servants of India Society
View Solution




Gopal Krishna Gokhale, a social reformer and freedom fighter, founded the Servants of India Society in 1905 to promote education, social reform, and national development.


Step 1: Purpose of the Society

It aimed to train dedicated individuals to serve the country and spread awareness about social issues.


Step 2: Distinguishing from other organizations

Other organizations were founded by different leaders for various political or social causes.


Thus, option (A) is correct. Quick Tip: Associate leaders with the organizations they founded to remember historical facts.
Focus on their contributions and the goals of their organizations.


Question 67:

Which Mughal emperor was known for his religious tolerance and the establishment of a new religion called Din-i-Ilahi?

  • (A) Jahangir
  • (B) Akbar
  • (C) Shah Jahan
  • (D) Aurangzeb
Correct Answer: (B) Akbar
View Solution




Emperor Akbar is renowned for his policy of religious tolerance and for founding Din-i-Ilahi, a syncretic religion that sought to combine elements of various faiths to promote unity.


Step 1: Akbar’s religious policies

He abolished the jizya tax on non-Muslims and included people of different faiths in his administration.


Step 2: Din-i-Ilahi

Though it did not gain many followers, it represented Akbar’s vision for harmony among religions.


Therefore, option (B) is correct. Quick Tip: Remember key Mughal emperors and their contributions:
Akbar – religious tolerance and Din-i-Ilahi; Aurangzeb – orthodox policies.


Question 68:

Which is the name of the famous temple built during the Vijayanagara Empire?

  • (A) Meenakshi Temple
  • (B) Brihadeeswarar Temple
  • (C) Hoysaleswara Temple
  • (D) Virupaksha Temple
Correct Answer: (D) Virupaksha Temple
View Solution




The Virupaksha Temple at Hampi is a famous temple constructed during the Vijayanagara Empire and is a significant architectural and cultural landmark.


Step 1: Location and importance

It served as a major religious center for the empire and is dedicated to Lord Shiva.


Step 2: Distinguishing from other temples

- Meenakshi Temple is in Madurai;

- Brihadeeswarar Temple is from the Chola period;

- Hoysaleswara Temple belongs to the Hoysala dynasty.


Hence, option (D) is correct. Quick Tip: Associate famous temples with their dynasties/empires to remember history.
Virupaksha Temple – Vijayanagara Empire; Brihadeeswarar – Cholas.


Question 69:

Who among the following played a key role in persuading Mahatma Gandhi to initiate a movement supporting the Khilafat?

  • (A) Abdul Ghaffar Khan
  • (B) Maulana Ali
  • (C) Syed Ahmad Bareili
  • (D) Mohammad Ali Jinnah
Correct Answer: (B) Maulana Ali
View Solution




Maulana Muhammad Ali was one of the prominent leaders who persuaded Mahatma Gandhi to support the Khilafat Movement, which aimed to protect the Ottoman Caliphate after World War I.


Step 1: Understanding the Khilafat Movement

It was a pan-Islamic movement seeking to preserve the Ottoman Caliph as the spiritual leader of Muslims.


Step 2: Role of Maulana Ali

He, along with his brother Maulana Shaukat Ali, worked to unite Indian Muslims and sought Gandhi’s support for non-cooperation against British rule.


Thus, option (B) is correct. Quick Tip: Know key leaders and their roles in freedom and social movements.
The Khilafat Movement united Muslims and Hindus in a common cause during the freedom struggle.


Question 70:

Who was the first female Cabinet Minister of India?

  • (A) Sarojini Naidu
  • (B) Amrit Kaur
  • (C) Sucheta Kriplani
  • (D) Indira Gandhi
Correct Answer: (B) Amrit Kaur
View Solution




Amrit Kaur was the first female Cabinet Minister of independent India, serving as the Health Minister in Jawaharlal Nehru’s cabinet.


Step 1: Role and contributions

She played a significant role in health reforms and women’s welfare.


Step 2: Clarification of other options

- Sarojini Naidu was the first female governor;

- Sucheta Kriplani was the first female Chief Minister of a state;

- Indira Gandhi became Prime Minister later.


Therefore, option (B) is correct. Quick Tip: Remember firsts in Indian political history to answer such questions.
Distinguish between roles like Minister, Governor, Chief Minister, and Prime Minister.


Question 71:

At which location did Buddha deliver his inaugural sermon?

  • (A) Sarnath
  • (B) Bodh-Gaya
  • (C) Rajagriha
  • (D) Kapilavastu
Correct Answer: (A) Sarnath
View Solution




Buddha delivered his first sermon, known as the "Dhammacakkappavattana Sutta" (Setting in Motion the Wheel of Dharma), at Sarnath near Varanasi.


Step 1: Importance of Sarnath

This event marked the beginning of Buddha’s teaching and the foundation of Buddhism.


Step 2: Clarify other locations

- Bodh-Gaya is where Buddha attained enlightenment.

- Rajagriha and Kapilavastu are important locations in his life but not where the first sermon was given.


Thus, option (A) is correct. Quick Tip: Associate key Buddhist events with their locations for better recall.
Enlightenment at Bodh-Gaya, first sermon at Sarnath.


Question 72:

What was the objective of the Reformation movement that occurred in the 16th century Europe?

  • (A) It aimed to challenge the authority of King Henry VIII.
  • (B) It sought to strengthen the Church's control over Rome.
  • (C) It aimed to rebel against the Pope and the dominance of indulgences.
  • (D) It aimed to revive and reform the art and literature of the 16th century.
Correct Answer: (C) It aimed to rebel against the Pope and the dominance of indulgences.
View Solution




The Reformation was a religious movement in 16th century Europe that sought to reform the Roman Catholic Church, primarily opposing the sale of indulgences and papal authority.


Step 1: Martin Luther's role

He challenged church practices and authority, sparking widespread religious reform.


Step 2: Clarify options

- Option (A) is incorrect as King Henry VIII’s challenge was separate, leading to the English Reformation.

- Option (B) contradicts the reformers’ aim.

- Option (D) relates to the Renaissance, not Reformation.


Therefore, option (C) is correct. Quick Tip: Distinguish between Renaissance (art/literature) and Reformation (religious reform).
Remember key figures like Martin Luther and their objectives.


Question 73:

Sutlej river originates from which of the following lakes?

  • (A) Lake Manasarovar
  • (B) Pangong Lake
  • (C) Ebi Lake
  • (D) Lake Rakshastal
Correct Answer: (D) Lake Rakshastal
View Solution




The Sutlej River originates from Lake Rakshastal in Tibet, near Lake Manasarovar, and flows through India and Pakistan.


Step 1: Geographic facts

Lake Rakshastal is a saltwater lake adjacent to the freshwater Lake Manasarovar.


Step 2: Importance of origin

Knowing the source helps in understanding river systems and hydrology of the region.


Thus, option (D) is correct. Quick Tip: Remember major rivers' sources along with key geographic features.
Sutlej - Lake Rakshastal; Ganges - Gangotri Glacier.


Question 74:

Sagar Matha is another name of which of the following mountain Peak?

  • (A) Nanda Devi
  • (B) Kamel
  • (C) Mt Everest
  • (D) Annapurna
Correct Answer: (C) Mt Everest
View Solution




"Sagar Matha" meaning "Mother of the Sea" is the Nepali name for Mount Everest, the highest peak in the world.


Step 1: Significance of the name

It reflects the mountain's towering presence above all other peaks and its snow cover appearing like the sea.


Step 2: Other options

Other peaks have distinct local names and significance but are not known as Sagar Matha.


Hence, option (C) is correct. Quick Tip: Learn local and native names of famous geographic landmarks.
Mt Everest - Sagar Matha (Nepali), Sagarmatha.


Question 75:

Which of the following is a cold desert in India?

  • (A) Thar Desert
  • (B) Kutch Desert
  • (C) Spiti Valley Desert
  • (D) Deccan Thorn Scrub Forests
Correct Answer: (C) Spiti Valley Desert
View Solution




Spiti Valley Desert, located in Himachal Pradesh, is a cold desert characterized by its high altitude and low temperatures, unlike the hot Thar or Kutch deserts.


Step 1: Features of cold deserts

Cold deserts experience freezing winters and limited vegetation.


Step 2: Distinguishing deserts

- Thar and Kutch are hot deserts;

- Deccan Thorn Scrub Forests are semi-arid regions, not deserts.


Therefore, option (C) is correct. Quick Tip: Understand differences between cold and hot deserts in India by location and climate.
Spiti and Ladakh are cold deserts; Thar is a hot desert.


Question 76:

The Dead Sea, known for its high salt content, is bordered by which two countries?

  • (A) Egypt and Sudan
  • (B) Israel and Jordan
  • (C) Turkey and Greece
  • (D) Iran and Iraq
Correct Answer: (B) Israel and Jordan
View Solution




The Dead Sea is a salt lake located at the lowest point on Earth's surface, approximately 430 meters below sea level.

It is renowned for its extremely high salinity, which makes it one of the saltiest bodies of water in the world, allowing people to float effortlessly.


Geographical Location:

The Dead Sea lies in the Jordan Rift Valley and is bordered by:

- Israel to the west

- Jordan to the east


It also touches the West Bank (Palestinian territories) along its western shore, but the two primary countries commonly associated with its borders are Israel and Jordan.


Why Other Options Are Incorrect:

- Egypt and Sudan are located further south along the northeastern African coast, bordering the Red Sea, not the Dead Sea.

- Turkey and Greece border parts of the Mediterranean and Aegean Seas, far from the Dead Sea region.

- Iran and Iraq are located in the Middle East but do not border the Dead Sea.
Quick Tip: - Remember the Dead Sea is situated between Israel and Jordan in the Middle East.
- Known for its unique high salinity and lowest elevation on Earth.
- Useful for geography questions related to physical features and political boundaries.


Question 77:

Which of the following is the largest gulf in the world?

  • (A) Persian Gulf
  • (B) Gulf of Mexico
  • (C) Gulf of Carpentaria
  • (D) Gulf of Mannar
Correct Answer: (B) Gulf of Mexico
View Solution




A gulf is a large coastal indentation of a sea or ocean partially enclosed by land. The Gulf of Mexico, located between the United States, Mexico, and Cuba, is the largest gulf in the world by area.


Step 1: Understanding a gulf

Gulfs serve as important areas for maritime trade, fisheries, and rich biodiversity.


Step 2: Comparison

- Persian Gulf is smaller and located in the Middle East.

- Gulf of Carpentaria is in northern Australia, smaller than the Gulf of Mexico.

- Gulf of Mannar lies between India and Sri Lanka and is much smaller.


Thus, the Gulf of Mexico is the largest gulf, making option (B) correct.
Quick Tip: Use a world map to visualize the size and location of gulfs.
The Gulf of Mexico is the largest gulf worldwide.


Question 78:

Which district in Kerala is referred to as the "Land of Looms and Lores"?

  • (A) Malappuram
  • (B) Kozhikode
  • (C) Thrissur
  • (D) Kannur
Correct Answer: (D) Kannur
View Solution




Kannur district in Kerala is famously called the "Land of Looms and Lores" because of its long tradition in handloom weaving ("looms") and rich folklore and cultural heritage ("lores").


Step 1: Explanation of the title

- "Looms" indicates the district’s significance in the textile and handloom industry.

- "Lores" refers to the extensive folk traditions, such as Theyyam, practiced there.


Step 2: Cultural importance

Kannur is known for its distinctive cultural identity, making this nickname appropriate.


Hence, option (D) is correct.
Quick Tip: Link place nicknames with their cultural or economic specialties.
Kannur’s handlooms and folk arts give it the title "Land of Looms and Lores."


Question 79:

Progress in increase in fisheries production in India was termed

  • (A) White revolution
  • (B) Green revolution
  • (C) Blue revolution
  • (D) Yellow revolution
Correct Answer: (C) Blue revolution
View Solution




The Blue Revolution refers specifically to the significant increase in fisheries and aquaculture production in India, leading to growth in fish yield and livelihoods.


Step 1: Understand the revolutions

- Green Revolution: Increase in crop production through high-yield varieties.

- White Revolution: Increase in milk production (dairy).

- Blue Revolution: Development of fisheries and aquaculture.

- Yellow Revolution: Growth in oilseed production.


Step 2: Fisheries growth

India’s Blue Revolution helped the country become a leading fish producer by enhancing sustainable fishing practices and aquaculture technology.


Therefore, option (C) is correct.
Quick Tip: Remember revolutions by their focus sectors:
Green – crops, White – dairy, Blue – fisheries, Yellow – oilseeds.


Question 80:

Which sector has the highest contribution to India's Gross National Product (GNP)?

  • (A) Tertiary sector
  • (B) Public sector
  • (C) Primary sector
  • (D) Secondary sector
Correct Answer: (A) Tertiary sector
View Solution




The tertiary sector, also known as the service sector, currently contributes the highest share to India's Gross National Product (GNP). It includes services such as banking, IT, tourism, education, and healthcare.


Step 1: Economic sectors

- Primary sector involves agriculture and raw material extraction.

- Secondary sector involves manufacturing and industry.

- Tertiary sector involves services.

- Public sector refers to government-owned enterprises cutting across these sectors.


Step 2: Current economic trends

Though agriculture (primary) employs many people, services contribute the most to the GNP due to India's expanding economy.


Hence, option (A) is correct.
Quick Tip: Differentiate sectors by their economic activities:
Services (tertiary) are growing fastest and contribute most to India’s GNP today.


Question 81:

Which function of money facilitates the expression of the exchange ratio of goods by serving as a common denomination?

  • (A) Medium of exchange
  • (B) Measure of value
  • (C) Store of value
  • (D) Standard of deferred payments
Correct Answer: (B) Measure of value
View Solution




Money serves several functions, one of which is the "measure of value" or "unit of account." This function allows money to be a common standard to express the value of goods and services, enabling comparison and exchange.


Step 1: Function explained

By assigning prices to goods and services, money helps determine the exchange ratio, i.e., how much of one good is worth in terms of another.


Step 2: Other functions

- Medium of exchange helps facilitate buying and selling.

- Store of value means money can preserve value over time.

- Standard of deferred payments allows settling debts in the future.


Hence, option (B) correctly identifies the function described.
Quick Tip: Remember the main functions of money: medium of exchange, measure of value, store of value, and standard of deferred payments.
Measure of value helps in pricing and comparing goods and services.


Question 82:

Which category of poor people keep moving up and down the poverty line?

  • (A) Chronic poor
  • (B) Transient poor
  • (C) Always poor
  • (D) Occasionally poor
Correct Answer: (B) Transient poor
View Solution




Transient poor are individuals or families who move above and below the poverty line due to temporary factors such as illness, job loss, or natural disasters.


Step 1: Define poverty categories

- Chronic poor remain in poverty for extended periods.

- Transient poor experience short-term poverty but can improve their status.

- Always poor is synonymous with chronic poor.

- Occasionally poor is not a standard category.


Step 2: Explanation

Transient poverty fluctuates due to changing circumstances, so these people sometimes have enough income and sometimes fall below the poverty line.


Thus, option (B) is correct.
Quick Tip: Learn poverty classifications to understand economic vulnerability.
Transient poor face short-term poverty, chronic poor face long-term poverty.


Question 83:

A regressive tax is a tax that follows which of the following patterns as an individual's income increases:

  • (A) Charged at a decreasing rate
  • (B) Charged at an increasing rate
  • (C) Charged at a flat percentage of the individual's income
  • (D) None of these
Correct Answer: (A) Charged at a decreasing rate
View Solution




A regressive tax is one where the tax rate decreases as the individual's income increases. This means that lower-income people pay a higher percentage of their income in taxes than higher-income people.


Step 1: Tax systems

- Progressive tax: higher income, higher tax rate.

- Regressive tax: higher income, lower tax rate.

- Proportional (flat) tax: same rate for all incomes.


Step 2: Examples

Sales taxes and excise taxes are often regressive because they take a larger share from low-income individuals.


Hence, option (A) is the correct choice.
Quick Tip: Understand tax types by their relationship between income and tax rate.
Regressive taxes affect the poor disproportionately by charging them higher rates relative to income.


Question 84:

Which of the following is an indirect tax?

  • (A) Wealth tax
  • (B) Excise tax
  • (C) Income tax
  • (D) None of these
Correct Answer: (B) Excise tax
View Solution




An indirect tax is one that is collected by an intermediary (such as a retailer) from the consumer. Excise tax is imposed on the manufacture or sale of goods and is included in the product price.


Step 1: Direct vs indirect taxes

- Direct taxes (Income tax, Wealth tax) are paid directly to the government by the individual.

- Indirect taxes (Excise tax, VAT, GST) are included in the price and paid indirectly.


Step 2: Explanation

Excise tax increases the cost of goods, and consumers pay it when buying products.


Thus, option (B) is correct.
Quick Tip: Classify taxes to understand who bears the tax burden directly or indirectly.
Excise tax is an indirect tax; income tax is direct.


Question 85:

Who achieved the distinction of being the first person to orbit the Earth?

  • (A) Yuri Gagarin
  • (B) John Glenn
  • (C) Valentina Tereshkova
  • (D) Neil Armstrong
Correct Answer: (A) Yuri Gagarin
View Solution




Yuri Gagarin, a Soviet cosmonaut, became the first human to orbit the Earth on April 12, 1961, marking a significant milestone in space exploration.


Step 1: Significance of the mission

Gagarin's flight demonstrated the possibility of human space travel and intensified the space race during the Cold War.


Step 2: Other astronauts

- John Glenn was the first American to orbit Earth.

- Valentina Tereshkova was the first woman in space.

- Neil Armstrong was the first human to walk on the Moon.


Therefore, option (A) is correct.
Quick Tip: Remember key firsts in space history:
Yuri Gagarin - first human orbit; Neil Armstrong - first Moonwalk.


Question 86:

What was the spinning jenny, one of the earliest innovations in the Industrial Revolution?

  • (A) A device for storing energy
  • (B) A device for making cloth
  • (C) A steam engine
  • (D) A water pump
Correct Answer: (B) A device for making cloth
View Solution




The spinning jenny, invented by James Hargreaves in the 18th century, was a multi-spindle spinning frame that revolutionized the production of yarn, greatly speeding up cloth making.


Step 1: Importance

It allowed one worker to spin multiple threads at once, boosting textile manufacturing efficiency.


Step 2: Industrial Revolution context

This invention was a key driver in the early Industrial Revolution, shifting production from manual to machine-based.


Therefore, option (B) is correct.
Quick Tip: Connect innovations like the spinning jenny to the start of mechanized textile production.
The Industrial Revolution began with advances in cloth-making technology.


Question 87:

What is the term used to describe the process, discovered by Goodyear, of adding sulfur to heated rubber?

  • (A) Oxidation
  • (B) Polymerization
  • (C) Vulcanization
  • (D) Distillation
Correct Answer: (C) Vulcanization
View Solution




Vulcanization is a chemical process discovered by Charles Goodyear in 1839, involving heating rubber with sulfur to improve elasticity, strength, and durability.


Step 1: Problem with raw rubber

Raw rubber is sticky and deforms easily. Vulcanization stabilizes it, making rubber useful for products like tires and footwear.


Step 2: Effect of sulfur

Sulfur forms cross-links between rubber molecules, enhancing mechanical properties.


Hence, option (C) is correct.
Quick Tip: Associate Goodyear with vulcanization and improved rubber products.
Remember vulcanization is essential for practical uses of rubber.


Question 88:

What is the general type of mammal that gives birth to undeveloped young that are kept in pouches?

  • (A) Marsupials
  • (B) Primates
  • (C) Rodents
  • (D) Cetaceans
Correct Answer: (A) Marsupials
View Solution




Marsupials are mammals that give birth to relatively undeveloped offspring, which continue to develop inside a pouch on the mother’s body. Examples include kangaroos, koalas, and opossums.


Step 1: Reproductive strategy

Unlike placental mammals, marsupials have a shorter gestation period and nurture their young in pouches.


Step 2: Contrast with other mammals

- Primates, rodents, and cetaceans give birth to more developed young.


Thus, option (A) is correct.
Quick Tip: Classify mammals by reproductive methods:
Monotremes lay eggs; marsupials have pouches; placentals give birth to developed young.


Question 89:

What is the primary objective of the Artemis III mission planned by NASA?

  • (A) Conduct scientific experiments on the Mars surface.
  • (B) Establish a permanent human presence on Mars.
  • (C) Perform a crewed Moon landing.
  • (D) Collect samples for analysis from the lunar South Pole.
Correct Answer: (C) Perform a crewed Moon landing.
View Solution




The Artemis III mission by NASA aims to return humans to the Moon, marking the first crewed lunar landing since the Apollo missions. The mission is planned to land astronauts on the lunar surface, particularly near the Moon’s South Pole, to conduct exploration and scientific studies.


Step 1: Understand the mission objectives

The primary goal is to perform a crewed Moon landing to establish sustainable exploration capabilities. It is a major milestone for NASA's Artemis program.


Step 2: Clarify other options

- Option (A) relates to Mars, which is a separate planetary exploration goal.

- Option (B) mentions establishing permanent presence on Mars, which is a longer-term future goal, not Artemis III's objective.

- Option (D) partially describes Artemis III activities, but the key point is the crewed landing, so option (C) is more accurate.


Thus, option (C) is the correct answer.
Quick Tip: Track space missions by their main goals:
Artemis missions focus on Moon exploration, while Mars missions are separate.
Understanding mission objectives helps select the best answer.


Question 90:

What is the name of the technology that allows for the creation of three-dimensional objects using a digital model?

  • (A) Neural Networks
  • (B) Additive Manufacturing
  • (C) Swarm Robotics
  • (D) Artificial Intelligence
Correct Answer: (B) Additive Manufacturing
View Solution




Additive Manufacturing, commonly known as 3D printing, is a process where objects are created layer-by-layer from a digital 3D model. This technology allows precise and customizable production of complex shapes and structures.


Step 1: Understanding Additive Manufacturing

Instead of removing material (subtractive methods), additive manufacturing builds up the object, adding material in successive layers.


Step 2: Clarify other options

- Neural Networks are AI systems modeled after the human brain.

- Swarm Robotics involves coordination of multiple robots.

- Artificial Intelligence refers to computer systems performing tasks requiring human intelligence.


Therefore, option (B) is correct as it specifically refers to 3D object creation technology.
Quick Tip: Differentiate between manufacturing and computing technologies.
Additive Manufacturing is key to 3D printing innovations.


Question 91:

What is the name of the force that holds the nucleus of an atom together?

  • (A) Electromagnetic force
  • (B) Gravitational force
  • (C) Strong nuclear force
  • (D) Weak nuclear force
Correct Answer: (C) Strong nuclear force
View Solution




The nucleus of an atom is composed of protons and neutrons. Despite the repulsive electromagnetic forces between positively charged protons, the nucleus stays together due to the strong nuclear force.


Step 1: Understanding forces

- Electromagnetic force causes repulsion between protons.

- Gravitational force is too weak at atomic scales to hold the nucleus.

- Strong nuclear force is a powerful, short-range force that binds protons and neutrons tightly.

- Weak nuclear force is responsible for radioactive decay, not nuclear binding.


Thus, the strong nuclear force is responsible for holding the nucleus together, making option (C) correct.
Quick Tip: Remember the four fundamental forces and their roles:
Strong force binds nucleus; Electromagnetic acts between charges; Gravity is weakest; Weak force causes decay.


Question 92:

The undersea exploration submersible named Titan operated by OceanGate Expeditions went missing in the depth of ____.

  • (A) South Atlantic Ocean
  • (B) South Pacific Ocean
  • (C) North Atlantic Ocean
  • (D) North Pacific Ocean
Correct Answer: (C) North Atlantic Ocean
View Solution




The submersible Titan, operated by OceanGate Expeditions, went missing in the North Atlantic Ocean while exploring the Titanic shipwreck site.


Step 1: Location of incident

The Titanic wreck lies deep in the North Atlantic Ocean near Newfoundland, Canada.


Step 2: Significance of the expedition

OceanGate’s missions aim to explore deep-sea wrecks, but the loss of Titan in the North Atlantic brought global attention to deep-sea exploration risks.


Therefore, option (C) is correct.
Quick Tip: Link oceanic expeditions with their geographic locations for better memory.
Titan submersible incident occurred in the North Atlantic, near Titanic’s wreck.


Question 93:

Which river is at the center of the ongoing dispute between Afghanistan and Iran?

  • (A) Indus River
  • (B) Tigris River
  • (C) Euphrates River
  • (D) Helmand River
Correct Answer: (D) Helmand River
View Solution




The Helmand River, which originates in Afghanistan and flows into Iran, has been the subject of dispute due to water sharing and usage rights between the two countries.


Step 1: Importance of the Helmand River

It is a critical water source for irrigation and drinking in both nations.


Step 2: Nature of dispute

The disagreement centers on dam construction and water diversion by Afghanistan, affecting Iran’s water supply downstream.


Hence, option (D) is the correct answer.
Quick Tip: Remember international rivers and their associated disputes for geography and current affairs.
Helmand River dispute is a key issue between Afghanistan and Iran.


Question 94:

Dick Fosbury was an American athlete who changed the sport of ____ forever with his unconventional technique.

  • (A) Long Jump
  • (B) High Jump
  • (C) Pole Vault
  • (D) Gymnastics
Correct Answer: (B) High Jump
View Solution




Dick Fosbury revolutionized the high jump event by introducing the "Fosbury Flop," a technique where the jumper goes over the bar headfirst and backward, allowing higher clearances.


Step 1: Previous techniques

Before Fosbury, athletes used the straddle or scissors technique, which were less efficient.


Step 2: Impact

Fosbury's method became the standard technique worldwide due to its success and effectiveness.


Therefore, option (B) is correct.
Quick Tip: Connect athletes with their innovations to remember sports history.
Dick Fosbury changed high jump with his backward-flop technique.


Question 95:

Who has been recently selected as the new World Bank President?

  • (A) Ajay Banga
  • (B) David Malpass
  • (C) Indermit Gill
  • (D) Geeta Gopinath
Correct Answer: (A) Ajay Banga
View Solution




Ajay Banga, an Indian-American business executive and former CEO of Mastercard, has been selected as the new President of the World Bank.


Step 1: Role of the World Bank President

The President leads the World Bank's efforts to reduce poverty and support economic development globally.


Step 2: Significance

Banga's appointment is notable as he brings private sector experience and represents a shift in leadership.


Hence, option (A) is correct.
Quick Tip: Keep track of appointments to key international institutions for current affairs.
Ajay Banga is the new World Bank President as of recent announcements.


Question 96:

What is the PM SVANIDHI scheme?

  • (A) A financial assistance program for street vendors
  • (B) A scholarship program for students from economically disadvantaged backgrounds
  • (C) A housing scheme for urban slum dwellers
  • (D) A healthcare initiative for rural areas
Correct Answer: (A) A financial assistance program for street vendors
View Solution




PM SVANIDHI (Prime Minister Street Vendor's AtmaNirbhar Nidhi) is a scheme launched by the Government of India to provide affordable working capital loans to street vendors affected by the COVID-19 pandemic.


Step 1: Purpose

The scheme aims to empower street vendors by providing them with loans at low interest rates to restart their businesses and become self-reliant.


Step 2: Benefits

It also encourages digital transactions and offers rewards for timely repayments, promoting financial inclusion.


Hence, option (A) is the correct answer.
Quick Tip: Connect scheme names with their target beneficiaries for easy recall.
PM SVANIDHI supports street vendors’ financial needs.


Question 97:

In which district of Kerala is the tribal heritage village of En Ooru located?

  • (A) Kozhikode District
  • (B) Thrissur District
  • (C) Wayanad District
  • (D) Kannur District
Correct Answer: (C) Wayanad District
View Solution




En Ooru is a tribal heritage village situated in Wayanad district of Kerala, showcasing the lifestyle, traditions, and culture of indigenous tribal communities.


Step 1: Importance of En Ooru

It promotes tourism and educates visitors about tribal customs and heritage.


Step 2: Location

Wayanad is known for its forested hills and tribal population, making it an apt location for such a heritage village.


Therefore, option (C) is correct.
Quick Tip: Associate cultural heritage sites with their districts to improve geographical knowledge.
En Ooru - tribal heritage village in Wayanad.


Question 98:

Which among these is a cycle racing event?

  • (A) The Super Bowl
  • (B) The Australian Open
  • (C) The Tour de France
  • (D) The Indianapolis 500
Correct Answer: (C) The Tour de France
View Solution




The Tour de France is a world-famous annual cycling race held primarily in France, regarded as one of the most prestigious events in professional cycling.


Step 1: Identify the events

- The Super Bowl is a championship game in American football.

- The Australian Open is a tennis Grand Slam tournament.

- Indianapolis 500 is an automobile race.


Therefore, option (C) is correct.
Quick Tip: Remember famous sports events by their categories:
Tour de France - cycling; Super Bowl - American football; Australian Open - tennis.


Question 99:

Which among this was the first municipal corporation in India?

  • (A) Madras
  • (B) Bombay
  • (C) Delhi
  • (D) Kolkata
Correct Answer: (A) Madras
View Solution




Madras (now Chennai) was the first city in India to establish a municipal corporation, set up in 1688 under British rule, to manage urban governance and civic administration.


Step 1: Historical context

This was part of British efforts to organize and control growing urban centers in India.


Step 2: Other cities

Municipal corporations in Bombay and Kolkata were established later.


Hence, option (A) is correct.
Quick Tip: Link historical urban governance developments to cities:
Madras - first municipal corporation in India.


Question 100:

Who painted the famous painting "The Starry Night"?

  • (A) Pablo Picasso
  • (B) Vincent van Gogh
  • (C) Claude Monet
  • (D) Salvador Dali
Correct Answer: (B) Vincent van Gogh
View Solution




"The Starry Night" is a famous painting by Dutch post-impressionist artist Vincent van Gogh, created in 1889. It depicts a swirling night sky over a quiet village.


Step 1: About the artist

Van Gogh is known for his expressive use of color and brushwork, influencing modern art greatly.


Step 2: Artistic significance

The painting is celebrated for its emotional intensity and unique style, making it one of the most recognized artworks globally.


Thus, option (B) is correct.
Quick Tip: Remember famous paintings and their artists for art history knowledge.
"The Starry Night" is by Vincent van Gogh.


Question 101:

What is the capital of Australia?

  • (A) Sydney
  • (B) Melbourne
  • (C) Canberra
  • (D) Perth
Correct Answer: (C) Canberra
View Solution




Canberra is the capital city of Australia, established as the capital in 1913 as a compromise between the two largest cities, Sydney and Melbourne.


Step 1: Understand the distinction

- Sydney and Melbourne are the largest cities but not the capital.

- Canberra was purpose-built to be the political center, housing government offices and the Parliament.


Hence, option (C) is correct.
Quick Tip: Remember capital cities may differ from the largest or most famous cities.
Canberra is the capital, not Sydney or Melbourne.


Question 102:

Who was the first Indian woman to win an Olympic medal?

  • (A) Karnam Malleswari
  • (B) P.T. Usha
  • (C) Sania Mirza
  • (D) Saina Nehwal
Correct Answer: (A) Karnam Malleswari
View Solution




Karnam Malleswari became the first Indian woman to win an Olympic medal by securing a bronze in weightlifting at the Sydney Olympics in 2000.


Step 1: Recognize contributions

She paved the way for women athletes in India and inspired future generations.


Step 2: Other athletes

- P.T. Usha came close but narrowly missed a medal.

- Sania Mirza and Saina Nehwal are prominent in tennis and badminton respectively but did not win the first Olympic medal.


Therefore, option (A) is correct.
Quick Tip: Remember firsts in Indian sports history to answer such questions confidently.
Karnam Malleswari – first Indian woman Olympic medalist.


Question 103:

Where is the Satish Dhawan Space Center located?

  • (A) Bangalore, Karnataka
  • (B) Balasore, Odisha
  • (C) Sriharikota, Andhra Pradesh
  • (D) Thumpa, Kerala
Correct Answer: (C) Sriharikota, Andhra Pradesh
View Solution




Satish Dhawan Space Center (SDSC) is located in Sriharikota, Andhra Pradesh. It is the main launch center of the Indian Space Research Organisation (ISRO).


Step 1: Function

It is used for launching satellites and space missions into orbit.


Step 2: Geographical advantage

Sriharikota's coastal location near the equator provides ideal conditions for rocket launches.


Hence, option (C) is correct.
Quick Tip: Associate important space centers with their locations.
SDSC is in Sriharikota, ISRO’s primary launch site.


Question 104:

Which of the following countries is NOT a member of the Shanghai Cooperation Organisation (SCO)?

  • (A) India
  • (B) Pakistan
  • (C) Iran
  • (D) Japan
Correct Answer: (D) Japan
View Solution




The Shanghai Cooperation Organisation (SCO) is a Eurasian political, economic, and security alliance. India, Pakistan, and Iran are members, but Japan is not part of the SCO.


Step 1: Member countries

- SCO members include China, Russia, India, Pakistan, Iran, and several Central Asian countries.


Step 2: Japan’s status

Japan is not a member but is a significant player in the Asia-Pacific region.


Therefore, option (D) is correct.
Quick Tip: Learn international organizations and their members carefully.
SCO includes Central and South Asian countries, excluding Japan.


Question 105:

Who recently became the youngest-ever senior world champion at 17 years and secured India's first-ever individual title at the World Archery Championships?

  • (A) Deepika Kumari
  • (B) Aditi Swami
  • (C) Bombayla Devi Laishram
  • (D) Dola Banerjee
Correct Answer: (B) Aditi Swami
View Solution




Aditi Swami became the youngest-ever senior world champion at the age of 17, winning India's first-ever individual title at the World Archery Championships.


Step 1: Achievement significance

Her victory marked a historic moment for Indian archery on the world stage.


Step 2: Other archers

Deepika Kumari and others have been successful but did not claim this particular record.


Hence, option (B) is correct.
Quick Tip: Stay updated with recent sports achievements for current affairs.
Aditi Swami is a rising star in Indian archery.


Question 106:

Find the missing number in the series 2, 6, 14, 30, ?, 126

  • (A) 63
  • (B) 73
  • (C) 62
  • (D) 95
Correct Answer: (C) 62
View Solution




Given the series: 2, 6, 14, 30, ?, 126, we are asked to find the missing number.


Step 1: Observe the pattern

Look at the differences between consecutive terms:

6 - 2 = 4

14 - 6 = 8

30 - 14 = 16


The differences are 4, 8, and 16, which double each time. This suggests the difference follows a pattern of powers of 2 (i.e., \(2^2\), \(2^3\), \(2^4\)).


Step 2: Find the next difference

Following this pattern, the next difference should be \(2^5 = 32\).


Step 3: Calculate the missing term

Add this difference to the last known term:

30 + 32 = 62


Step 4: Verify with the next term

Next difference should be \(2^6 = 64\). Check if:

62 + 64 = 126 \quad \checkmark


Thus, the missing number in the series is 62.
Quick Tip: When analyzing number series, check the difference between terms first.
Look for arithmetic patterns such as constant difference, doubling difference, or multiplication.


Question 107:

John has 12 red pencils and 15 blue pencils. He wants to bundle them into groups of the same size, with no pencils left over. What is the largest number of pencils that can be in each bundle?

  • (A) 1
  • (B) 3
  • (C) 5
  • (D) 6
Correct Answer: (B) 3
View Solution




John has 12 red and 15 blue pencils and wants to divide them into bundles such that each bundle has the same number of pencils with no pencils left over. We need to find the largest bundle size possible.


Step 1: Understand the problem

The problem is essentially about finding the greatest common divisor (GCD) of 12 and 15 because the GCD represents the largest number that divides both numbers exactly.


Step 2: Find factors

Factors of 12: 1, 2, 3, 4, 6, 12

Factors of 15: 1, 3, 5, 15


Step 3: Find common factors

Common factors: 1, 3

Greatest common factor = 3


Step 4: Conclusion

Therefore, the largest number of pencils in each bundle is 3.
Quick Tip: Use the GCD method to find the largest equal grouping without remainder in problems involving division of two quantities.


Question 108:

A shoe seller has 1000 shoes in his shop. He wants to arrange them in a rack in such a way that the number of rows and columns remains the same. What is the minimum number of shoes he needs more for this purpose?

  • (A) 24
  • (B) 32
  • (C) 34
  • (D) 14
Correct Answer: (A) 24
View Solution




The seller wants to arrange the shoes in a rack such that the number of rows equals the number of columns. This means the total number of shoes should be a perfect square number.


Step 1: Identify the perfect squares near 1000

- \(31^2 = 961\) (less than 1000)

- \(32^2 = 1024\) (just above 1000)


Step 2: Calculate additional shoes needed

To have a perfect square, the seller needs to have 1024 shoes.

Additional shoes required = \(1024 - 1000 = 24\)


Step 3: Conclusion

So, the minimum number of shoes he needs more is 24.
Quick Tip: For arranging items in a square, find the closest perfect square number greater than or equal to the total count.
The difference is the number of items needed to make a perfect square arrangement.


Question 109:

Average ages of Ram and Manohar is 15, average ages of Manohar and Shyam is 12 and average ages of Ram and Shyam is 13, then the age of Manohar is?

  • (A) 13
  • (B) 16
  • (C) 14
  • (D) 12
Correct Answer: (C) 14
View Solution




Let the ages of Ram, Manohar, and Shyam be \(R\), \(M\), and \(S\) respectively.


Step 1: Translate average ages into equations

Average age of Ram and Manohar = 15
\(\Rightarrow \frac{R + M}{2} = 15 \implies R + M = 30\)


Average age of Manohar and Shyam = 12
\(\Rightarrow \frac{M + S}{2} = 12 \implies M + S = 24\)


Average age of Ram and Shyam = 13
\(\Rightarrow \frac{R + S}{2} = 13 \implies R + S = 26\)


Step 2: Add all three equations
\((R + M) + (M + S) + (R + S) = 30 + 24 + 26 = 80\)

Simplify:
\(2(R + M + S) = 80 \implies R + M + S = 40\)


Step 3: Use substitution to find \(M\)

From \(R + M = 30\), we get \(R = 30 - M\).

From \(R + S = 26\), substituting for \(R\),
\((30 - M) + S = 26 \implies S = 26 - 30 + M = M - 4\)


Using \(M + S = 24\), substitute \(S\):
\(M + (M - 4) = 24 \implies 2M - 4 = 24\)
\(2M = 28 \implies M = 14\)


Step 4: Conclusion

Manohar's age is 14.
Quick Tip: Use systems of linear equations for solving average age problems.
Sum of pairwise averages helps to find the sum of all individuals.


Question 110:

Arjun and Kiran, friends, receive a bonus of Rs 2000 each in their bank accounts. They already have Rs. 47,000 and Rs. 54,000 in their respective bank accounts. What is the ratio of the amounts in their bank accounts after the bonus?

  • (A) 7:8
  • (B) 7:9
  • (C) 8:9
  • (D) 6:8
Correct Answer: (A) 7:8
View Solution




Step 1: Calculate new amounts after bonus

Arjun's amount after bonus = 47,000 + 2,000 = 49,000

Kiran's amount after bonus = 54,000 + 2,000 = 56,000


Step 2: Find the ratio

Ratio = \(\frac{49,000}{56,000} = \frac{49}{56}\)


Step 3: Simplify the ratio

Both numerator and denominator are divisible by 7:
\(\frac{49 \div 7}{56 \div 7} = \frac{7}{8}\)


Step 4: Conclusion

The ratio of the amounts in their bank accounts after the bonus is 7:8.
Quick Tip: When adding the same amount to two quantities, add first, then find the ratio.
Always simplify ratios to their lowest terms.


Question 111:

In a class of 40 students and 5 teachers, each student got sweets that are 15% of the total number of students and each teacher got sweets that are 20% of the total number of students. How many sweets were there?

  • (A) 240
  • (B) 280
  • (C) 320
  • (D) 360
Correct Answer: (B) 280
View Solution




Given:

- Total students = 40

- Total teachers = 5


Step 1: Calculate sweets per student

Each student got sweets equal to 15% of total students

= 15% of 40 = \(\frac{15}{100} \times 40 = 6\) sweets per student


Step 2: Calculate sweets per teacher

Each teacher got sweets equal to 20% of total students

= 20% of 40 = \(\frac{20}{100} \times 40 = 8\) sweets per teacher


Step 3: Calculate total sweets

Total sweets for students = \(40 \times 6 = 240\)

Total sweets for teachers = \(5 \times 8 = 40\)

Total sweets = 240 + 40 = 280


Hence, the total number of sweets is 280.
Quick Tip: When dealing with percentages of a group, multiply the percentage by the total number to find individual amounts.
Sum the quantities for all groups to get the total.


Question 112:

A shopkeeper sold an article for Rs 3750. If he had charged 24% less, even then he would have earned a profit of 14%. What is the original cost of the article?

  • (A) 2850
  • (B) 2717
  • (C) 3289
  • (D) 2500
Correct Answer: (D) 2500
View Solution




Let the original cost price (CP) of the article be \(x\).


Step 1: Understand the selling price (SP)

Original SP = Rs 3750


If SP was 24% less:

Reduced SP = \(3750 \times (1 - 0.24) = 3750 \times 0.76 = Rs 2850\)


Step 2: Profit condition on reduced SP

Even with reduced SP of Rs 2850, the shopkeeper makes a profit of 14%.

Profit = 14% of CP

So, \(2850 = x + 0.14x = 1.14x\)


Step 3: Calculate original CP
\(x = \frac{2850}{1.14} = 2500\)


Thus, the original cost price is Rs 2500.
Quick Tip: Remember:
SP = CP + Profit
Use percentage formulas carefully to set up equations and solve.


Question 113:

The difference of simple interest from two banks for Rs. 1000 in 2 years is Rs 20. Find the difference in rates of interest.

  • (A) 2%
  • (B) 1%
  • (C) 1.5%
  • (D) 2.5%
Correct Answer: (B) 1%
View Solution




Given:

Principal (P) = Rs 1000

Time (T) = 2 years

Difference in simple interest (SI) = Rs 20


Step 1: Use formula for simple interest

Simple Interest, \(SI = \frac{P \times R \times T}{100}\), where \(R\) is rate of interest.


Let rates of the two banks be \(R_1\) and \(R_2\).

Difference in SI:
\(\frac{1000 \times R_1 \times 2}{100} - \frac{1000 \times R_2 \times 2}{100} = 20\)


Step 2: Simplify equation
\(20(R_1 - R_2) = 20\)
\(R_1 - R_2 = 1\)


Step 3: Conclusion

The difference in rates of interest is 1%.
Quick Tip: In simple interest problems, the difference in interest can be used to find difference in rates easily.
Keep the principal and time same to simplify calculations.


Question 114:

Srishti and Anuj started a business investing amounts of Rs 1,85,000 and Rs 2,25,000 respectively. If Anuj's share in profit earned by them is Rs 9000, then what is the total profit earned by them together?

  • (A) Rs 17400
  • (B) Rs 16400
  • (C) Rs 16800
  • (D) Rs 17800
Correct Answer: (B) Rs 16400
View Solution




Given:

Investment of Srishti = Rs 1,85,000

Investment of Anuj = Rs 2,25,000

Anuj's share of profit = Rs 9000


Step 1: Find profit ratio

Profit is divided in the ratio of investments:
\(Profit ratio = 185000 : 225000 = 37 : 45\) (dividing by 5000)


Step 2: Calculate total profit

Let total profit = \(P\)

Anuj's share = \(\frac{45}{37 + 45} \times P = \frac{45}{82} P\)


Given Anuj's share = Rs 9000, so
\(\frac{45}{82} P = 9000 \implies P = \frac{9000 \times 82}{45} = 16400\)


So, total profit earned is Rs 16,400.
Quick Tip: Profit is divided in the ratio of investments unless otherwise stated.
Calculate the total profit by using one partner's share and their investment ratio.


Question 115:

300 gram of sugar solution has 40% of sugar by weight in it. How much sugar should be added to make it 50% in the solution?

  • (A) 80 g
  • (B) 60 g
  • (C) 120 g
  • (D) 40 g
Correct Answer: (B) 60 g
View Solution




Given:

Weight of solution = 300 g

Sugar percentage = 40%


Step 1: Calculate amount of sugar in original solution

Sugar weight = \(40%\) of 300 = \(\frac{40}{100} \times 300 = 120\) g


Step 2: Let \(x\) be the amount of sugar to be added


After adding \(x\) g of sugar, total sugar = \(120 + x\) g

Total solution weight = \(300 + x\) g


Step 3: Set up equation for 50% sugar concentration
\(\frac{120 + x}{300 + x} = \frac{50}{100} = 0.5\)


Step 4: Solve for \(x\)
\(120 + x = 0.5(300 + x)\)
\(120 + x = 150 + 0.5x\)
\(x - 0.5x = 150 - 120\)
\(0.5x = 30\)
\(x = 60\) g


Therefore, 60 grams of sugar should be added.
Quick Tip: When concentration changes, use the formula:
(New amount of solute) / (New total weight) = New concentration
Solve for the unknown added quantity.


Question 116:

A can do a work in 20 days and B can do it in 10 days. A starts the work and works alone for 5 days. Then B joins A and they finish the work. In how many days total does the work get finished?

  • (A) 10
  • (B) 12
  • (C) 9
  • (D) 8
Correct Answer: (A) 10
View Solution




Step 1: Find daily work rate of A and B

- A can complete work in 20 days, so A's work rate = \(\frac{1}{20}\) work/day

- B can complete work in 10 days, so B's work rate = \(\frac{1}{10}\) work/day


Step 2: Work done by A alone in 5 days

Work done by A in 5 days = \(5 \times \frac{1}{20} = \frac{5}{20} = \frac{1}{4}\)


Step 3: Remaining work

Remaining work = \(1 - \frac{1}{4} = \frac{3}{4}\)


Step 4: Combined daily work rate of A and B

Combined rate = \(\frac{1}{20} + \frac{1}{10} = \frac{1}{20} + \frac{2}{20} = \frac{3}{20}\) work/day


Step 5: Time taken by A and B together to complete remaining work

Time = \(\frac{Work}{Rate} = \frac{3/4}{3/20} = \frac{3}{4} \times \frac{20}{3} = 5\) days


Step 6: Total time

Total time = 5 days (A alone) + 5 days (A and B together) = 10 days


Hence, the work is completed in 10 days.
Quick Tip: Calculate individual work rates, then sum them for combined work rate.
Use remaining work and combined rate to find remaining time.


Question 117:

A truck covers a distance of 368 km at a certain speed in 8 hours. How much time would a car take at an average speed which is 18 km/h more than that of the truck to cover a distance which is 16 km more than that traveled by the truck?

  • (A) 7 hours
  • (B) 5 hours
  • (C) 6 hours
  • (D) 8 hours
Correct Answer: (C) 6 hours
View Solution




Step 1: Calculate the speed of the truck

Speed = \(\frac{Distance}{Time} = \frac{368}{8} = 46\) km/h


Step 2: Calculate the speed of the car

Car's speed = \(46 + 18 = 64\) km/h


Step 3: Calculate distance traveled by the car

Distance = \(368 + 16 = 384\) km


Step 4: Calculate time taken by the car

Time = \(\frac{Distance}{Speed} = \frac{384}{64} = 6\) hours


Therefore, the car will take 6 hours to cover the distance.
Quick Tip: Speed = Distance / Time. Use given speeds and distances to find missing time or speed.
Add or subtract distances or speeds carefully before calculations.


Question 118:

Pipe A when open alone can fill the tank in 20 hours. Pipe B when open alone can fill the tank in 10 hours. When A, B, and C are open together, the tank will be full after 7.5 hours. How many hours will be taken by pipe C when open alone to empty the full tank?

  • (A) 30 hours
  • (B) 45 hours
  • (C) 60 hours
  • (D) 90 hours
Correct Answer: (C) 60 hours
View Solution




Step 1: Calculate individual rates

- Rate of A = \(\frac{1}{20}\) tank/hour

- Rate of B = \(\frac{1}{10}\) tank/hour


Step 2: Calculate combined rate of A, B, and C

Combined rate of A, B, and C = \(\frac{1}{7.5} = \frac{2}{15}\) tank/hour


Step 3: Calculate rate of C

Rate of C = Combined rate - Rate of A - Rate of B

= \(\frac{2}{15} - \frac{1}{20} - \frac{1}{10}\)


Find common denominator (60):
\(\frac{2}{15} = \frac{8}{60}\), \(\frac{1}{20} = \frac{3}{60}\), \(\frac{1}{10} = \frac{6}{60}\)


Rate of C = \(\frac{8}{60} - \frac{3}{60} - \frac{6}{60} = \frac{-1}{60}\) tank/hour


Since rate is negative, pipe C empties the tank at the rate of \(\frac{1}{60}\) tank/hour.


Step 4: Time taken by pipe C to empty the tank

Time = \(\frac{1}{Rate} = 60\) hours


Therefore, pipe C will take 60 hours to empty the tank.
Quick Tip: If a pipe empties, its rate is negative when combined with filling pipes.
Calculate individual rates carefully and subtract to find emptying pipe rate.


Question 119:

Select the letter cluster from among the given options which can replace the question (?) mark: BDF, CFI, DHL, ?

  • (A) EJO
  • (B) CJM
  • (C) EML
  • (D) EMI
Correct Answer: (A) EJO
View Solution




Given series: BDF, CFI, DHL, ?


Step 1: Analyze letters in each cluster

Positions of letters in the alphabet:

B (2), D (4), F (6)

C (3), F (6), I (9)

D (4), H (8), L (12)


Step 2: Observe pattern

- 1st letters: B(2), C(3), D(4) — increasing by 1

- 2nd letters: D(4), F(6), H(8) — increasing by 2

- 3rd letters: F(6), I(9), L(12) — increasing by 3


Step 3: Find next cluster

- 1st letter: D(4) + 1 = E(5)

- 2nd letter: H(8) + 2 = J(10)

- 3rd letter: L(12) + 3 = O(15)


Next cluster is EJO.
Quick Tip: For letter series, check alphabetical positions and differences between letters in each position.
Look for arithmetic progression in letter positions.


Question 120:

Select the option that is related to the third word in the same way as the second word is related to the first word.

Lawyer : Judge :: Actor : ?

  • (A) Crowd
  • (B) Director
  • (C) Stage
  • (D) Audience
Correct Answer: (D) Audience
View Solution




Step 1: Understand relationship

A lawyer is related to a judge as a person practicing law is related to the person who judges in a court.

Similarly, an actor performs for an audience.


Step 2: Analyze options

- Crowd is a general group of people, but audience specifically watches a performance.

- Director is a different role related to a play or film.

- Stage is the place of performance, not a person.

- Audience are the people who watch the actor perform.


Thus, the correct relation is Actor : Audience.
Quick Tip: Identify the logical relationship between pairs before choosing the matching option.
Look for roles or functions directly linked in a context.


Question 121:

In the following question, which one of the following responses would be a meaningful order of the following:

1. Smash

2. Rally

3. Serve

4. Celebrate

5. Score

  • (A) 4, 1, 2, 3, 5
  • (B) 3, 2, 1, 5, 4
  • (C) 2, 3, 1, 4, 5
  • (D) 3, 1, 4, 2, 5
Correct Answer: (B) 3, 2, 1, 5, 4
View Solution




This sequence describes a typical flow of events in racket sports such as badminton or tennis. Let us break down each term and their logical order:


Step 1: Understanding each term

- Serve (3): This is the starting action where the player puts the shuttlecock or ball into play.

- Rally (2): After the serve, players hit the shuttlecock/ball back and forth. This continuous exchange is called a rally.

- Smash (1): A powerful, aggressive shot played to end the rally by winning the point.

- Score (5): Points are scored when one player wins the rally.

- Celebrate (4): Players celebrate their successful points or victories.


Step 2: Logical sequence

The game starts with a serve, followed by a rally. When a player performs a smash, it often ends the rally, resulting in scoring. After scoring, players celebrate.

Thus, the correct order is: Serve (3), Rally (2), Smash (1), Score (5), Celebrate (4).


Step 3: Analyze the options

- Option (B) matches this sequence exactly.

- Other options place these actions in illogical orders.


Therefore, option (B) is the correct meaningful order.
Quick Tip: For sequencing questions, understand the natural flow of events.
Visualize the process or scenario to arrange the terms logically.


Question 122:

From the given alternatives, select the word which cannot be formed using the letters of the given word: TECHNOLOGY

  • (A) Echo
  • (B) Tone
  • (C) Slog
  • (D) Honey
Correct Answer: (C) Slog
View Solution




We need to check if all letters of each option are present in the word "TECHNOLOGY" with sufficient frequency.


Step 1: Letters in TECHNOLOGY

T, E, C, H, N, O, L, O, G, Y


Step 2: Check each option

- Echo: E, C, H, O - All letters present.

- Tone: T, O, N, E - All letters present.

- Slog: S, L, O, G - The letter S is not present in TECHNOLOGY. Hence, "Slog" cannot be formed.

- Honey: H, O, N, E, Y - All letters present.


Therefore, the word "Slog" cannot be formed from the letters of TECHNOLOGY.
Quick Tip: Check carefully each letter in the word and the given options.
Letter frequency matters; if a letter is missing, the word cannot be formed.


Question 123:

A man goes 5 km east, then he turns right and goes 4 km, then he turns left and goes 5 km. Which direction is he facing now?

  • (A) North
  • (B) South
  • (C) East
  • (D) West
Correct Answer: (C) East
View Solution




Let's track the man’s direction step-by-step:


Step 1: Initial movement

He starts facing East and moves 5 km East.


Step 2: First turn

Turning right from East leads to South. He moves 4 km South.


Step 3: Second turn

Turning left from South leads to East again. He moves 5 km East.


Step 4: Final direction

After these moves, the man is facing East.
Quick Tip: Remember the directions and their right/left turns:
- Right turn from East → South
- Left turn from South → East
Draw simple diagrams to visualize.


Question 124:

In a gathering seven members are sitting in a row. C is sitting to the left of B but on the right to D, A is sitting right of B, F is sitting right of E but left to D. H is sitting left to E. Find the person sitting in the middle.

  • (A) C
  • (B) D
  • (C) E
  • (D) F
Correct Answer: (B) D
View Solution




Let’s deduce the seating arrangement step by step:


Step 1: From C, B, and D

- C is to the left of B → C is somewhere left of B.

- C is to the right of D → D is left of C.

So order: D - C - B


Step 2: From A and B

- A is right of B → A sits to the right of B.

So order extends to: D - C - B - A


Step 3: From F, E, and D

- F is right of E but left of D → E - F - D


Step 4: From H and E

- H is left of E → H - E - F - D


Step 5: Combine both sequences

From above: H - E - F - D - C - B - A


Step 6: Find the middle person

There are 7 members, so the 4th person in the sequence is in the middle.

Counting: 1-H, 2-E, 3-F, 4-D, 5-C, 6-B, 7-A


Therefore, D is sitting in the middle.
Quick Tip: Visualize seating with given positional clues; write them down stepwise.
Identify the middle seat by counting total members and their order.


Question 125:

In the following question below are given some statements followed by some conclusions. Taking the given statements to be true even if they seem to be at variance from commonly known facts, read all the conclusions and then decide which of the given conclusion logically follows the given statements?

Statements:

1. No mirrors are glass

2. No reflective are mirrors


Conclusions:

I. All glass are reflective

II. All reflective are glass

  • (A) Only conclusion I follows
  • (B) Only conclusion II follows
  • (C) Neither conclusion I nor II follows
  • (D) Both conclusions follow
Correct Answer: (C) Neither conclusion I nor II follows
View Solution




Step 1: Analyze the statements

- Statement 1 says no mirrors are glass, meaning mirrors and glass are completely separate sets.

- Statement 2 says no reflective objects are mirrors, meaning reflective objects and mirrors do not overlap.


Step 2: Check conclusions

- Conclusion I: "All glass are reflective"

This does not necessarily follow because the statements do not state anything about all glass being reflective.

- Conclusion II: "All reflective are glass"

This also does not follow because the statements do not provide information about the entire set of reflective objects.


Step 3: Logical reasoning

The statements only specify exclusions (no mirrors are glass, no reflective are mirrors) but do not affirm inclusions needed for either conclusion.


Hence, neither conclusion I nor II logically follows.
Quick Tip: In syllogisms, only accept conclusions directly supported by the statements.
Be cautious about assuming inclusion from exclusion statements.


Question 126:

In the following question below are given some statements followed by some conclusions. Taking the given statements to be true even if they seem to be at variance from commonly known facts, read all the conclusions and then decide which of the given conclusion logically follows the given statements?

Statements:

1. All apples are oranges

2. Some oranges are papayas


Conclusions:

I. Some apples are papayas

II. Some papayas are oranges

  • (A) Only conclusion I follows
  • (B) Only conclusion II follows
  • (C) Neither conclusion I nor II follows
  • (D) Both conclusions follow
Correct Answer: (B) Only conclusion II follows
View Solution




Step 1: Analyze statements

- Statement 1 says all apples are oranges, so the set of apples is completely inside the set of oranges.

- Statement 2 says some oranges are papayas, meaning some members of the orange set overlap with papayas.


Step 2: Analyze conclusions

- Conclusion I: Some apples are papayas

This conclusion does not necessarily follow. Even though apples are inside oranges, it is not guaranteed that apples overlap with papayas. The "some oranges are papayas" might not include apples.

- Conclusion II: Some papayas are oranges

This conclusion logically follows because statement 2 explicitly says some oranges are papayas. The overlap exists.


Therefore, only conclusion II logically follows.
Quick Tip: Visualize sets using Venn diagrams to check overlaps.
A subset does not automatically overlap with another subset unless stated.


Question 127:

Read the given statements and conclusions carefully and decide which of the conclusions logically follow(s) from the statements.

Statements:

1. AIDS is a killer disease

2. It is easier to prevent AIDS than to treat it


Conclusions:

I. AIDS prevention is very expensive

II. People will not cooperate for AIDS prevention

  • (A) Only conclusion I follows
  • (B) Only conclusion II follows
  • (C) Neither conclusion I nor II follows
  • (D) Both conclusions follow
Correct Answer: (C) Neither conclusion I nor II follows
View Solution




Step 1: Understand the statements

- AIDS is a killer disease — factual statement.

- It is easier to prevent AIDS than to treat it — emphasizes prevention is preferable.


Step 2: Analyze conclusions

- Conclusion I: AIDS prevention is very expensive

The statements say nothing about the cost of prevention, so this conclusion is unsupported.

- Conclusion II: People will not cooperate for AIDS prevention

The statements do not indicate anything about people's cooperation or behavior.

Thus, this conclusion is also unsupported.


Step 3: Logical outcome

Neither conclusion is directly supported by the statements given.


Therefore, neither conclusion I nor II logically follows.
Quick Tip: Focus on what is explicitly stated in statements.
Do not infer assumptions about cost or behavior without evidence.


Question 128:

If January 1 is a Friday, what is the first day of the month of March in a leap year?

  • (A) Wednesday
  • (B) Thursday
  • (C) Tuesday
  • (D) Friday
Correct Answer: (C) Tuesday
View Solution




Step 1: Days in January and February in a leap year

- January has 31 days

- February has 29 days (leap year)


Step 2: Calculate total days passed before March 1

Total days = 31 (Jan) + 29 (Feb) = 60 days


Step 3: Calculate day of March 1

- Days passed modulo 7: \(60 \mod 7 = 4\) days

- Since Jan 1 is Friday, add 4 days: Friday → Saturday (1), Sunday (2), Monday (3), Tuesday (4)


Therefore, March 1 falls on Tuesday.
Quick Tip: Remember leap year February has 29 days.
Calculate days passed and use modulo 7 to find the weekday.


Question 129:

By looking in a mirror it appears that it is 6:30 on the clock. What is the real time?

  • (A) 6:30
  • (B) 6:40
  • (C) 5:00
  • (D) 5:30
Correct Answer: (D) 5:30
View Solution




Step 1: Understand mirror image time

The mirror image of a clock reverses the time. To find real time from mirror time, use the formula:

\[ Real time = 12:00 - Mirror time \]

Step 2: Calculate real time

- Mirror time = 6:30

- Subtract from 12:00 → 12:00 - 6:30 = 5:30


Therefore, the real time is 5:30.
Quick Tip: To find real time from mirror image, subtract mirror time from 12:00.
Be careful with hour and minute subtraction.


Question 130:

Showing a man, a woman said: ``His brother's father is the only son of my grandfather". How is the woman related to the man?

  • (A) Aunt
  • (B) Sister
  • (C) Daughter
  • (D) Mother
Correct Answer: (B) Sister
View Solution




Step 1: Analyze the statement

- ``His brother's father" means the man's father (because his brother and he share the same father).

- ``The only son of my grandfather" means the speaker's father (since the only son of one's grandfather is the father).


So, the man's father is the speaker's father.


Step 2: Relationship deduction

If the man and the woman share the same father, they are siblings.


Therefore, the woman is the sister of the man.
Quick Tip: Break down family relation sentences carefully step-by-step.
Identify common ancestors to find relationships.


Question 131:

Which factor does a country prioritize when granting Most Favored Nation (MFN) status to another country?

  • (A) Increasing trade with that country
  • (B) Strengthening military relations
  • (C) Enhancing cultural relations
  • (D) Facilitating people-to-people contact
Correct Answer: (A) Increasing trade with that country
View Solution




Step 1: Understand MFN status

The Most Favored Nation (MFN) status is a principle in international trade agreements. It means that the country granting this status will provide the other country with trade advantages, such as reduced tariffs or fewer trade barriers, equal to those given to its most favored trading partner.


Step 2: Prioritization factor

The primary aim of MFN status is to promote trade relations between countries by ensuring non-discriminatory trade policies. This helps increase trade volumes and economic cooperation.

Military relations, cultural relations, or people-to-people contact are not the direct reasons for granting MFN status.


Step 3: Conclusion

Therefore, the main factor prioritized is increasing trade with that country.
Quick Tip: Remember MFN is mainly about trade advantages and economic cooperation.
It ensures equal trade treatment to all favored countries.


Question 132:

What does the principle of territorial integrity in International Law signify?

  • (A) The security and inviolability of a state's boundaries
  • (B) A state's interference in the internal affairs of other states
  • (C) A state's non-interference in the affairs of other states
  • (D) None of the above options
Correct Answer: (A) The security and inviolability of a state's boundaries
View Solution




Step 1: Definition of territorial integrity

Territorial integrity is a fundamental principle in international law that respects the sovereignty of a state over its geographical territory. It prohibits external aggression, invasion, or any unauthorized intrusion.


Step 2: Meaning

It means the security and inviolability of a state's boundaries must be maintained. No other state has the right to violate or alter them forcibly.


Step 3: Explanation of other options

- Option B suggests interference, which is contrary to sovereignty.

- Option C talks about non-interference but is different from territorial integrity which focuses on boundaries specifically.

- Option D is invalid.


Therefore, option A is the correct answer.
Quick Tip: Territorial integrity ensures the borders of a country are respected and protected under international law.


Question 133:

Principle of "Jus Soli" is associated with?

  • (A) Immigration
  • (B) Extradition
  • (C) Asylum
  • (D) Citizenship
Correct Answer: (D) Citizenship
View Solution




Step 1: Understand "Jus Soli"

"Jus Soli" is a Latin term meaning "right of the soil". It is a principle of nationality law by which citizenship is determined by the place of birth. If a person is born in a country, they automatically acquire citizenship of that country.


Step 2: Clarify other options

- Immigration refers to moving to another country.

- Extradition is the process of handing over a person accused or convicted of a crime to another jurisdiction.

- Asylum is protection granted to refugees.


Hence, "Jus Soli" is associated with citizenship.
Quick Tip: Remember "Jus Soli" deals with birthright citizenship based on location of birth.


Question 134:

What does the phrase ``pacta sunt servanda" imply?

  • (A) Treaties must be respected and upheld
  • (B) Treaties serve as a foundation for International Law
  • (C) Treaties are legally binding agreements
  • (D) Treaties possess validity within the realm of International Law
Correct Answer: (A) Treaties must be respected and upheld
View Solution




Step 1: Meaning of the phrase

"Pacta sunt servanda" is a Latin phrase meaning "agreements must be kept". It is a fundamental principle of international law that all treaties and agreements between states are binding and must be honored in good faith.


Step 2: Explanation of options

- Option A accurately captures the essence of the principle.

- Options B, C, and D describe related ideas but do not capture the core imperative of respecting treaties.


Therefore, option A is correct.
Quick Tip: This principle ensures reliability and trust in international agreements.
"Agreements must be kept" is the literal and legal essence.


Question 135:

Which crimes were specifically addressed by the Hague Convention of 1970?

  • (A) Genocide and crimes against humanity
  • (B) Cyber crimes and digital security
  • (C) Illicit trafficking of cultural property
  • (D) Hijacking of Aircrafts
Correct Answer: (C) Illicit trafficking of cultural property
View Solution




Step 1: Understand the Hague Convention 1970

The Hague Convention of 1970 is an international treaty aimed at preventing the illicit import, export, and transfer of ownership of cultural property. It focuses on protecting cultural heritage during conflicts and peacetime.


Step 2: Explanation of other options

- Genocide and crimes against humanity are addressed under other treaties like the Rome Statute.

- Cyber crimes and digital security are not covered in this convention.

- Hijacking of aircrafts is covered under different international protocols.


Therefore, option C is the correct answer.
Quick Tip: The Hague Convention 1970 protects cultural heritage from illegal trade and theft.
It is crucial for preserving global cultural identity.


Question 136:

Immanuel Kant was an exponent of which of the following theories of punishment?

  • (A) Retributive Theory
  • (B) Preventive Theory
  • (C) Deterrent Theory
  • (D) Reformatory Theory
Correct Answer: (A) Retributive Theory
View Solution




Step 1: Understanding Retributive Theory

- Retributive theory of punishment is based on the idea that offenders deserve punishment because they have committed a wrongdoing.

- It focuses on justice and moral blameworthiness rather than on deterrence or rehabilitation.


Step 2: Immanuel Kant's contribution

- Kant advocated that punishment is justified only if the offender deserves it and must be proportionate to the crime.

- His philosophy supports the principle of "just deserts" which underpins retributive justice.


Step 3: Other theories

- Preventive theory focuses on preventing crime.

- Deterrent theory aims to discourage crime by fear of punishment.

- Reformatory theory emphasizes reforming the offender.


Therefore, Kant is most associated with the Retributive Theory.
Quick Tip: Retributive Theory is about punishing because one deserves it, not just to prevent or reform.
Kant's philosophy emphasizes justice and moral responsibility.


Question 137:

The term "Ultra vires" means?

  • (A) Inside the powers
  • (B) Judicial powers
  • (C) Within fundamental rights
  • (D) Outside the powers
Correct Answer: (D) Outside the powers
View Solution




Step 1: Definition of Ultra Vires

- "Ultra vires" is a Latin term meaning "beyond the powers".

- It refers to acts or decisions taken by a government body, corporation, or official that exceed the scope of power granted by law or corporate charter.


Step 2: Legal implication

- Such acts are considered null and void because they are beyond the legal authority of the actor.


Step 3: Contrast with intra vires

- "Intra vires" means within the powers, which is opposite to ultra vires.


Therefore, the correct meaning is outside the powers.
Quick Tip: Remember "Ultra vires" = beyond legal authority, "Intra vires" = within legal authority.
Acts beyond powers are invalid legally.


Question 138:

Which term describes a matter that is pending before a court?

  • (A) Res judicata
  • (B) Estoppel
  • (C) Inquest
  • (D) Res subjudice
Correct Answer: (D) Res subjudice
View Solution




Step 1: Define terms

- Res judicata: A matter that has been finally decided by a competent court and cannot be re-litigated.

- Estoppel: A legal principle preventing someone from arguing something contrary to a previous claim or conduct.

- Inquest: A judicial inquiry to ascertain the cause of death.

- Res subjudice: A matter currently pending and under consideration before a court.


Step 2: Apply to question

The term that specifically describes a case that is pending before the court is Res subjudice.
Quick Tip: Res judicata means settled matter, res subjudice means pending matter in court.
Differentiate between settled and pending cases.


Question 139:

Concept of 'Dialectical Materialism' is associated with:

  • (A) Liberalism
  • (B) Utilitarianism
  • (C) Marxism
  • (D) Post Modernism
Correct Answer: (C) Marxism
View Solution




Step 1: Understanding Dialectical Materialism

Dialectical Materialism is a philosophical framework developed by Karl Marx and Friedrich Engels. It explains the material basis of reality and history through the dialectic process, emphasizing conflict and change.


Step 2: Association with Marxism

Marxism uses dialectical materialism as its core methodology to analyze social and economic systems, focusing on class struggle and the material conditions of society.


Step 3: Other options

- Liberalism is centered on individual freedoms and rights.

- Utilitarianism focuses on maximizing happiness or utility.

- Post Modernism critiques grand narratives and objective truths.


Hence, dialectical materialism is specifically associated with Marxism.
Quick Tip: Dialectical Materialism is the Marxist way of explaining historical change through material conditions and contradictions.


Question 140:

A legal principle that prevents the same case or issue from being re-litigated between the same parties once a final judgment has been reached is called:

  • (A) Estoppel
  • (B) Res judicata
  • (C) Precedent
  • (D) Preemptory Plea
Correct Answer: (B) Res judicata
View Solution




Step 1: Definition of Res judicata

Res judicata means "a matter judged." It is a legal doctrine preventing parties from re-litigating the same claim or issue once it has been finally decided by a competent court.


Step 2: Purpose

It ensures finality in legal proceedings, avoids multiple lawsuits over the same matter, and conserves judicial resources.


Step 3: Explanation of other options

- Estoppel prevents a party from contradicting something previously established.

- Precedent refers to earlier court decisions used as guidance.

- Preemptory plea is a plea used to avoid trial without discussing merits.


Thus, Res judicata is the correct answer.
Quick Tip: Remember: Res judicata = case closed forever; Estoppel = prevents contradictory claims.


Question 141:

What is a person called who takes legal proceedings against the accused on behalf of the state?

  • (A) Plaintiff
  • (B) Prosecutor
  • (C) Lawyer
  • (D) Appellant
Correct Answer: (B) Prosecutor
View Solution




Step 1: Role of Prosecutor

A prosecutor is a legal representative of the government who brings charges and conducts prosecution against the accused in criminal cases.


Step 2: Explanation of other terms

- Plaintiff is the person who initiates a lawsuit in civil cases.

- Lawyer is a general term for legal professionals.

- Appellant is someone who appeals a court decision.


Thus, the person who takes legal action on behalf of the state in criminal cases is the Prosecutor.
Quick Tip: Prosecutor = government lawyer in criminal cases; Plaintiff = party initiating civil lawsuit.


Question 142:

Joint session of the parliament is presided by

  • (A) The Vice President
  • (B) The President
  • (C) The Speaker of Lok Sabha
  • (D) The Prime Minister
Correct Answer: (C) The Speaker of Lok Sabha
View Solution




Step 1: Understand Parliamentary Joint Session

A joint session of the Parliament of India is convened when there is a deadlock between the Lok Sabha and Rajya Sabha on a bill, particularly a Money Bill or other legislative matters.


Step 2: Presiding Officer

The Speaker of Lok Sabha presides over the joint session.


Step 3: Role of others

- The Vice President is the ex-officio Chairperson of Rajya Sabha but does not preside over joint sessions.

- The President and Prime Minister do not preside over parliamentary sessions.


Therefore, the correct answer is The Speaker of Lok Sabha.
Quick Tip: Remember, Speaker of Lok Sabha presides joint sessions to resolve deadlocks between houses.


Question 143:

Supreme Court of India can decide disputes between States and the Centre under its

  • (A) Original jurisdiction
  • (B) Appellate jurisdiction
  • (C) Advisory Jurisdiction
  • (D) Epistolary Jurisdiction
Correct Answer: (A) Original jurisdiction
View Solution




Step 1: Types of jurisdiction of Supreme Court

- Original jurisdiction: The power to hear cases directly without them coming from lower courts.

- Appellate jurisdiction: The power to hear appeals from lower courts.

- Advisory jurisdiction: The power to give advice on legal questions referred by the President.

- Epistolary jurisdiction: Not a recognized term in Indian judicial system (likely a distractor).


Step 2: Disputes between Centre and States

The Constitution of India grants the Supreme Court original jurisdiction over disputes between the Centre and States or between two or more States.


Therefore, the correct answer is Original jurisdiction.
Quick Tip: Supreme Court's original jurisdiction includes inter-state and Centre-State disputes.


Question 144:

Who administers oath to Chief Justice of the Supreme Court?

  • (A) Speaker of Lok Sabha
  • (B) Prime Minister of India
  • (C) President of India
  • (D) Previous Chief Justice
Correct Answer: (C) President of India
View Solution




Step 1: Constitutional provision

According to Article 124(2) of the Indian Constitution, the President of India administers the oath or affirmation to the Chief Justice of India.


Step 2: Importance of oath

The oath is a solemn promise to uphold the Constitution and discharge duties impartially.


Step 3: Explanation of other options

- Speaker of Lok Sabha and Prime Minister have no role in this ceremony.

- Previous Chief Justice may administer oath to other judges but not the Chief Justice himself.


Hence, the President administers the oath to the Chief Justice.
Quick Tip: Remember, the highest constitutional authority, the President, administers oath to Chief Justice of India.


Question 145:

Fundamental rights are dealt in

  • (A) Part III of the Constitution
  • (B) Part IV of the Constitution
  • (C) Part V of the Constitution
  • (D) Part VI of the Constitution
Correct Answer: (A) Part III of the Constitution
View Solution




Step 1: Constitutional Parts related to Rights

- Part III of the Indian Constitution deals with Fundamental Rights, which guarantee civil liberties to individuals to protect them from arbitrary state action.

- Part IV deals with Directive Principles of State Policy (non-justiciable guidelines).

- Part V and VI deal with Union and State Governments respectively.


Step 2: Importance

Fundamental Rights are essential for safeguarding democracy and individual freedoms in India.


Therefore, the correct answer is Part III of the Constitution.
Quick Tip: Remember: Fundamental Rights are in Part III; Directive Principles in Part IV of Indian Constitution.


Question 146:

Article 343 of the Indian Constitution deals with:

  • (A) Official language of the Union
  • (B) Protection of linguistic minorities
  • (C) National Language of the Union
  • (D) Administrative Language of the Union
Correct Answer: (A) Official language of the Union
View Solution




To determine what Article 343 of the Indian Constitution deals with, we need to examine the actual content of this Article.


Step 1: Refer to Article 343

Article 343 states that the official language of the Union shall be Hindi in the Devanagari script. It also mentions that for a period of 15 years from the commencement of the Constitution (i.e., from 1950), English may also be used for official purposes of the Union.


Step 2: Understanding the intent of the Article

The objective of Article 343 is to define the language to be used by the Union Government for official communication and administrative purposes. This Article also led to the enactment of the Official Languages Act, 1963, which extended the use of English indefinitely along with Hindi.


Step 3: Evaluate the options

- Option A: Official language of the Union – This matches exactly with the purpose of Article 343.

- Option B: Protection of linguistic minorities – This is covered under Article 350A and Article 350B.

- Option C: National Language of the Union – The Constitution does not declare any national language.

- Option D: Administrative Language of the Union – While administrative usage is related, the article explicitly uses the term “official language.”


Thus, the correct answer is Option A. Quick Tip: To remember language-related provisions in the Constitution:
- Article 343: Official language of the Union (Hindi in Devanagari script)
- Article 344: Commission and Committee of Parliament on Official Language
- Article 350A/B: Special provisions for linguistic minorities
- No language is declared as the “national language” in the Constitution of India.


Question 147:

From which country’s constitution was the Directive Principles of State Policy in India based on?

  • (A) Britain
  • (B) Germany
  • (C) France
  • (D) Ireland
Correct Answer: (D) Ireland
View Solution




The Directive Principles of State Policy (DPSPs) are guidelines or principles laid down in Part IV of the Indian Constitution to be followed by the government while framing laws and policies.


Step 1: Source of DPSPs

The idea of DPSPs was borrowed from the Irish Constitution. Ireland included similar provisions inspired by the Spanish Constitution, but it is specifically the Irish version that served as a model for India.


Step 2: Purpose of DPSPs

These principles aim to create social and economic democracy and serve as a moral compass for the governance of the country.


Step 3: Evaluation of options

- Option A (Britain): Source of parliamentary system and rule of law, not DPSPs.

- Option B (Germany): Source for emergency provisions.

- Option C (France): Source of ideals like liberty, equality, fraternity.

- Option D (Ireland): Correct source of DPSPs.
Quick Tip: Remember important sources of Indian Constitution:
- Parliamentary System – Britain
- Fundamental Rights – USA
- Directive Principles of State Policy – Ireland
- Emergency Provisions – Germany
- Liberty, Equality, Fraternity – France


Question 148:

How is the Vice-President of India elected?

  • (A) By direct popular vote
  • (B) By members of Lok Sabha only
  • (C) By members of Lok Sabha and Rajya Sabha
  • (D) By members of Lok Sabha, Rajya Sabha and State Assemblies
Correct Answer: (C) By members of Lok Sabha and Rajya Sabha
View Solution




Step 1: Constitutional Provision

Article 66 of the Indian Constitution deals with the election of the Vice-President.


Step 2: Electoral College

The Vice-President is elected by an electoral college consisting of the members of both Houses of Parliament (Lok Sabha and Rajya Sabha), using the system of proportional representation by means of a single transferable vote.


Step 3: Evaluation of options

- Option A: Incorrect. There is no direct election by the people.

- Option B: Incorrect. It excludes Rajya Sabha.

- Option C: Correct. The Vice-President is elected by both Lok Sabha and Rajya Sabha members.

- Option D: Incorrect. State Assemblies are not involved in this election.
Quick Tip: Differentiate between elections:
- President: Elected by MPs and MLAs
- Vice-President: Elected only by MPs (Lok Sabha + Rajya Sabha)
- Prime Minister: Leader of majority in Lok Sabha


Question 149:

Which of the following is not within the jurisdiction and purpose of the International Criminal Court (ICC)?

  • (A) Prosecuting Crimes against Humanity
  • (B) Prosecuting Genocide
  • (C) Prosecuting War Crimes
  • (D) Prosecuting International Corporate Frauds
Correct Answer: (D) Prosecuting International Corporate Frauds
View Solution




The International Criminal Court (ICC), established under the Rome Statute in 2002, has the jurisdiction to try individuals for the most serious crimes of concern to the international community.


Step 1: ICC’s Jurisdiction Includes:

- Genocide

- Crimes against humanity

- War crimes

- Crime of aggression


Step 2: Evaluation of Options

- Option A: Correct jurisdiction.

- Option B: Genocide is one of the four core crimes under ICC.

- Option C: War crimes are clearly under ICC's purview.

- Option D: Corporate fraud is a financial/economic crime, not covered by the ICC.
Quick Tip: ICC deals with serious international crimes affecting humanity:
- Genocide, war crimes, crimes against humanity, crime of aggression.
- It does not handle economic crimes like money laundering or fraud.


Question 150:

Which of the following options is NOT considered an intellectual property?

  • (A) Trademark
  • (B) Patent
  • (C) Real Estate
  • (D) Geographical indications
Correct Answer: (C) Real Estate
View Solution




Step 1: Understand Intellectual Property (IP)

Intellectual property refers to creations of the mind, such as inventions, literary and artistic works, symbols, names, and images used in commerce.


Step 2: Evaluate each option

- Trademark: A type of intellectual property consisting of recognizable signs or expressions identifying products or services.

- Patent: A form of IP that gives inventors exclusive rights to their inventions.

- Geographical indications: IP rights that identify goods as originating from a specific place with qualities or reputation due to that location.

- Real Estate: Refers to physical property or land, which is not an intellectual creation, hence not intellectual property.


Therefore, Real Estate is not considered intellectual property. Quick Tip: IP includes intangible assets like inventions, designs, symbols, and names. Tangible assets like land and buildings (real estate) are excluded from IP classification.


Question 151:

The Latin phrase ``Ignorantia juris non excusat" means:

  • (A) Ignorance of the law is a valid excuse.
  • (B) Ignorance of the law is not a valid excuse.
  • (C) Ignorance of the law can be considered as a partial excuse.
  • (D) Ignorance of the law is only applicable in certain circumstances.
Correct Answer: (B) Ignorance of the law is not a valid excuse.
View Solution




Step 1: Translation

The Latin phrase ``Ignorantia juris non excusat" translates to "ignorance of the law is no excuse."


Step 2: Legal Principle

This principle asserts that a person who is unaware of a law may not escape liability for violating it simply because they did not know it existed.


Step 3: Apply to Options

- Option A: Opposite of the principle; hence incorrect.

- Option B: Matches the exact meaning.

- Option C \& D: Introduce exceptions which are not part of this absolute legal maxim.
Quick Tip: Latin legal phrases often express foundational legal doctrines. “Ignorantia juris non excusat” is a strict rule used globally to uphold legal responsibility.


Question 152:

Which theory works with the dictums ``you cannot cure by killing" and ``Crime is like a disease"?

  • (A) Deterrence Theory
  • (B) Retributive Theory
  • (C) Reformative Theory
  • (D) Compensation Theory
Correct Answer: (C) Reformative Theory
View Solution




Step 1: Understand the quote

Phrases like ``you cannot cure by killing” and “crime is like a disease” reflect a humane and reform-based approach to criminal justice.


Step 2: Link to theory

- Reformative Theory considers criminals as sick individuals who can be reformed through rehabilitation, education, and moral training rather than punishment.

- Deterrence Theory seeks to prevent crime through fear of punishment.

- Retributive Theory emphasizes punishment as a form of vengeance or justice.

- Compensation Theory focuses on compensating victims.


Reformative Theory best matches the humane, curative approach stated in the question. Quick Tip: Reformative theory treats crime as a symptom of deeper issues (like social or psychological problems) and aims to rehabilitate the offender rather than punish harshly.


Question 153:

Which of the following means that the prosecution has to prove the charge against the accused in criminal cases?

  • (A) Judicial review
  • (B) The margin of appreciation
  • (C) Burden of proof
  • (D) Devolution
Correct Answer: (C) Burden of proof
View Solution




In criminal law, the "burden of proof" is the duty placed on the prosecution to establish the accused's guilt beyond a reasonable doubt.


Step 1: Understand the term

- Burden of proof refers to the obligation to present evidence to support one's claim.

- In criminal trials, this burden lies on the prosecution.


Step 2: Evaluate the options

- Judicial review: Refers to court’s authority to review laws or actions.

- Margin of appreciation: A doctrine in international law.

- Devolution: Transfer of powers from central to local government.


Only "burden of proof" fits the definition. Quick Tip: In criminal trials, always remember: the prosecution must prove the case beyond reasonable doubt. This principle protects the presumption of innocence.


Question 154:

The company’s nationality is decided by its

  • (A) Residence of majority shareholders
  • (B) Registered office
  • (C) Place at books of accounts are kept
  • (D) None of the above
Correct Answer: (B) Registered office
View Solution




Step 1: Legal Principle

A company is an artificial legal person, and its nationality is usually determined by the location of its registered office.


Step 2: Evaluate options

- Residence of shareholders: Not a legal basis for company nationality.

- Place where books are kept: Related to accounting, not nationality.

- Registered office: Legally recognized address of the company—used for jurisdiction and legal identification.


Hence, the correct answer is Option B. Quick Tip: The registered office is the legal seat of a company and plays a vital role in determining its jurisdiction and nationality.


Question 155:

Which of the following statements accurately reflects the provision mentioned in the Constitution of India?

  • (A) The Supreme Court shall have a maximum of 34 judges, including the Chief Justice.
  • (B) The Supreme Court shall have a minimum of 34 judges, including the Chief Justice.
  • (C) The Supreme Court shall have an unlimited number of judges
  • (D) The Constitution of India does not specify the maximum number of judges in the Supreme Court
Correct Answer: (A) The Supreme Court shall have a maximum of 34 judges, including the Chief Justice.
View Solution




Step 1: Constitutional Provision

Article 124 of the Indian Constitution gives Parliament the power to regulate the number of judges.


Step 2: Current legal provision

The Supreme Court (Number of Judges) Amendment Act, 2019 sets the maximum strength at 34 judges (including the Chief Justice).


Step 3: Analyze options

- Option A: Matches the current statute.

- Option B: Incorrect—no minimum stated.

- Option C: Incorrect—limited by law.

- Option D: Incorrect—statutory limit exists.
Quick Tip: The Parliament decides the strength of the Supreme Court through legislation—not directly by the Constitution text itself.


Question 156:

Which of the following rights is not a fundamental right?

  • (A) Right to Freedom
  • (B) Right to Property
  • (C) Right to Religion
  • (D) Right to Equality
Correct Answer: (B) Right to Property
View Solution




Step 1: Historical context

Originally, the Right to Property was a fundamental right under Article 31.


Step 2: Constitutional Amendment

The 44th Constitutional Amendment Act, 1978 removed it from Part III (Fundamental Rights) and inserted Article 300A under Part XII (Legal Rights).


Step 3: Evaluate remaining rights

- Rights to freedom, religion, and equality are part of Part III and remain fundamental rights.
Quick Tip: Remember: Right to Property is now a legal right, not a fundamental right. Fundamental rights can be directly enforced in court; legal rights cannot.


Question 157:

What does the term ``adjudication" mean in a legal context?

  • (A) The process of initiating a lawsuit.
  • (B) The act of settling a legal dispute outside of court.
  • (C) The process of settling a legal issue through the justice system.
  • (D) The legal document that initiates a criminal prosecution.
Correct Answer: (C) The process of settling a legal issue through the justice system.
View Solution




Adjudication is a formal legal process where a judge reviews evidence and arguments to come to a decision or judgment.


Step 1: Evaluate options

- Option A: Refers to filing a case, not adjudication.

- Option B: Describes alternative dispute resolution, not court-based adjudication.

- Option C: Correct definition of adjudication.

- Option D: Refers to "charge sheet" or "complaint," not adjudication.
Quick Tip: Adjudication = Legal judgment by a court. It involves reviewing evidence and law to settle a dispute.


Question 158:

What does the term "de jure" mean?

  • (A) Exceptions to be made in specific cases.
  • (B) Act of formally presenting evidence in a court of law.
  • (C) Thing that is established according to the law
  • (D) Act of appointing a legal representative to handle a case.
Correct Answer: (C) Thing that is established according to the law
View Solution




"De jure" is a Latin term that means "by law" or "legally recognized," as opposed to "de facto," which means "in fact or in practice."


Step 1: Definition

"De jure" refers to practices that are legally recognized, regardless of whether the practice exists in reality.


Step 2: Evaluate the options

- Option A: Incorrect meaning.

- Option B: Describes "testimony" or "evidence presentation."

- Option C: Correct – de jure means "according to law."

- Option D: Refers to appointment of legal representative, not relevant.
Quick Tip: Remember the distinction:
- "De jure" = By law
- "De facto" = In practice, even if not legally recognized


Question 159:

Assertion (A): Fundamental Rights in the Indian Constitution are not absolute.

Reason (R): The Indian Constitution provides for reasonable restrictions on Fundamental Rights in the interest of public order, morality, and national security.

  • (A) Both A and R are true and R is the correct explanation of A.
  • (B) Both A and R are true but R is not the correct explanation of A.
  • (C) A is true but R is false.
  • (D) A is false but R is true.
Correct Answer: (A) Both A and R are true and R is the correct explanation of A.
View Solution




Step 1: Understand Assertion

Fundamental Rights are subject to reasonable restrictions. Hence, they are not absolute.


Step 2: Understand Reason

The Constitution permits restrictions for reasons such as public order, morality, and national security.


Step 3: Relationship

The Reason directly explains why Fundamental Rights are not absolute.
Quick Tip: Fundamental Rights are enforceable but can be restricted under reasonable conditions mentioned in the Constitution (e.g., Article 19).


Question 160:

Choose the correct option:

(1) There are some wrongful acts which come under Civil and Criminal Law both.

(2) Both Criminal and Civil cases can be proved on a balance of probabilities.

  • (A) Only (1) is true
  • (B) Only (2) is true
  • (C) Both (1) and (2) are true
  • (D) Both (1) and (2) are false
Correct Answer: (A) Only (1) is true
View Solution




Statement (1) is true: Certain acts like assault or defamation may involve both criminal and civil liability.


Statement (2) is false: Criminal cases require proof beyond a reasonable doubt, while civil cases follow the "balance of probabilities" standard.
Quick Tip: - Civil cases = balance of probabilities
- Criminal cases = beyond reasonable doubt
Some acts (e.g., fraud, defamation) can fall under both laws.


Question 161:

Intestate Succession determines the mode of devolution of the property based on the rules of inheritance when a person dies without a ____.

  • (A) Legal Heir
  • (B) Executor
  • (C) Testamentary Will
  • (D) Probate
Correct Answer: (C) Testamentary Will
View Solution




Step 1: Understand Intestate Succession

When a person dies without leaving a valid will, their estate is distributed as per succession laws.


Step 2: Evaluate options

- Legal Heir: Refers to the beneficiary, not the absence of a will.

- Executor: Carries out the instructions of a will.

- Testamentary Will: Its absence leads to intestate succession.

- Probate: Legal proof of a will—not relevant if no will exists.
Quick Tip: “Intestate” = dying without a will.
“Testamentary” = related to a will. Intestate succession laws apply when no testamentary will is left behind.


Question 162:

Choose the correct option based on the statements given:


Statement (1): ‘Nemo judex in causa sua’ means to hear the other party or no one should be condemned unheard.

Statement (2): 'Audi alteram partem’ means no one should be made a judge in his own cause and the rule against bias.

  • (A) Only (1) is true
  • (B) Only (2) is true
  • (C) Both (1) and (2) are true
  • (D) Both (1) and (2) are false
Correct Answer: (D) Both (1) and (2) are false
View Solution




Step 1: Understand the legal maxims

- “Nemo judex in causa sua” means "no one should be a judge in his own cause." It reflects the principle against bias in justice.

- “Audi alteram partem” means "hear the other side" or "no one should be condemned unheard." It reflects the right to a fair hearing.


Step 2: Analyze statements

- Statement (1) confuses the meaning of “nemo judex in causa sua” with that of “audi alteram partem.” Hence, it is false.

- Statement (2) reverses the correct meaning too, attributing the rule against bias to “audi alteram partem.” Hence, false.
Quick Tip: - \textbf{Nemo judex in causa sua} = No one should judge their own case.
- \textbf{Audi alteram partem} = Everyone has the right to be heard.
These are principles of natural justice.


Question 163:

The jurisdiction of a court refers to the power or extent of the authority of the court to administer justice, with reference to

  • (A) the local limits
  • (B) the subject matter of litigation
  • (C) the pecuniary value
  • (D) all of the above
Correct Answer: (D) all of the above
View Solution




Step 1: Understanding Jurisdiction

Jurisdiction refers to the authority granted to a legal body to administer justice within a defined field.


Types of jurisdiction:

- Territorial (local limits): Geographical boundary within which a court can operate.

- Subject-matter jurisdiction: The type of cases a court can hear (civil, criminal, family, etc.).

- Pecuniary jurisdiction: The monetary limits under which a court can hear cases.
Quick Tip: Court jurisdiction depends on three key factors: territorial area, type of case, and monetary value. All must be satisfied for a case to be validly tried.


Question 164:

_________ is the principle allowing a state to act in ways that would normally be illegal in emergency or exigent circumstances:

  • (A) Principle of Legality
  • (B) Doctrine of Necessity
  • (C) Rule of Law
  • (D) Doctrine of Proportionality
Correct Answer: (B) Doctrine of Necessity
View Solution




The Doctrine of Necessity is a legal principle which allows a government or authority to take actions that are technically illegal but necessary to preserve order or prevent greater harm in emergency situations.


Step 1: Understand the context

This principle is often invoked to justify extraordinary measures during states of emergency or when normal procedures cannot be followed.


Step 2: Evaluate options

- Principle of Legality: Requires actions to be lawful—opposite of necessity.

- Rule of Law: Calls for adherence to law—conflicts with exceptions.

- Doctrine of Proportionality: Concerns fairness in legal responses, not emergency powers.
Quick Tip: The Doctrine of Necessity is used when urgent action is needed to prevent danger or chaos, even if the action temporarily violates standard legal norms.


Question 165:

In the following situation, what offense can be attributed to Ramu?

Ramu ascends to the top of a building and threatens to jump off, but is prevented from doing so.

  • (A) Cheating
  • (B) Attempt to murder
  • (C) Attempt to suicide
  • (D) No offense
Correct Answer: (C) Attempt to suicide
View Solution




Step 1: Understand the scenario

Ramu has taken a concrete step toward ending his life, which constitutes an "attempt" even though he is stopped.


Step 2: Legal reference

Under Section 309 of the Indian Penal Code, attempt to commit suicide was earlier punishable. Though the Mental Healthcare Act, 2017 has decriminalized it to an extent, it is still legally recognized as an offense unless covered by protection under the Act.


Step 3: Eliminate wrong options

- Cheating: Involves deception for personal gain—irrelevant.

- Attempt to murder: Would require intent to kill someone else.

- No offense: Incorrect as the act falls under "attempt."
Quick Tip: "Attempt to suicide" is still legally recognized, though not always punished—look out for changes under mental health laws.


Question 166:

Choose the correct option based on the statements below:


Statement 1: Summons case are cases relating to an offence punishable with imprisonment of less than three years.

Statement 2: In summons cases, the magistrate holds a regular trial only after framing the charge.

  • (A) Only statement 1 is true
  • (B) Only statement 2 is true
  • (C) Both statements are true
  • (D) Both statements are false
Correct Answer: (D) Both statements are false
View Solution




Statement 1: False

Summons cases usually relate to offences punishable with imprisonment not exceeding two years, not three.


Statement 2: False

In summons cases, the procedure is simplified, and framing of formal charge is not required. The trial begins directly after explaining the particulars of the offence.
Quick Tip: - Summons cases: Minor offences, simpler procedures, no formal charge.
- Warrant cases: More serious, involve formal charges and detailed trial.


Question 167:

Choose the correct option based on the statements below:


Statement 1: A crime is a deliberate or reckless act that causes harm to another - either to his/her person or to his/her property.

Statement 2: In Criminal Law, an individual may report a crime but can never file a case against another individual, only the government can file the case against the offender.

  • (A) Only statement 1 is true
  • (B) Only statement 2 is true
  • (C) Both statements are true
  • (D) Both statements are false
Correct Answer: (A) Only statement 1 is true
View Solution




Statement 1: True

A crime is a wrongful act that causes harm or injury to a person or property, often involving intent or recklessness.


Statement 2: False

While the government prosecutes criminal offenses, an individual (the complainant) can initiate criminal proceedings by filing a First Information Report (FIR) or complaint. The government takes over prosecution, but the individual can still trigger the process.
Quick Tip: Crimes affect public order and are prosecuted by the state, but individuals can initiate complaints that lead to prosecution.


Question 168:

Situation: Shyam is a police officer executing a warrant of arrest against Vijay. Shyam asks Hari to identify "Mr. Vijay". Hari deliberately tells Shyam that Ram is "Vijay" and consequently Ram is arrested.

What is the offence committed by Hari?

  • (A) Abetment by Instigation
  • (B) Abetment by Aiding
  • (C) Abetment by False representation
  • (D) Abetment by Mischief
Correct Answer: (C) Abetment by False representation
View Solution




Step 1: Understand the act

Hari knowingly misleads a police officer, resulting in wrongful arrest of another person.


Step 2: Legal classification

This fits the definition of abetment by false representation, as Hari deliberately caused harm by deceiving the officer.


Other options:

- Instigation involves encouraging someone to commit an offence.

- Aiding means helping physically or logistically.

- Mischief involves damaging property—not applicable here.
Quick Tip: Abetment by false representation occurs when someone deceives an authority by misidentification or lies that lead to an unlawful act.


Question 169:

Which among the following is the difference between a tort and a crime?

  • (A) Tort is an infringement of the private right belonging to an individual. A crime is an invasion of public rights or duties affecting the whole society.
  • (B) In tort, the civil action is brought by the injured party himself in the civil court; whereas in crime, the proceedings are taken and conducted in the name of the state in criminal court.
  • (C) In tort, the wrongdoer has to pay damages to the injured party. In crime, he is punished by the state and the fine imposed on him goes to the Government treasury.
  • (D) All of the above
Correct Answer: (D) All of the above
View Solution




All statements correctly distinguish torts from crimes:

- Torts affect individual rights and result in compensation.

- Crimes affect society and result in state prosecution and punishment.
Quick Tip: Torts = Private wrongs (civil remedies)
Crimes = Public wrongs (state punishment)


Question 170:

Which of the following disputes can be resolved through Alternative Dispute Resolution (ADR)?

  • (A) Matters involving criminal questions, or question of public laws.
  • (B) Matrimonial matters, like divorce, maintenance or custody of child.
  • (C) Cases involving recovery of money.
  • (D) Dissolution of an incorporated Company
Correct Answer: (C) Cases involving recovery of money
View Solution




Step 1: Understand ADR

Alternative Dispute Resolution methods (like mediation, arbitration, and conciliation) are used to resolve civil and commercial disputes out of court.


Step 2: Evaluate options

- Option A: Criminal matters cannot be resolved via ADR.

- Option B: Certain matrimonial matters may go to mediation, but not full ADR for binding resolution.

- Option C: Monetary disputes are commonly resolved through ADR.

- Option D: Dissolution of a company requires statutory/legal proceedings.
Quick Tip: ADR is best suited for civil/commercial cases, including money recovery, contract disputes, and property claims. Not for criminal or constitutional issues.


Question 171:

Consider the following statements:

1. Any retired judge of the Supreme Court of India can be requested to act as a Judge of the Supreme Court by the Chief Justice of India with prior permission of the President of India.

2. A High court in India has the power to review its own judgement as the Supreme Court does.

Which of the statements given above is/are correct?

  • (A) 1 only
  • (B) 2 only
  • (C) Both 1 and 2
  • (D) Neither 1 nor 2
Correct Answer: (C) Both 1 and 2
View Solution




Statement 1: True

Under Article 128 of the Constitution of India, the Chief Justice of India may request a retired Supreme Court judge to sit and act as a judge of the Supreme Court with the prior consent of the President.


Statement 2: True

Though not explicitly mentioned in the Constitution, High Courts have the inherent power to review their judgments under Article 226, much like the Supreme Court under Article 137.
Quick Tip: Both the Supreme Court and High Courts have mechanisms for review of judgments, and retired judges may be recalled with presidential approval for judicial work.


Question 172:

Choose the correct option:

(1) The National Security Act allows the state to detain a person without a formal charge and without trial to prevent them from acting in any manner prejudicial to the security of the state or for the maintenance of public order.

(2) The Indian Constitution allows both preventive detention and the right of protection against arrest and detention in certain cases, as stated in Article 22.

(3) Article 22(3) of the Indian Constitution states that the rights available to an arrested person are not applicable in the case of preventive detention.

  • (A) Only (1) and (2) are true
  • (B) Only (2) and (3) are true
  • (C) Both (1) and (3) are true
  • (D) (1), (2) and (3) are true
Correct Answer: (D) (1), (2) and (3) are true
View Solution




Statement 1: True

The National Security Act (NSA), 1980 authorizes preventive detention to safeguard national security and public order, without formal charges or trial.


Statement 2: True

Article 22 of the Constitution provides protection against arrest but also allows preventive detention under certain conditions.


Statement 3: True

Article 22(3) specifically mentions that certain rights like being informed of grounds for arrest or consulting a lawyer are not applicable in cases of preventive detention.
Quick Tip: Preventive detention laws override some fundamental rights temporarily, especially under Article 22(3). Learn the exceptions for constitutional protections.


Question 173:

Choose the correct option based on the statements given:


(1) Procedure Established by Law is a principle which states that a law is considered valid if it has been properly enacted by the relevant authority, following the correct procedure. Whereas Due Process of Law is a concept that ensures any law in question is fair and not arbitrary.


(2) It was in the case of A. K. Gopalan vs State of Madras, the Supreme Court of India determined that the phrase "procedure established by law" does not refer to any arbitrary procedure set by the Parliament. Instead, it should be interpreted in line with the "due process", which emphasizes fairness and the protection of fundamental rights.

  • (A) Only (1) is true
  • (B) Only (2) is true
  • (C) Both (1) and (2) are true
  • (D) Both (1) and (2) are false
Correct Answer: (A) Only (1) is true
View Solution




Statement 1: True

The term "Procedure Established by Law" is derived from Article 21 of the Indian Constitution. It means that if a law has been duly enacted by a competent legislature, then it is valid, regardless of whether it is just or unjust. The Indian Constitution initially adopted this principle from the British legal system, which focuses more on the existence of a law rather than its fairness.


Statement 2: False

The case of A.K. Gopalan v. State of Madras (1950) actually upheld the strict interpretation of "Procedure Established by Law" and rejected the American concept of "Due Process of Law." The court stated that any law passed by the legislature would be valid as long as it followed the prescribed procedure, even if the law was arbitrary or unfair.

It was only later, in the Maneka Gandhi v. Union of India case (1978), that the Supreme Court broadened the interpretation of Article 21 and held that the procedure must be "just, fair, and reasonable," thereby incorporating elements of "Due Process of Law." Quick Tip: "Procedure Established by Law" checks if the law is properly enacted.
"Due Process of Law" checks both enactment and fairness.
India originally followed the former but later moved towards including the latter via judicial interpretation.


Question 174:

__________ is the Latin term that signifies 'to stand by past decisions or precedents and not to disturb the settled points'?

  • (A) Habeas Corpus
  • (B) Stare Decisis
  • (C) Pro Bono
  • (D) Res Ipsa Loquitur
Correct Answer: (B) Stare Decisis
View Solution




"Stare Decisis" is a Latin legal principle that means "to stand by things decided." It emphasizes the importance of following precedent in legal decisions. This principle ensures consistency, stability, and predictability in the legal system.


Under this doctrine, courts are expected to follow earlier judicial decisions when the same points arise again in litigation. It helps lower courts to follow rulings made by higher courts in the same jurisdiction.


Explanation of other options:

- Habeas Corpus: A writ used to bring a person before a court to determine if their detention is lawful.

- Pro Bono: Legal work done voluntarily and free of charge.

- Res Ipsa Loquitur: "The thing speaks for itself"—used in tort cases where negligence is inferred from the nature of the accident. Quick Tip: "Stare Decisis" ensures legal certainty by following past judgments. It is a foundation of common law systems like India's.


Question 175:

The branch of law that regulates the organization and functioning of the State and determines the relationship of the State is known as __________.

  • (A) Civil law
  • (B) Substantive law
  • (C) International law
  • (D) Public law
Correct Answer: (D) Public law
View Solution




Public law governs the relationship between individuals and the state. It includes constitutional law, administrative law, and criminal law. It deals with issues that affect the public at large, including governmental authority, legal obligations of public officials, and the functioning of public institutions.


Explanation of other options:

- Civil law: Concerned with disputes between individuals (e.g., contracts, torts, property disputes).

- Substantive law: Defines rights and duties (both civil and criminal) but doesn’t specifically regulate the State's structure.

- International law: Governs relations between nations, not internal state functioning.
Quick Tip: Public law = governs State and public institutions.
Civil law = governs disputes between private individuals.
Administrative + Constitutional + Criminal law = part of public law.


Question 176:

Choose the correct option based on the statements given:

1. Indian Judiciary is a single integrated and unified system of Courts for the Union as well as the States.

2. Article 131 gives both the Supreme Court and High Courts the original jurisdiction in a dispute between one State and another, or between a group of States and others.

  • (A) Only (1) is true
  • (B) Only (2) is true
  • (C) Both (1) and (2) are true
  • (D) Both (1) and (2) are false
Correct Answer: (A) Only (1) is true
View Solution



Statement 1: True — The Indian judiciary is a unified system. Though we have state-level High Courts and subordinate courts, all courts are part of a single integrated judicial system under the Supreme Court.

Statement 2: False — Article 131 of the Constitution provides original jurisdiction only to the Supreme Court, not to the High Courts, in disputes between States or between the Centre and States.



\begin{quicktipbox
Remember: Only the Supreme Court has original jurisdiction in inter-state disputes under Article 131.
\end{quicktipbox Quick Tip: Remember: Only the Supreme Court has original jurisdiction in inter-state disputes under Article 131.


Question 177:

There are various wrongs that find their place both under criminal law and the law of torts. These wrongs are:

  • (A) assault, defamation and negligence.
  • (B) assault, defamation, negligence, conspiracy and nuisance.
  • (C) assault, defamation, negligence and conspiracy.
  • (D) defamation, conspiracy and negligence.
Correct Answer: (B) assault, defamation, negligence, conspiracy and nuisance
View Solution



Certain wrongs are considered both civil and criminal in nature. For example:

- Assault: It is a civil wrong (tort) and also punishable under IPC.

- Defamation: Can be pursued as a tort for damages or prosecuted under IPC.

- Negligence: Typically civil, but can be criminal if gross or willful.

- Conspiracy and Nuisance: Have implications under both branches depending on context.



\begin{quicktipbox
Some acts like defamation or assault can lead to both criminal prosecution and civil suits for damages.
\end{quicktipbox Quick Tip: Some acts like defamation or assault can lead to both criminal prosecution and civil suits for damages.


Question 178:

Which of the following is not a true statement in the context of Transfer of Property Act?

  • (A) The property that is going to be transferred should be free from encumbrances
  • (B) In a transfer of property, the transfer should be between two or more living persons.
  • (C) The transfer may be an unlawful object or an unlawful consideration
  • (D) The transferor who transfers the property must be entitled to the transferable property
Correct Answer: (C) The transfer may be an unlawful object or an unlawful consideration
View Solution



Under Section 6 of the Transfer of Property Act, a transfer of property must involve a lawful object and consideration. Any transfer involving unlawful object or consideration is void.

Thus, option C contradicts the legal provision and is false.

The other options reflect valid principles:

- Property may be encumbered, but it should be clearly disclosed.

- Transfer must be between two living persons.

- Transferor must have title or legal authority to transfer.



\begin{quicktipbox
A transfer with an unlawful object or purpose is void under the Transfer of Property Act. Always check legality of purpose.
\end{quicktipbox Quick Tip: A transfer with an unlawful object or purpose is void under the Transfer of Property Act. Always check legality of purpose.


Question 179:

A characteristic of a valid 'Custom' in jurisprudence is ________.

  • (A) Non-conformity
  • (B) Continuity
  • (C) Modernity
  • (D) Uncertainty
Correct Answer: (B) Continuity
View Solution



A valid legal custom must fulfill several criteria, and one of the most important is continuity — it must have been practiced consistently over a long time.

Other criteria include: reasonableness, certainty, and conformity with law.

- Non-conformity and uncertainty negate legal validity.

- Modernity is not required; in fact, ancient practices are often accepted if continuous.



\begin{quicktipbox
Customs must be continuous, certain, and reasonable to be valid legal sources in jurisprudence.
\end{quicktipbox Quick Tip: Customs must be continuous, certain, and reasonable to be valid legal sources in jurisprudence.


Question 180:

Correct order of stages of crime are

  • (A) Commission, Intention, Preparation, Attempt
  • (B) Intention, Preparation, Attempt, Commission
  • (C) Preparation, Commission, Intention, Attempt
  • (D) Intention, Preparation, Commission, Attempt
Correct Answer: (B) Intention, Preparation, Attempt, Commission
View Solution



The four stages in the commission of a crime are:

1. Intention: The mental decision to commit a crime.

2. Preparation: Making necessary arrangements (e.g., buying weapons).

3. Attempt: Direct movement towards the execution of the crime.

4. Commission: The actual completion of the criminal act.



\begin{quicktipbox
Crimes follow a sequence: Intention → Preparation → Attempt → Commission. Law often punishes attempt and commission; intention alone is not punishable unless accompanied by acts.
\end{quicktipbox Quick Tip: Crimes follow a sequence: Intention → Preparation → Attempt → Commission. Law often punishes attempt and commission; intention alone is not punishable unless accompanied by acts.


Question 181:

Lapsed gift means a gift in which a person whose favour a will was created dies...

  • (A) Before the death of his legal heir
  • (B) After the death of his legal heir
  • (C) Before the death of the testator
  • (D) After the death of the testator
Correct Answer: (C) Before the death of the testator
View Solution



In the context of wills and testaments, a lapsed gift refers to a situation where a gift mentioned in a will cannot be transferred because the beneficiary dies before the person who created the will (i.e., the testator).


Why this happens:

When a will is executed, the gifts come into effect only upon the testator’s death. If the intended recipient is not alive at that time, they cannot receive the gift. Unless the will has a provision for substitution (i.e., alternate beneficiary), the gift fails and is said to “lapse.”


Example:

If A creates a will giving his car to his friend B, but B dies before A, then the gift of the car lapses.


Options explained:

- (A) and (B) talk about the legal heir — irrelevant in the context of lapsed gifts.

- (D) is incorrect because if the beneficiary dies after the testator, the gift is valid and goes to the beneficiary’s estate.



\begin{quicktipbox
A lapsed gift occurs when the beneficiary dies before the testator. The gift fails unless a substitute is named.
\end{quicktipbox Quick Tip: A lapsed gift occurs when the beneficiary dies before the testator. The gift fails unless a substitute is named.


Question 182:

Interlocutory Application is a petition seeking:

  • (A) Final judgment and resolution of a case
  • (B) Revision of a court's decision
  • (C) Temporary relief during the pendency of a case
  • (D) Dismissal of a case without trial
Correct Answer: (C) Temporary relief during the pendency of a case
View Solution



An Interlocutory Application (IA) is a motion or petition filed in a pending case to seek relief that is temporary in nature and necessary for the fair and efficient handling of the case.


Examples:

- Seeking interim injunctions

- Requesting a stay on proceedings

- Seeking temporary custody in family matters


These are filed before the final resolution of the case and help preserve the rights or maintain status quo until the main matter is decided.


Why other options are incorrect:

- (A) refers to final judgment, not interim relief.

- (B) is related to appellate or revisional jurisdiction.

- (D) deals with dismissal, not temporary relief.



\begin{quicktipbox
Interlocutory Applications are used to seek temporary relief or direction while the case is still ongoing.
\end{quicktipbox Quick Tip: Interlocutory Applications are used to seek temporary relief or direction while the case is still ongoing.


Question 183:

A confession made voluntarily by an accused person in a court of law under Section 164 of the CrPC is called:

  • (A) Coerced confession
  • (B) Witness confession
  • (C) Judicial confession
  • (D) Retracted confession
Correct Answer: (C) Judicial confession
View Solution



Section 164 of the Criminal Procedure Code (CrPC) allows a magistrate to record confessions and statements during investigation. If the accused voluntarily confesses before a magistrate, it is known as a judicial confession.


Conditions for validity:

- Must be made voluntarily (no police pressure).

- Magistrate must explain that the accused is not bound to confess.

- The magistrate must ensure the accused understands the consequences.


Types of Confessions:

- Judicial confession: Made before a magistrate.

- Extra-judicial confession: Made to any person other than a magistrate.


Incorrect options:

- (A) Coerced confession is involuntary and inadmissible.

- (B) Witness confession is not a legal term.

- (D) Retracted confession is one that is later withdrawn.



\begin{quicktipbox
A judicial confession under Section 164 CrPC is made voluntarily before a magistrate and carries high evidentiary value.
\end{quicktipbox Quick Tip: A judicial confession under Section 164 CrPC is made voluntarily before a magistrate and carries high evidentiary value.


Question 184:

What is vicarious liability?

  • (A) Liability imposed on a person for their own intentional wrongdoing
  • (B) Liability imposed on an employer for the wrongful actions of their employees
  • (C) Liability imposed on a person for their negligence in fulfilling a duty of care
  • (D) Liability imposed on a person for their involvement in a criminal offense
Correct Answer: (B) Liability imposed on an employer for the wrongful actions of their employees
View Solution



Vicarious liability is a principle in tort law where one person is held legally responsible for the actions of another. This usually occurs in employer-employee relationships.


If an employee commits a tort in the course of their employment, the employer can be held liable even if the employer was not directly involved.


Key requirement: The wrongful act must be committed in the course of employment.


Why others are incorrect:

- (A) refers to personal liability.

- (C) concerns negligence, not vicarious liability.

- (D) relates to criminal law, not civil vicarious liability.



\begin{quicktipbox
Under vicarious liability, employers are held responsible for torts committed by employees in the scope of employment.
\end{quicktipbox Quick Tip: Under vicarious liability, employers are held responsible for torts committed by employees in the scope of employment.


Question 185:

What does the term ``restitution" refer to?

  • (A) The punishment imposed on a defendant for a criminal offense
  • (B) The act of the restoration of something lost or stolen to its proper owner
  • (C) The process of resolving disputes through negotiation and compromise
  • (D) The legal document that initiates a lawsuit or legal action
Correct Answer: (B) The act of the restoration of something lost or stolen to its proper owner
View Solution



Restitution is a legal remedy that involves restoring a person to their original position by returning what was lost or compensating them for the loss.


It is commonly used in both civil and criminal proceedings:

- In civil law: Restores unjust enrichment or returns property.

- In criminal law: Courts may order an offender to pay restitution to the victim.


Why others are incorrect:

- (A) refers to punishment, not restoration.

- (C) defines mediation/conciliation, not restitution.

- (D) refers to a plaint or legal petition, not restitution.



\begin{quicktipbox
Restitution means returning the wrongfully taken item or money to its rightful owner, often used as a legal remedy.
\end{quicktipbox Quick Tip: Restitution means returning the wrongfully taken item or money to its rightful owner, often used as a legal remedy.


Question 186:

Ramu keeps his mobile inside the bag of Sheela, with the intention that it may be found in the bag and the circumstance leads to Sheela being convicted of theft. Ramu has committed the offence of:

  • (A) Fabricating evidence
  • (B) Theft
  • (C) Conspiracy
  • (D) Obstruction of justice
Correct Answer: (A) Fabricating evidence
View Solution



Ramu planted his mobile in Sheela’s bag to falsely implicate her in a crime. This act is classified under Section 192 of the Indian Penal Code (IPC), which defines "fabricating false evidence" as intentionally creating a false circumstance to mislead the court.


According to IPC, when a person causes a false situation to exist or makes false entries with the intent to mislead a judge or public servant, it constitutes fabricating evidence. Ramu’s conduct squarely fits this definition.



\begin{quicktipbox
Planting false items to frame someone is punishable as "fabrication of evidence" under IPC.
\end{quicktipbox Quick Tip: Planting false items to frame someone is punishable as "fabrication of evidence" under IPC.


Question 187:

Name of Union given in the Constitution is

  • (A) India i.e. Hindustan
  • (B) India i.e. Bharat
  • (C) Hindustan i.e. Bharat
  • (D) India i.e. Bharatvarsha
Correct Answer: (B) India i.e. Bharat
View Solution



Article 1 of the Constitution of India clearly states:

"India, that is Bharat, shall be a Union of States."


This constitutional provision affirms the dual naming convention of the country. No other synonyms like “Hindustan” or “Bharatvarsha” are used in the Constitution.



\begin{quicktipbox
Refer to Article 1 of the Indian Constitution for the official name: “India, that is Bharat.”
\end{quicktipbox Quick Tip: Refer to Article 1 of the Indian Constitution for the official name: “India, that is Bharat.”


Question 188:

Term used to describe an accused who seeks pardon from the court by agreeing to testify against all others involved in the crime is

  • (A) Accomplice
  • (B) Informant
  • (C) Co-conspirator
  • (D) Approver
Correct Answer: (D) Approver
View Solution



An approver is an accused person who is granted pardon on the condition that they will give full and true disclosure about the crime and help the prosecution. This is provided under Section 306 of the Criminal Procedure Code (CrPC).


The term is used specifically in legal parlance to distinguish a person who turns against co-accused in return for leniency.



\begin{quicktipbox
An “approver” is an accused who testifies for the prosecution in exchange for pardon under CrPC.
\end{quicktipbox Quick Tip: An “approver” is an accused who testifies for the prosecution in exchange for pardon under CrPC.


Question 189:

The communication of a proposal is considered complete when it is

  • (A) Accepted by the intended recipient
  • (B) Is in course of transmission
  • (C) Sent by the proposer
  • (D) Comes to the knowledge of the intended recipient
Correct Answer: (D) Comes to the knowledge of the intended recipient
View Solution



According to Section 4 of the Indian Contract Act, 1872, the communication of a proposal is said to be complete when it comes to the knowledge of the person to whom it is made.


This means that even if a proposal is sent, it is not effective unless the receiver becomes aware of it.


Illustration:

If A posts a letter of proposal to B, the proposal is complete only when B receives and reads the letter.



\begin{quicktipbox
For a valid contract, the proposal must come to the knowledge of the other party — mere sending is not enough.
\end{quicktipbox Quick Tip: For a valid contract, the proposal must come to the knowledge of the other party — mere sending is not enough.


Question 190:

Choose the correct answer based on the following statements:

1. All void contracts are not necessarily illegal.

2. All illegal contracts are void.

  • (A) Only (1) is true
  • (B) Only (2) is true
  • (C) Both (1) and (2) are true
  • (D) Both (1) and (2) are false
Correct Answer: (C) Both (1) and (2) are true
View Solution



Statement 1 is true: A void contract is one that is unenforceable by law (e.g., due to lack of consideration), but it is not necessarily illegal.


Statement 2 is also true: All illegal contracts (such as those involving fraud, or against public policy) are void ab initio and cannot be enforced by law.


Summary:

- Illegal contracts are always void.

- Void contracts are not always illegal (they could be due to technical defects).



\begin{quicktipbox
Void \(\neq\) Illegal always. Illegal = Void always. Distinction matters for enforceability and collateral agreements.
\end{quicktipbox Quick Tip: Void \(\neq\) Illegal always. Illegal = Void always. Distinction matters for enforceability and collateral agreements.


Question 191:

Power to grant pardon is a/an __________ power

  • (A) Quasi Judicial
  • (B) Judicial
  • (C) Legislative
  • (D) Executive
Correct Answer: (D) Executive
View Solution



The power to grant pardon is enshrined in Article 72 of the Constitution of India for the President and Article 161 for the Governor.

This power enables the President or Governor to grant pardons, reprieves, respites, or remissions of punishment, or to suspend, remit, or commute the sentence of any person convicted of any offence.

Though it affects judicial decisions, this is not a judicial function — it is a sovereign and discretionary act performed by the head of the Executive.

Hence, it is classified under Executive powers and is based on principles of justice, humanity, and public welfare.


\begin{quicktipbox
Pardon powers are not judicial reviews; they are mercy powers granted to the Executive for correcting judicial errors or for humanitarian relief.
\end{quicktipbox Quick Tip: Pardon powers are not judicial reviews; they are mercy powers granted to the Executive for correcting judicial errors or for humanitarian relief.


Question 192:

Legal term for a right or claim that is enforceable against a particular thing or property rather than against a specific person

  • (A) Right in rem
  • (B) Right in personam
  • (C) Locus standi
  • (D) Fundamental Right
Correct Answer: (A) Right in rem
View Solution



``Right in rem" refers to a right enforceable against the whole world. It relates to ownership and possession of property.

For example, an owner's right to exclude everyone else from using his land is a right in rem.

This is distinct from "right in personam", which is enforceable only against a specific person (e.g., a contractual obligation).

``Locus standi" means the right to approach the court, and ``Fundamental Right" is a constitutional guarantee — neither directly answers the question.


\begin{quicktipbox
"Right in rem" = enforceable against the world (property rights); "Right in personam" = enforceable against an individual.
\end{quicktipbox Quick Tip: "Right in rem" = enforceable against the world (property rights); "Right in personam" = enforceable against an individual.


Question 193:

Ownership does not connote

  • (A) Right to use and enjoy
  • (B) Right to Destroy
  • (C) Right to Dispossess
  • (D) Right to Possess
Correct Answer: (C) Right to Dispossess
View Solution



Ownership refers to a bundle of rights which typically includes:

- Right to possess

- Right to use and enjoy

- Right to destroy or alienate

- Right to exclude others

However, ``right to dispossess” means taking away another's property, which is not a right granted by ownership — it may imply illegality unless it’s through lawful authority (e.g., state repossession).

Hence, it is not a valid incident of ownership.


\begin{quicktipbox
Ownership grants rights over one’s own property — not over others’ (i.e., no right to dispossess others unlawfully).
\end{quicktipbox Quick Tip: Ownership grants rights over one’s own property — not over others’ (i.e., no right to dispossess others unlawfully).


Question 194:

Mr. Hariharan professes to gift his property at Calicut worth 1 lakh to Ms. Shyamala and by the same instrument another property at Kochi for 5 lakh. As Ms. Shyamala can stay only at one place among the two:

  • (A) She may retain the property at Calicut and reject the transfer of property at Kochi.
  • (B) She has to retain the property at Kochi if she wants to retain the property at Calicut
  • (C) Both Option 1 and Option 2 are true
  • (D) Both Option 1 and Option 2 are false
Correct Answer: (B) She has to retain the property at Kochi if she wants to retain the property at Calicut
View Solution



This is a case of a composite or conditional transfer. Under the Indian Contract Act and Transfer of Property Act, if two transfers are made through a single transaction or document, they may be conditional upon each other.

Here, the gift of the Calicut property (worth less) appears dependent on the acceptance of the Kochi property.

Hence, if Ms. Shyamala accepts one, she must accept the other, unless explicitly stated otherwise in the document.


\begin{quicktipbox
In composite transfers, rejecting one part may nullify the whole. Accepting one benefit might bind you to another obligation.
\end{quicktipbox Quick Tip: In composite transfers, rejecting one part may nullify the whole. Accepting one benefit might bind you to another obligation.


Question 195:

According to the Montevideo Convention of 1933, what are the essential requirements for a state?

  • (A) A state must possess a permanent population, defined territory, government, and the capacity to enter into relations with other states.
  • (B) A state must have a defined territory and government, but the permanent population and capacity to enter into relations are not required.
  • (C) A state must have a permanent population and defined territory, but the government and capacity to enter into relations are not necessary.
  • (D) A state must have the capacity to enter into relations with other states, but the permanent population, defined territory, and government are not essential.
Correct Answer: (A) A state must possess a permanent population, defined territory, government, and the capacity to enter into relations with other states.
View Solution



The Montevideo Convention of 1933 is a cornerstone of international law, outlining the declarative theory of statehood.

According to Article 1 of the Convention, a state must fulfill the following criteria:

1. Permanent population

2. Defined territory

3. Government

4. Capacity to enter into relations with other states

These four elements ensure that a state has political organization, identity, and legitimacy in international relations.


\begin{quicktipbox
To qualify as a state in international law, four conditions are mandatory — population, territory, government, and diplomatic capacity.
\end{quicktipbox Quick Tip: To qualify as a state in international law, four conditions are mandatory — population, territory, government, and diplomatic capacity.


Question 196:

Which motion allows for a full discussion on an important and urgent issue in Parliament?

  • (A) Adjournment Motion
  • (B) Calling Attention Motion
  • (C) Half an Hour Discussion
  • (D) No Confidence Motion
Correct Answer: (A) Adjournment Motion
View Solution



An adjournment motion is used to draw the attention of the House to a definite matter of urgent public importance.

Its adoption leads to an interruption of the normal business of the House, and the matter is then discussed in detail.

This motion highlights government negligence or failure and is considered a strong tool in the hands of the opposition.


\begin{quicktipbox
Adjournment Motion leads to a full debate and is used only for urgent and serious issues affecting the nation.
\end{quicktipbox Quick Tip: Adjournment Motion leads to a full debate and is used only for urgent and serious issues affecting the nation.


Question 197:

What actions can the Parliament undertake based on the powers specified in Articles 2 and 3 of the Constitution?

  • (A) Admit new territories as new state of India
  • (B) Split or merge existing Indian states
  • (C) Alter the names and boundaries of existing Indian States
  • (D) All of above
Correct Answer: (D) All of above
View Solution



Articles 2 and 3 of the Indian Constitution empower the Parliament to:

- Admit new states into the Union (Article 2)

- Form new states or alter boundaries, names, or areas of existing states (Article 3)

These articles grant wide discretion to Parliament to reorganize state territories.


\begin{quicktipbox
Remember: Article 2 deals with admitting new territories; Article 3 deals with reorganization of existing states.
\end{quicktipbox Quick Tip: Remember: Article 2 deals with admitting new territories; Article 3 deals with reorganization of existing states.


Question 198:

The National Commission for Protection of Child Rights (NCPCR) is a _______ in India...

  • (A) Tribunal
  • (B) Statutory body
  • (C) Constitutional body
  • (D) Regulatory body
Correct Answer: (B) Statutory body
View Solution



The NCPCR was established by an Act of Parliament called the Commission for Protection of Child Rights Act, 2005.

It is therefore a statutory body, not a constitutional or regulatory one.

Its role is to ensure the protection and rights of children as per Indian laws and international agreements.


\begin{quicktipbox
Statutory bodies are created by laws enacted by Parliament; NCPCR was created under the 2005 Act.
\end{quicktipbox Quick Tip: Statutory bodies are created by laws enacted by Parliament; NCPCR was created under the 2005 Act.


Question 199:

Which of the following is NOT a tort?

  • (A) Negligence
  • (B) Battery
  • (C) Trespass to land
  • (D) Breach of contract
Correct Answer: (D) Breach of contract
View Solution



Torts are civil wrongs that arise independently of contracts. Examples include negligence, battery, and trespass.

A breach of contract, on the other hand, is a violation of the terms agreed upon by parties in a legally binding contract.

Hence, it is governed by contract law, not tort law.


\begin{quicktipbox
Torts involve duties fixed by law, not by agreement — breach of contract involves agreed obligations.
\end{quicktipbox Quick Tip: Torts involve duties fixed by law, not by agreement — breach of contract involves agreed obligations.


Question 200:

Which of the following is the most important principle underlying the writ of habeas corpus?

  • (A) The right to a fair trial
  • (B) The right to be free from arbitrary detention
  • (C) The right to due process of law
  • (D) The right to be represented by counsel
Correct Answer: (B) The right to be free from arbitrary detention
View Solution



The writ of habeas corpus is a legal remedy against unlawful detention. It ensures that a person in custody must be presented before a judge.

Its main purpose is to safeguard individuals from being held without justification.

It ensures liberty by preventing arbitrary or illegal detention by the state.


\begin{quicktipbox
Habeas Corpus = "produce the body" — it’s the most powerful safeguard against illegal or unjustified imprisonment.
\end{quicktipbox Quick Tip: Habeas Corpus = "produce the body" — it’s the most powerful safeguard against illegal or unjustified imprisonment.

Comments


No Comments To Show